Qui si risolve LOGO
a

Menu

M

Chiudi

Massimi e minimi vincolati – Esercizi – Volume 2

Massimi e minimi liberi e vincolati

Home » Massimi e minimi vincolati – Esercizi – Volume 2

In questo secondo volume di esercizi sui massimi e minimi vincolati per funzioni in più variabili presentiamo ulteriori 9 problemi sulla ricerca dei punti di massimo e minimo di una funzione su un insieme compatto D \subset \mathbb{R}^n. Come in Massimi e minimi vincolati – Esercizi – Volume 1, lo studio riguarda sia la parte interna di D, sia particolarmente il bordo \partial D, mediante parametrizzazione o il metodo dei moltiplicatori di Lagrange.

Gli esercizi sono completamente risolti, offrendo spesso più soluzioni alternative per aumentare la varietà di tecniche mostrata al lettore.
Questo secondo volume arricchisce e completa quindi il lavoro contenuto nella prima porzione di esercizi, ed è rivolto a studenti e appassionati.

Segnaliamo il seguente materiale su argomenti correlati:

Buona lettura!

 
 

Autori e revisori


 
 

Introduzione

Leggi...

La seguente dispensa è una collezione di esercizi riguardanti il problema della determinazione del massimo e del minimo di una funzione su un insieme compatto di \mathbb{R}^3. Essa è composta da esercizi di varia difficoltà ed è rivolta a studenti di Ingegneria, Fisica e Matematica che desiderano affrontare l’esame di Analisi 2. Tutti gli esercizi presentano una soluzione convenzionale, mentre alcuni ne hanno altre più ad hoc. L’obiettivo è quello di suggerire allo studente strade alternative per risolvere l’esercizio più velocemente. Gli strumenti utilizzati sono il Teorema di Fermat per quanto riguarda la parte interna dell’insieme, mentre per la sua frontiera adotteremo il metodo della parametrizzazione, qualora possibile, o il metodo dei moltiplicatori di Lagrange, quando il vincolo è espresso come luogo di zeri.

 
 

Richiami di teoria

Leggi...

Prima delle soluzioni, richiamiamo alcuni risultati fondamentali, che saranno utilizzati nella risoluzione degli esercizi:

Teorema 1 (teorema di Weierstrass). Sia K\subset\mathbb{R}^n un insieme compatto e sia f:K\rightarrow\mathbb{R} una funzione continua. Allora f ammette massimo e minimo su K.

Teorema 2 (teorema di Heine-Borel). Un insieme E\subset\mathbb{R}^n è compatto se e solo se è chiuso e limitato.

Teorema 3 (teorema di Fermat). Siano A\subset\mathbb{R}^n un insieme aperto, f:A\rightarrow\mathbb{R} e x_0\in A tale che f è differenziabile in x_0. Allora, se x_0 è un punto di massimo o minimo relativo per f su A, vale \nabla f(x_0)=0.

Corollario 1. Sia f:A\subset\mathbb{R}^n\rightarrow\mathbb{R} una funzione differenziabile su tutto A, insieme aperto. Allora, tutti i punti di massimo o minimo relativo di f su A sono, in particolare, punti critici.

Teorema 4 (teorema dei moltiplicatori di Lagrange). Siano f,g:\mathbb{R}^n\rightarrow\mathbb{R} due funzioni di classe \mathcal{C}^1, sia \Gamma:=\{x\in\mathbb{R}^n\mid g(x)=0\}. Supponiamo che x_0\in\Gamma sia un punto di massimo o minimo di f su \Gamma e che \nabla g(x_0)\neq0, allora \exists\lambda\in\mathbb{R} tale che \nabla f(x_0)=\lambda\nabla g(x_0).

Corollario 2. siano f,g_1,...,g_k:\mathbb{R}^n\rightarrow\mathbb{R} funzioni differenziabili e sia

\begin{equation*}             \Gamma:=\{\overline{x}\in\mathbb{R}^n\mid g_1(\overline{x})=...=g_k(\overline{x})=0\}.         \end{equation*}

Supponiamo che \overline{x}_0\in\Gamma sia punto di massimo o di minimo relativo per f su \Gamma e che i vettori \nabla g_i(\overline{x}_0) non siano linearmente dipendenti. Allora, definita la Lagrangiana \mathcal{L}:\mathbb{R}^n\times\mathbb{R}^k\rightarrow\mathbb{R},

\begin{equation*}             \mathcal{L}(x_1,...,x_n,\lambda_1,...,\lambda_k):=f(x_1,...,x_n)-\lambda_1g_1(x_1,...,x_n)-...-\lambda_kg_k(x_1,...,x_n),         \end{equation*}

vale

\begin{equation*}             \nabla\mathcal{L}(\overline{x}_0,\lambda_1,...,\lambda_k)=(0,...,0),         \end{equation*}

cioè \overline{x}_0 è un punto critico della Lagrangiana.


 
 

Esercizi

\[\quad\]

Esercizio 1  (\bigstar\bigstar\bigstar\largewhitestar\largewhitestar). Data la funzione f:\mathbb{R}^3\rightarrow\mathbb{R}, definita da

\[f(x,y,z):=4x+y+z^2,\]

determinare, se esistono, il massimo e il minimo di f sull’insieme

\[A:=\{(x,y,z)\in\mathbb{R}^3\mid x^2+y^2\leq z\leq 4\}.\]

Svolgimento 1.

massimi e minimi vincolati

Figura 1: l’insieme A.

\[\quad\]

\[\quad\]

Osserviamo che f è un polinomio, pertanto è una funzione continua su \mathbb{R}^3, mentre l’insieme A è chiuso e limitato. La chiusura segue dal fatto che A è intersezione di controimmagini di insiemi chiusi di \mathbb{R} tramite funzioni continue, mentre per la limitatezza notiamo che esso è un settore di paraboloide ellittico, contenuto in una sfera di raggio 2\sqrt{5}. Infatti, dalle equazioni definenti A, per ogni (x,y,z)\in A risulta

\begin{equation*} ||(x,y,z)||=\sqrt{x^2+y^2+z^2}\leq\sqrt{z+z^2}\leq\sqrt{4+16}=2\sqrt{5}. \end{equation*}

Dunque è un chiuso e limitato di \mathbb{R}^3, compatto per il teorema di Heine-Borel. Possiamo allora applicare il teorema di Weierstrass, il quale garantisce l’esistenza del massimo e del minimo di f su A.

Distinguiamo l’insieme A nella sua parte interna e la sua frontiera: A=A^\circ\sqcup\partial A. Studiamo separatamente e in maniera differente le due componenti.

A^\circ) Essendo f differenziabile sull’insieme aperto A^\circ, il teorema di Fermat ci assicura che gli estremi relativi di f in A^\circ vanno ricercati tra i suoi eventuali punti critici in A^\circ, ovvero i punti P\in A^\circ tali che \nabla f(P)=(0,0,0) e scartare tutti i punti che non soddisfano tale condizione. Calcoliamo il gradiente di f svolgendo le derivate parziali:

\[\begin{cases} \dfrac{\partial f}{\partial x}(x,y)=4\\\\ \dfrac{\partial f}{\partial y}(x,y)=1\\\\ \dfrac{\partial f}{\partial z}(x,y)=2z, \end{cases}\]

dunque \nabla f(x,y,z)=(4,1,2z), che non si annulla per nessuna scelta di (x,y,z)\in A^\circ. Ciò significa che non vi sono punti critici.

\partial A) La frontiera di A sarà descritta dalle equazioni, le cui disequazioni associate definiscono A. Dunque

\begin{equation*}     \partial A=\left\{(x,y,z)\in\mathbb{R}^3\mid x^2+y^2=z, 0\leq z\leq4\right\}\cup\left\{(x,y,z)\in\mathbb{R}^3\mid z=4, x^2+y^2\leq z\right\}. \end{equation*}

Possiamo distinguere \partial A in due componenti: \partial A=B\cup C, dove

\begin{equation*}     B=\left\{(x,y,z)\in\mathbb{R}^3\mid x^2+y^2=z, 0\leq z\leq4\right\} \end{equation*}

e

\begin{equation*}     C=\left\{(x,y,z)\in\mathbb{R}^3\mid z=4, x^2+y^2\leq z\right\}. \end{equation*}

\[\quad\]

\[\quad\]

\[\quad\]

massimi e minimi vincolati

Figura 2: Gli insiemi B e C che compongono la frontiera di A.

\[\quad\]

\[\quad\]

A questo punto, studiamo il comportamento di f su B e C: dovremo trovare due parametrizzazioni \varphi di B e \psi di C e studiare massimi e minimi delle funzioni composte f\circ\varphi e f\circ\psi sui loro rispettivi domini.

B) È immediato osservare che B è unione di circonferenze di raggio \sqrt{z} a quota z, per z\in[0,4]. Dunque, una sua possibile parametrizzazione è la seguente:

\begin{equation*}     \begin{array}{rcl}          \varphi:[0,2\pi]\times[0,2]& \rightarrow&\mathbb{R}^3 \\         (\theta,\rho) & \mapsto&(\rho\cos\theta,\rho\sin\theta,\rho^2).     \end{array} \end{equation*}

\[\quad\]

\[\quad\]

\[\quad\]

massimi e minimi vincolati

\[\quad\]

\[\quad\]

Calcoliamo allora (f\circ\varphi)(\theta,\rho):

\begin{equation*}     f(\varphi(\theta,\rho))=f(\rho\cos\theta,\rho\sin\theta,\rho^2)=4\rho\cos\theta+\rho\sin\theta+\rho^4. \end{equation*}

Sia R:=[0,2\pi]\times[0,2] il dominio della parametrizzazione. Osserviamo che f\circ\varphi è somma e prodotto di funzioni continue e R è un chiuso e limitato di \mathbb{R}^2, dunque compatto. Per il teorema di Weierstrass, esisteranno il massimo ed il minimo di f\circ\varphi su R. Per trovarli, distinguiamo nuovamente l’insieme nella sua parte interna, R^\circ=(0,2\pi)\times(0,2) e la sua frontiera, formata dai 4 lati del rettangolo.

Per quanto riguarda l’insieme aperto R^\circ, essendo f\circ\varphi una funzione differenziabile, il teorema di Fermat afferma che condizione necessaria affinchè P\in R^\circ sia un punto di massimo o di minimo è che \nabla (f\circ\varphi)(P)=(0,0). Calcoliamo allora il gradiente di f\circ\varphi tramite le derivate parziali:

\[\begin{cases} \dfrac{\partial (f\circ\varphi)}{\partial \theta}(\theta,\rho)=-4\rho\sin\theta+\rho\cos\theta\\\\ \dfrac{\partial (f\circ\varphi)}{\partial \rho}(\theta,\rho)=4\cos\theta+\sin\theta+4\rho^3. \end{cases}\]

Dunque

(1) \begin{equation*}     \nabla (f\circ\varphi)(\theta,\rho)=(0,0)\Leftrightarrow\displaystyle\left\{\begin{array}{l} -4\rho\sin\theta+\rho\cos\theta=0\\\\ 4\cos\theta+\sin\theta+4\rho^3=0 \end{array} \right.\Leftrightarrow\displaystyle\left\{\begin{array}{l} \rho(\cos\theta-4\sin\theta)=0\\\\ 4\cos\theta+\sin\theta+4\rho^3=0. \end{array} \right. \end{equation*}

Poichè \rho\in(0,2), la prima equazione si annulla solamente se \cos\theta=4\sin\theta. Da tale relazione è possibile recuperare il valore di \sin\theta e \cos\theta risolvendo il sistema

\begin{equation*}     \left\{\begin{array}{l}          X^2+Y^2=1  \\\\           X=4Y,     \end{array}     \right. \end{equation*}

in cui X rappresenta \cos\theta e Y \sin\theta. Dunque si ha

\begin{equation*}     \left\{\begin{array}{l}          X^2+Y^2=1  \\\\           X=4Y,     \end{array}     \right.\Leftrightarrow     \left\{\begin{array}{l}          16Y^2+Y^2=1  \\\\           X=4Y,     \end{array}     \right.\Leftrightarrow     \left\{\begin{array}{l}          17Y^2=1  \\\\           X=4Y,     \end{array}     \right.\Leftrightarrow     \left\{\begin{array}{l}          Y=\pm\dfrac{1}{\sqrt{17}}  \\\\           X=4Y=\pm\dfrac{4}{\sqrt{17}}.     \end{array}     \right. \end{equation*}

\[\quad\]

\[\quad\]

\[\quad\]

massimi e minimi vincolati

Figura 5: soluzioni dell’equazione lineare goniometrica.

\[\quad\]

\[\quad\]

Abbiamo dunque trovato che la prima equazione di (1) è verificata per \left(\cos\theta,\sin\theta\right)=\pm\left(\dfrac{4}{\sqrt{17}},\dfrac{1}{\sqrt{17}}\right).

Inseriamo questi valori nella seconda equazione di (1) per trovare \rho:

\begin{equation*}     4\rho^3\pm4\dfrac{4}{\sqrt{17}}\pm\dfrac{1}{\sqrt{17}}=0\Leftrightarrow4\rho^3\pm\dfrac{17}{\sqrt{17}}=0\Leftrightarrow\rho^3=\mp\dfrac{\sqrt{17}}{4}\Leftrightarrow\rho=\mp\sqrt[3]{\dfrac{\sqrt{17}}{4}} \end{equation*}

Possiamo accettare solo il valore \rho=+\sqrt[3]{\dfrac{\sqrt{17}}{4}}\approx 1\in(0,2).

Abbiamo trovato dunque il punto critico di f\circ\varphi su R^\circ

\[\Tilde{P}_1=\left(\arccos{\left(-\dfrac{4}{\sqrt{17}}\right)},\sqrt[3]{\dfrac{\sqrt{17}}{4}}\right),\]

a cui corrisponde il punto P_1 su B

\begin{equation*}     P_1=\varphi\left(\Tilde{P}_1\right)=\left(-\sqrt[3]{\dfrac{\sqrt{17}}{4}}\dfrac{4}{\sqrt{17}},-\sqrt[3]{\dfrac{\sqrt{17}}{4}}\dfrac{1}{\sqrt{17}},\sqrt[3]{\dfrac{17}{16}}\right)=\left(-\sqrt[3]{\dfrac{16}{17}},-\dfrac{1}{4}\sqrt[3]{\dfrac{16}{17}},\sqrt[3]{\dfrac{17}{16}}\right). \end{equation*}

\[\quad\]

\[\quad\]

\[\quad\]

massimi e minimi vincolati

Figura 6: il punto critico \tilde{P}_1 della funzione parametrizzata f\circ\varphi a cui corrisponde il punto P_1 su B\subset\partial A.

\[\quad\]

\[\quad\]

Per quanto riguarda la frontiera \partial R, essa è composta dai 4 lati L_1,L_2,L_3 e L_4, come mostrato in figura. Notiamo, tuttavia, che non è necessario studiare f\circ\varphi su L_2 e L_4, i lati corrispondenti ai valori di \theta=0, \theta=2\pi, poichè la funzione f\circ\varphi è 2\pi-periodica rispetto a \theta. Infatti, avremmo potuto parametrizzare B tramite \tilde{\varphi}:[-\pi,\pi]\times[0,2], con \tilde{\varphi}(\theta,\rho)=\varphi(\theta,\rho), laddove entrambe definite. In tal modo la parte di frontiera relativa a \theta=0 non avrebbe svolto alcun ruolo nella ricerca dei massimi e minimi di f\circ\varphi su R. Lo stesso discorso vale analogamente per \theta=2\pi.

\[\quad\]

\[\quad\]

\[\quad\]

massimi e minimi vincolati

\[\quad\]

\[\quad\]

Il lato L_1, corrispondente al settore \rho=0 viene parametrizzato da

\begin{equation*}     \begin{array}{rcl}         \alpha_1:[0,2\pi] & \rightarrow&\mathbb{R}^2 \\          t&\mapsto&(t,0)     \end{array} \end{equation*}

e

\begin{equation*}     (f\circ\varphi)(\alpha_1(t))=f(\varphi(t,0))=f(0,0,0)=0. \end{equation*}

Ciò significa che f\circ\varphi è costante su L_1=[0,2\pi]\times\{0\} che risulta quindi un insieme di punti critici per f\circ\varphi, a cui corrisponde su B il solo punto \{(0,0,0)\}=\varphi(L_1), che denotiamo con P_2.

Per quanto riguarda L_3=[0,2\pi]\times\{2\}, parametrizzato da

\begin{equation*}     \begin{array}{rcl}         \alpha_3:[0,2\pi] & \rightarrow&\mathbb{R}^2 \\          t&\mapsto&(t,2).     \end{array} \end{equation*}

calcoliamo (f\circ\varphi)(\alpha_3(t)) e i suoi punti critici.

\begin{equation*}     (f\circ\varphi)(\alpha_3(t))=f(2\cos t,2\sin t,4)=8\cos t+2\sin t+16, \end{equation*}

\begin{equation*}     ((f\circ\varphi)\circ\alpha_3)'(t)=-8\sin t+2\cos t. \end{equation*}

Dunque i punti critici di ((f\circ\varphi)\circ\alpha_3), ovvero i punti che annullano la sua derivata prima, sono i valori di t\in[0,2\pi] per cui vale

\begin{equation*}     \cos t=4\sin t, \end{equation*}

equazione risolta in precedenza, dove abbiamo ottenuto i valori

\begin{equation*}     t_k=\arccos{\left(\dfrac{4}{\sqrt{17}}\right)}+k\pi,\qquad k\in\mathbb{Z} \end{equation*}

Di tale successioni, solo t_0,t_1\in[0,2\pi], dominio della parametrizzazione. I punti su R corrispondenti a t_0 e t_1 sono rispettivamente \Tilde{P}_3 e \Tilde{P}_4 definiti da

\begin{equation*}     \Tilde{P}_3=\alpha_3\left(\arccos{\left(\dfrac{4}{\sqrt{17}}\right)}\right)=\left(\arccos{\left(\dfrac{4}{\sqrt{17}}\right)},2\right) \end{equation*}

e

\begin{equation*}     \Tilde{P}_4=\alpha_3\left(\arccos{\left(\dfrac{4}{\sqrt{17}}\right)}+\pi\right)=\left(\arccos{\left(\dfrac{4}{\sqrt{17}}\right)}+\pi,2\right), \end{equation*}

a cui corrispondono a loro volta i punti su B

\begin{equation*}     P_3=\varphi\left(\arccos{\left(\dfrac{4}{\sqrt{17}}\right)},2\right)=\left(\dfrac{8}{\sqrt{17}},\dfrac{2}{\sqrt{17}},4\right) \end{equation*}

e

\begin{equation*}     P_4=\varphi\left(\arccos{\left(\dfrac{4}{\sqrt{17}}\right)}+\pi,2\right)=\left(-\dfrac{8}{\sqrt{17}},-\dfrac{2}{\sqrt{17}},4\right). \end{equation*}

massimi e minimi vincolati

Figura 9: rappresentazione dei punti interessanti trovati su \partial R e la loro immagine su \partial A.

\[\quad\]

\[\quad\]

Questo conclude lo studio di f su B che ha portato alla nostra attenzione 4 candidati ad essere punti di massimo o di minimo di f su A: i punti P_1,P_2,P_3 e P_4.

C) Osserviamo immediatamente che C è un cerchio di raggio 2 a quota z=4. Pertanto esso è parametrizzabile da

\begin{equation*} \begin{array}{rcl}     \psi:R=[0,2\pi]\times[0,2] & \rightarrow&\mathbb{R}^3 \\     (\theta,\rho) & \mapsto&(\rho\cos\theta,\rho\sin\theta,4). \end{array}     \end{equation*}

\[\quad\]

\[\quad\]

\[\quad\]

massimi e minimi vincolati

Figura 10: parametrizzazione di C. Vengono rappresentate alcune linee coordinate per aiutare la visualizzazione della mappa \psi.

\[\quad\]

\[\quad\]

Calcoliamo f\circ\psi

\begin{equation*}     (f\circ\psi)(\theta,\rho)=f(\rho\cos\theta,\rho\sin\theta,4)=4\rho\cos\theta+\rho\sin\theta+16. \end{equation*}

Analogamente a quanto detto in precedenza, anche per f\circ\psi il teorema di Weierstrass garantisce l’esistenza del massimo e del minimo su R. Studiamo inizialmente il comportamento di f\circ\psi sull’insieme aperto R^\circ tramite la ricerca dei punti critici, per poi studiare f\circ\psi su \partial R.

Calcoliamo le derivate parziali di f\circ\psi:

\[\begin{cases} \dfrac{\partial (f\circ\psi)}{\partial \theta}(\theta,\rho)=-4\rho\sin\theta+\rho\cos\theta\\\\ \dfrac{\partial (f\circ\psi)}{\partial \rho}(\theta,\rho)=4\cos\theta+\sin\theta. \end{cases}\]

Dunque

(2) \begin{equation*}     \nabla (f\circ\psi)(\theta,\rho)=(0,0)\Leftrightarrow\displaystyle\left\{\begin{array}{l} -4\rho\sin\theta+\rho\cos\theta=0\\\\ 4\cos\theta+\sin\theta=0 \end{array} \right.\Leftrightarrow\displaystyle\left\{\begin{array}{l} \rho(\cos\theta-4\sin\theta)=0\\\\ 4\cos\theta+\sin\theta=0. \end{array} \right. \end{equation*}

Se (\theta,\rho)\in R^\circ, \rho>0 e il precedente sistema è equivalente a

\begin{equation*}     \left\{\begin{array}{l} \cos\theta-4\sin\theta=0\\\\ 4\cos\theta+\sin\theta=0. \end{array} \right.\Leftrightarrow\displaystyle\left\{\begin{array}{l} \cos\theta=4\sin\theta\\\\ \cos\theta=-\dfrac{1}{4}\sin\theta, \end{array} \right. \end{equation*}

che non ammette soluzioni: ciò significa che non ci sono punti critici di f\circ\psi su R^\circ.

Potevamo giungere a questa conclusione osservando che la restrizione di f a C è affine, dunque non può ammettere punti critici.

Per quanto riguarda \partial R, poichè anche f\circ\psi è 2\pi-periodica rispetto a \theta, lo stesso discorso fatto in precedenza ci autorizza ad ignorare i lati corrispondenti ai valori \theta=0 e \theta=2\pi. I lati rimasti sono M_1=[0,2\pi]\times\{0\} e M_3=[0,2\pi]\times\{2\}. Osserviamo tuttavia che \psi(M_1)=\{(0,0,4)\}, dunque anziché parametrizzare il lato M_1 e studiare f\circ\psi attraverso tale parametrizzazione, a mio avviso è più rapido annettere il punto P_5=(0,0,4) tra i possibili candidati e semplicemente valutare f(P_5).

Infine, per quanto riguarda M_3, osserviamo che f\circ\psi|_{M_3}=f\circ\varphi|_{L_3}, che abbiamo ampiamente studiato in precedenza.

\[\quad\]

\[\quad\]

\[\quad\]

massimi e minimi vincolati

Figura 11: immagine di \partial R attraverso \varphi. Come si evince, non vi è nulla di speciale in \psi(M_2)=\psi(M_4) rappresentati in nero.

\[\quad\]

\[\quad\]

Questo conclude lo studio di \partial A=B\cup C.

I possibili candidati ad essere punti di massimo o di minimo di f su A sono i seguenti

\begin{equation*} \begin{split}      &P_1=\left(-\sqrt[3]{\dfrac{16}{17}},-\dfrac{1}{4}\sqrt[3]{\dfrac{16}{17}},\sqrt[3]{\dfrac{17}{16}}\right),\qquad P_2=(0,0,0),\qquad P_3=\left(\dfrac{8}{\sqrt{17}},\dfrac{2}{\sqrt{17}},4\right),\\      & P_4=\left(-\dfrac{8}{\sqrt{17}},-\dfrac{2}{\sqrt{17}},4\right),\qquad P_5=(0,0,4), \end{split} \end{equation*}

\[\quad\]

\[\quad\]

\[\quad\]

massimi e minimi vincolati

Figura 12: tutti i possibili candidati ad essere punti di massimo o minimo di f su A.

\[\quad\]

\[\quad\]

i cui valori rispetto a f sono

\begin{equation*} \begin{split}     &f(P_1)=-\dfrac{3}{4}\left(\dfrac{17}{2}\right)^{\frac{2}{3}}\approx-3.1,\qquad f(P_2)=0,\qquad f(P_3)=16+2\sqrt{17}\approx 24.2,\\     &f(P_4)=16-2\sqrt{17}\approx 7.8,\qquad f(P_5)=16. \end{split} \end{equation*}

\[\quad\]

La soluzione dell’esercizio è la seguente: f ha massimo su A pari a 16+2\sqrt{17}, assunto nel punto P_3=\left(\dfrac{8}{\sqrt{17}},\dfrac{2}{\sqrt{17}},4\right) e valore minimo -\dfrac{3}{4}\left(\dfrac{17}{2}\right)^{\frac{2}{3}}, assunto in P_1=\left(-\sqrt[3]{\dfrac{16}{17}},-\dfrac{1}{4}\sqrt[3]{\dfrac{16}{17}},\sqrt[3]{\dfrac{17}{16}}\right).

Svolgimento 2.

Occupiamoci della frontiera di A, questa volta utilizzando il metodo dei moltiplicatori di Lagrange, anziché passare attraverso la parametrizzazione.

Possiamo separare la frontiera di A in questo modo:

\begin{equation*}     \partial A=D\sqcup E\sqcup F, \end{equation*}

dove

\begin{equation*}     D:=\{(x,y,z)\in\mathbb{R}^3\mid x^2+y^2=z, z<4\} \end{equation*}

\begin{equation*}     E:=\{(x,y,z)\in\mathbb{R}^3\mid x^2+y^2<z, z=4\} \end{equation*}

e

\begin{equation*}     F:=\{(x,y,z)\in\mathbb{R}^3\mid x^2+y^2=z=4\}=\overline{D}\cap\overline{E}. \end{equation*}

\[\quad\]

\[\quad\]

\[\quad\]

massimi e minimi vincolati

Figura 13: gli insiemi D, E ed F, che compongono \partial A.

\[\quad\]

massimi e minimi vincolati

Figura 14: si nota che F=\overline{D}\cap\overline{E}.

\[\quad\]

\[\quad\]

Sarà quindi necessario costruire tre Lagrangiane, \mathcal{L}_D, \mathcal{L}_E e \mathcal{L}_F, per ciascun vincolo.

D) Consideriamo la funzione

\begin{equation*}         \begin{array}{rcl}              g:\mathbb{R}^3& \rightarrow&\mathbb{R} \\             (x,y,z) & \mapsto& x^2+y^2-z         \end{array}     \end{equation*}

e restringiamola al cilindro aperto \mathcal{C}:=\{(x,y,z)\in\mathbb{R}^3\mid x^2+y^2< 4\}. Notiamo che B=\{(x,y,z)\in\mathcal{C}\mid g(x,y,z)=0\}. Abbiamo cioè espresso il vincolo B come luogo di zeri della funzione g. A questo punto possiamo costruire la Lagrangiana \mathcal{L}_D

\begin{equation*}         \begin{split}             \mathcal{L}_D(x,y,z,\lambda):&=f(x,y,z)-\lambda g(x,y,z)\\             &=4x+y+z^2-\lambda(x^2+y^2-z).         \end{split}     \end{equation*}

Troviamo i punti critici della Lagrangiana, secondo il teorema dei moltiplicatori di Lagrange:

\begin{equation*} \begin{split}         \nabla\mathcal{L}_D(x,y,z,\lambda)=(0,0,0,0) \\ & \iff \left\{\begin{array}{l}               \dfrac{\partial\mathcal{L}_D}{\partial x}=4-2\lambda x=0 \\\\               \dfrac{\partial\mathcal{L}_D}{\partial y}=1-2\lambda y=0 \\\\               \dfrac{\partial\mathcal{L}_D}{\partial z}=2z+\lambda=0 \\\\               \dfrac{\partial\mathcal{L}_D}{\partial \lambda}=-(x^2+y^2-z)=0         \end{array}\right. \\ &\iff \left\{\begin{array}{l}               4+4zx=0 \\\\               1+4z y=0 \\\\               \lambda=-2z \\\\               x^2+y^2=z         \end{array}\right. \\ & \iff \left\{\begin{array}{l}               z=-\dfrac{1}{x} \\\\               z=-\dfrac{1}{4y} \\\\               \lambda=-2z \\\\               x^2+y^2=z.         \end{array}\right. \end{split}     \end{equation*}

Le prime due equazioni impongono la relazione y=\dfrac{x}{4} e, considerato che z=-\dfrac{1}{x}, l’ultima equazione diventa

\begin{equation*}         x^2+\dfrac{x^2}{16}=-\dfrac{1}{x}\Leftrightarrow \dfrac{17}{16}x^3=-1\Leftrightarrow x=-\sqrt[3]{\dfrac{16}{17}}.     \end{equation*}

Recuperiamo i valori delle altre coordinate:

\begin{equation*}         y=\dfrac{-\sqrt[3]{\dfrac{16}{17}}}{4}=-\dfrac{1}{4}\sqrt[3]{\dfrac{16}{17}},\qquad z=-\dfrac{1}{-\sqrt[3]{\dfrac{16}{17}}}=\sqrt[3]{\dfrac{17}{16}}.     \end{equation*}

Abbiamo quindi trovato il punto critico P_1=\left(-\sqrt[3]{\dfrac{16}{17}},-\dfrac{1}{4}\sqrt[3]{\dfrac{16}{17}},\sqrt[3]{\dfrac{17}{16}}\right)\in D. Osserviamo anche che

\[\nabla g(P_1)=\left(-2\sqrt[3]{\dfrac{16}{17}},-2\dfrac{1}{4}\sqrt[3]{\dfrac{16}{17}},-1\right)\neq(0,0,0).\]

\[\quad\]

\[\quad\]

\[\quad\]

massimi e minimi vincolati

Figura 15: l’unico punto critico su D.

\[\quad\]

\[\quad\]

E) Ricordando che E:=\{(x,y,z)\in\mathbb{R}^3\mid x^2+y^2<z=4\}, possiamo esprimere E come luogo di zero della funzione h:\mathcal{C}\rightarrow\mathbb{R}, definita semplicemente h(x,y,z):=z-4. Ripetendo i passi del punto precedente, calcoliamo la Lagrangiana \mathcal{L}_E e i suoi punti critici.

\begin{equation*}         \begin{split}             \mathcal{L}_E(x,y,z,\mu):&=f(x,y,z)-\mu h(x,y,z)\\             &=4x+y+z^2-\mu(z-4)         \end{split}     \end{equation*}

\begin{equation*}         \nabla\mathcal{L}_E=(0,0,0,0)\Leftrightarrow\left\{\begin{array}{l}         \dfrac{\partial\mathcal{L}_E}{\partial x}=4=0 \\\\               \dfrac{\partial\mathcal{L}_E}{\partial y}=1=0 \\\\               \dfrac{\partial\mathcal{L}_E}{\partial z}=2z-\mu=0 \\\\               \dfrac{\partial\mathcal{L}_E}{\partial \mu}=-(z-4)=0.         \end{array}\right.     \end{equation*}

Il sistema è evidentemente impossibile.

Rimane da studiare il vincolo unidimensionale F, ottenuto come intersezione tra (la chiusura de) i due vincoli D ed E.

Applichiamo 2 sfruttando le funzioni che definivano i precedenti vincoli: g e h. Vale infatti

\begin{equation*}     \begin{split}         F&=\{(x,y,z)\in\mathcal{C}\mid x^2+y^2=z, \ z=4\}\\         &=\{(x,y,z)\in\mathcal{C}\mid g(x,y,z)=0, \ h(x,y,z)=0\}.     \end{split}     \end{equation*}

Seguendo l’enunciato, costruiamo la Lagrangiana \mathcal{L}_F:\mathcal{C}\times\mathbb{R}^2\subset\mathbb{R}^5\rightarrow\mathbb{R}

\begin{equation*}         \begin{split}             \mathcal{L}_F(x,y,z,\lambda,\mu):&=f(x,y,z)-\lambda g(x,y,z)-\mu h(x,y,z)\\             &=4x+y+z^2-\lambda(x^2+y^2-z)-\mu(z-4)         \end{split}     \end{equation*}

ed, infine, i suoi punti critici:

\begin{equation*}         \nabla\mathcal{L}(x,y,z,\lambda,\mu)=(0,0,0,0,0)\Leftrightarrow\left\{\begin{array}{l}         \dfrac{\partial\mathcal{L}_F}{\partial x}=4-2\lambda x=0 \\\\               \dfrac{\partial\mathcal{L}_F}{\partial y}=1-2\lambda y=0 \\\\               \dfrac{\partial\mathcal{L}_F}{\partial z}=2z+\lambda+\mu=0 \\\\               \dfrac{\partial\mathcal{L}_F}{\partial \lambda}=-(x^2+y^2-z)=0 \\\\               \dfrac{\partial\mathcal{L}_F}{\partial \mu}=-(z-4)=0.         \end{array}\right.\Leftrightarrow\left\{\begin{array}{l}               \lambda=\dfrac{2}{x} \\\\               \lambda=\dfrac{1}{2y} \\\\               \lambda+\mu=8 \\\\               x^2+y^2=4 \\\\               z=4.         \end{array}\right.     \end{equation*}

Confrontando le prime due equazioni troviamo la relazione x=4y. Utilizzandola nella quarta equazione si ricava

\begin{equation*}         16y^2+y^2=4\Leftrightarrow y=\pm\dfrac{2}{\sqrt{17}}.     \end{equation*}

Ricaviamo anche le altre coordinate:

\begin{equation*}         x=\pm\dfrac{8}{\sqrt{17}},\qquad z=4.     \end{equation*}

Abbiamo quindi trovato altri due punti critici: P_2=\left(\dfrac{8}{\sqrt{17}},\dfrac{2}{\sqrt{17}},4\right) e P_3=\left(-\dfrac{8}{\sqrt{17}},-\dfrac{2}{\sqrt{17}},4\right). Infine, per accettare i punti trovati, dobbiamo verificare che i gradienti delle funzioni definenti i vincoli non siano linearmente dipendenti in quei punti. Vale

\begin{equation*}     \nabla g(P_2)=\left(\dfrac{16}{\sqrt{17}},\dfrac{4}{\sqrt{17}},-1\right),\qquad  \nabla h(P_2)=(0,0,1),     \end{equation*}

\begin{equation*}    \nabla g(P_3)=\left(-\dfrac{16}{\sqrt{17}},-\dfrac{4}{\sqrt{17}},-1\right),\qquad\nabla h(P_3)=(0,0,1).     \end{equation*}

I gradienti delle funzioni non sono linearmente dipendenti nè in P_2 nè in P_3, che quindi possono essere accettati.

\[\quad\]

\[\quad\]

\[\quad\]

massimi e minimi vincolati

Figura 16: i due punti critici su F.

\[\quad\]

\[\quad\]

Abbiamo trovato 3 candidati ad essere massimo e minimo di f su A:

\begin{equation*}     P_1=\left(-\sqrt[3]{\dfrac{16}{17}},-\dfrac{1}{4}\sqrt[3]{\dfrac{16}{17}},\sqrt[3]{\dfrac{17}{16}}\right),\qquad P_2=\left(\dfrac{8}{\sqrt{17}},\dfrac{2}{\sqrt{17}},4\right),\qquad P_3=\left(-\dfrac{8}{\sqrt{17}},-\dfrac{2}{\sqrt{17}},4\right). \end{equation*}

Calcoliamo il valore di f su ciascuno di essi e confrontiamoli.

\begin{equation*} \begin{gathered}     f(P_1)=f\left(-\sqrt[3]{\dfrac{16}{17}},-\dfrac{1}{4}\sqrt[3]{\dfrac{16}{17}},\sqrt[3]{\dfrac{17}{16}}\right)=-\dfrac{3}{4}\left(\dfrac{17}{2}\right)^{\dfrac{2}{3}}, \\  f(P_2)=f\left(\dfrac{8}{\sqrt{17}},\dfrac{2}{\sqrt{17}},4\right)=16+2\sqrt{17},\\     f(P_3)=f\left(-\dfrac{8}{\sqrt{17}},-\dfrac{2}{\sqrt{17}},4\right)=16-2\sqrt{17}. \end{gathered} \end{equation*}

\[\quad\]

La soluzione dell’esercizio è la seguente: f ha massimo su A pari a 16+2\sqrt{17}, assunto nel punto P_2=\left(\dfrac{8}{\sqrt{17}},\dfrac{2}{\sqrt{17}},4\right) e valore minimo -\dfrac{3}{4}\left(\dfrac{17}{2}\right)^{\dfrac{2}{3}}, assunto in P_1=\left(-\sqrt[3]{\dfrac{16}{17}},-\dfrac{1}{4}\sqrt[3]{\dfrac{16}{17}},\sqrt[3]{\dfrac{17}{16}}\right).

 
 

Esercizio 2  (\bigstar\bigstar\bigstar\largewhitestar\largewhitestar). Data la funzione f:\mathbb{R}^3\rightarrow\mathbb{R}, definita da

\[f(x,y,z):=xy^2z,\]

determinare, se esistono, il massimo e il minimo di f sull’insieme

\[A:=\left\{(x,y,z)\in\mathbb{R}^3\mid x+z=0, \ \dfrac{x^2}{4}+\dfrac{y^2}{9}+z^2=1\right\}.\]

Svolgimento 1.

massimi e minimi vincolati

\[\quad\]

massimi e minimi vincolati

Figura 19: l’insieme A, intersezione delle superfici precedenti.

\[\quad\]

\[\quad\]

Osserviamo che f è un monomio, pertanto è una funzione continua su \mathbb{R}^3, mentre l’insieme A è chiuso e limitato. La chiusura segue dal fatto che A è intersezione di controimmagini di insiemi chiusi di \mathbb{R} tramite funzioni continue, mentre per la limitatezza notiamo che esso è intersezione tra un piano e un’ellissoide, quest’ultimo fornisce la limitatezza ad A. Dunque è un chiuso e limitato di \mathbb{R}^3, compatto per il teorema di Heine-Borel. Possiamo allora applicare il teorema di Weierstrass, il quale garantisce l’esistenza del massimo e del minimo di f su A.

Osserviamo anche che A^\circ=\emptyset, ovvero A=\partial A: l’intero problema, si riduce quindi a studiare il massimo ed il minimo di f attraverso i metodi che abbiamo solitamente utilizzato per studiare la frontiera di A.

Come detto, A è intersezione tra il piano z=-x e l’ellissoide \dfrac{x^2}{4}+\dfrac{y^2}{9}+z^2=1. Cerchiamo una parametrizzazione di questa curva, mettendo a sistema le due equazioni.

\begin{equation*}     \left\{\begin{array}{l}         z=-x   \\\\         \dfrac{x^2}{4}+\dfrac{y^2}{9}+z^2=1       \end{array}\right.\Leftrightarrow\left\{\begin{array}{l}         z=-x   \\\\         \dfrac{x^2}{4}+\dfrac{y^2}{9}+x^2=1       \end{array}\right.\Leftrightarrow\left\{\begin{array}{l}         z=-x   \\\\         \dfrac{x^2}{\left(\dfrac{2}{\sqrt{5}}\right)^2}+\dfrac{y^2}{3^2}=1       \end{array}\right. \end{equation*}

Possiamo riconoscere che la seconda equazione descrive un’ellisse di semiassi \dfrac{2}{\sqrt{5}} e 3, la cui parametrizzazione è

\begin{equation*}     \begin{array}{rcl}          \alpha:[0,2\pi]& \rightarrow&\mathbb{R}^2 \\         t & \mapsto&\left(\dfrac{2}{\sqrt{5}}\cos t,3\sin t\right).     \end{array} \end{equation*}

\[\quad\]

\[\quad\]

\[\quad\]

massimi e minimi vincolati

Figura 20: tratteggiata in blu la proiezione di A sul piano z=0.

\[\quad\]

\[\quad\]

Essa è la proiezione su \mathbb{R}^2 di A, che invece vive in \mathbb{R}^3, più precisamente nel piano z=-x. Ecco dunque che una parametrizzazione di A è la seguente:

\begin{equation*}     \begin{array}{rcl}         \beta:[0,2\pi] & \rightarrow&\mathbb{R}^3 \\         t & \mapsto&\left(\dfrac{2}{\sqrt{5}}\cos t,3\sin t,-\dfrac{2}{\sqrt{5}}\cos t\right)     \end{array} \end{equation*}

A questo punto, esplicitiamo f\circ\beta e studiamo il suo massimo e minimo su [0,2\pi]. Essendo f\circ\beta composizione di funzioni continue e l’intervallo [0,2\pi] compatto, il teorema di Weierstrass garantisce nuovamente il massimo ed il minimo di f\circ\beta su [0,2\pi].

Calcoliamo dunque f\circ\beta:

\begin{equation*}     (f\circ\beta)(t)=f\left(\dfrac{2}{\sqrt{5}}\cos t,3\sin t,-\dfrac{2}{\sqrt{5}}\cos t\right)=\dfrac{2}{\sqrt{5}}\cos t(3\sin t)^2\left(-\dfrac{2}{\sqrt{5}}\cos t\right)=-\dfrac{36}{5}\cos^2t\sin^2t. \end{equation*}

Seguendo le indicazioni del teorema di Fermat, i possibili punti di massimo e di minimo vanno ricercati tra i punti critici di f\circ\beta nella parte interna del dominio della parametrizzazione, tra gli eventuali punti di non derivabilità e tra gli estremi \{0\} e \{2\pi\}. Essendo f\circ\beta derivabile, non vi sono punti di non derivabilità. Cerchiamo allora i punti critici, ovvero i valori t\in(0,2\pi) tali per cui accade (f\circ\beta)'(t)=0.

\begin{equation*} \begin{split}     &(f\circ\beta)'(t)=\dfrac{72}{5}\cos t\sin t(\sin^2 t-\cos^2 t)=0\Leftrightarrow\\      &\cos t=0\lor\sin t=0\lor \cos t=\sin t\lor\cos t=-\sin t. \end{split} \end{equation*}

I valori di t per cui ciò accade sono t_k=\dfrac{k}{4}\pi, con k=1,...,8, ovvero

\begin{equation*} \begin{gathered}     t_1=0,\qquad t_2=\dfrac{\pi}{4},\qquad t_3=\dfrac{\pi}{2},\qquad t_4=\dfrac{3}{4}\pi, \\ t_5=\pi,\qquad t_6=\dfrac{5}{4}\pi, \qquad t_7=\dfrac{3}{2}\pi,\qquad t_8=\dfrac{7}{4}\pi. \end{gathered} \end{equation*}

\[\quad\]

\[\quad\]

\[\quad\]

massimi e minimi vincolati

Figura 21: istanti corrispondenti a punti critici della mappa f\circ\beta, la rilettura di f su A.

\[\quad\]

\[\quad\]

Nonostante t_1\notin(0,2\pi), avremo comunque dovuto considerarlo tra gli estremi del dominio, assieme a t_9=2\pi. A questi istanti t_1, corrispondono i punti P_i=\beta(t_i) su A:

\begin{equation*}     \begin{split}         &P_1=\beta(0)=\left(\dfrac{2}{\sqrt{5}},0,-\dfrac{2}{\sqrt{5}}\right),\qquad P_2=\beta\left(\dfrac{\pi}{4}\right)=\left(\sqrt{\dfrac{2}{5}},\dfrac{3\sqrt{2}}{2},-\sqrt{\dfrac{2}{5}}\right),\qquad P_3=\beta\left(\dfrac{\pi}{2}\right)=(0,3,0),\\         &P_4=\beta\left(\dfrac{3}{4}\pi\right)=\left(-\sqrt{\dfrac{2}{5}},\dfrac{3\sqrt{2}}{2},\sqrt{\dfrac{2}{5}}\right),\qquad P_5=\beta(\pi)=\left(-\dfrac{2}{\sqrt{5}},0,\dfrac{2}{\sqrt{5}}\right),\\ & P_6=\beta\left(\dfrac{5}{4}\pi\right)=\left(-\sqrt{\dfrac{2}{5}},-\dfrac{3\sqrt{2}}{2},\sqrt{\dfrac{2}{5}}\right),\\         &P_7=\beta\left(\dfrac{3}{2}\pi\right)=(0,-3,0),\qquad P_8=\beta\left(\dfrac{7}{8}\pi\right)=\left(\sqrt{\dfrac{2}{5}},-\dfrac{3\sqrt{2}}{2},-\sqrt{\dfrac{2}{5}}\right),\qquad P_9=\beta(2\pi)=P_1.     \end{split} \end{equation*}

\[\quad\]

\[\quad\]

\[\quad\]

massimi e minimi vincolati

Figura 22: rappresentazione di tutti i candidati ad essere punti di massimo o minimo di f su A.

\[\quad\]

\[\quad\]

Abbiamo trovato 8 candidati ad essere massimo e minimo di f su A. Calcoliamo il loro valore attraverso f e confrontiamoli.

\begin{equation*}     \begin{split}         &f(P_1)=f\left(\dfrac{2}{\sqrt{5}},0,-\dfrac{2}{\sqrt{5}}\right)=0,\qquad f(P_2)=f\left(\sqrt{\dfrac{2}{5}},\dfrac{3\sqrt{2}}{2},-\sqrt{\dfrac{2}{5}}\right)=-\dfrac{9}{5},\qquad f(P_3)=f(0,3,0)=0\\         &f(P_4)=f\left(-\sqrt{\dfrac{2}{5}},\dfrac{3\sqrt{2}}{2},\sqrt{\dfrac{2}{5}}\right)=-\dfrac{9}{5},\qquad f(P_5)=f\left(-\dfrac{2}{\sqrt{5}},0,\dfrac{2}{\sqrt{5}}\right)=0,\qquad f(P_6)=f\left(-\sqrt{\dfrac{2}{5}},-\dfrac{3\sqrt{2}}{2},\sqrt{\dfrac{2}{5}}\right)=-\dfrac{9}{5}\\         &f(P_7)=f(0,-3,0)=0,\qquad f(P_8)=f\left(\sqrt{\dfrac{2}{5}},-\dfrac{3\sqrt{2}}{2},-\sqrt{\dfrac{2}{5}}\right)=-\dfrac{9}{5}.     \end{split} \end{equation*}

\[\quad\]

La soluzione dell’esercizio è la seguente: f ha massimo su A pari a 0, assunto nei punti {P_1=\left(\dfrac{2}{\sqrt{5}},0,-\dfrac{2}{\sqrt{5}}\right)}, P_3=(0,3,0), P_5=\left(-\dfrac{2}{\sqrt{5}},0,\dfrac{2}{\sqrt{5}}\right) e P_7=(0,-3,0) e valore minimo -\dfrac{9}{5}, assunto in P_2=\left(\sqrt{\dfrac{2}{5}},\dfrac{3\sqrt{2}}{2},-\sqrt{\dfrac{2}{5}}\right), P_4=\left(-\sqrt{\dfrac{2}{5}},\dfrac{3\sqrt{2}}{2},\sqrt{\dfrac{2}{5}}\right), P_6=\left(-\sqrt{\dfrac{2}{5}},-\dfrac{3\sqrt{2}}{2},\sqrt{\dfrac{2}{5}}\right) e P_8=\left(\sqrt{\dfrac{2}{5}},-\dfrac{3\sqrt{2}}{2},-\sqrt{\dfrac{2}{5}}\right).

Svolgimento 2.

Risolviamo l’esercizio attraverso il teorema dei moltiplicatori di Lagrange riferendoci ad A come vincolo, il quale può essere espresso come intersezione di due vincoli: A=\mathcal{P}\cap\mathcal{E}, dove \mathcal{P} è il piano

\begin{equation*}     \mathcal{P}:=\{(x,y,z)\in\mathbb{R}^3\mid x+z=0\}, \end{equation*}

mentre \mathcal{E} è l’ellissoide

\begin{equation*}     \mathcal{E}:=\left\{(x,y,z)\in\mathbb{R}^3\mid \dfrac{x^2}{4}+\dfrac{y^2}{9}+z^2=1\right\}. \end{equation*}

Osserviamo che \mathcal{P}=\{(x,y,z)\in\mathbb{R}^3\mid g(x,y,z)=0\} e \mathcal{E}=\{(x,y,z)\in\mathbb{R}^3\mid h(x,y,z)=0\}, dove g(x,y,z):=x+z e h(x,y,z)=\dfrac{x^2}{4}+\dfrac{y^2}{9}+z^2-1.

Il teorema dei moltiplicatori di Lagrange, declinato in questo contesto, afferma che, in corrispondenza dei punti di massimo e di minimo relativi di f su A=\mathcal{P}\cap\mathcal{E} succede un fenomeno che possiamo rileggere sia in chiave geometrica che algebrica: geometricamente lo si può interpretare dal fatto che i gradienti di f, g ed h sono complanari. Da un punto di vista algebrico, detto (x,y,z) un punto di estremo relativo, esisteranno \lambda,\mu\in\mathbb{R} tali che

\begin{equation*}     \nabla f(x,y,z)=\lambda\nabla g(x,y,z)+\mu\nabla h(x,y,z). \end{equation*}

Per questo motivo, costruiamo la Lagrangiana \mathcal{L}:\mathbb{R}^5\rightarrow\mathbb{R}, definita tramite

\begin{equation*}     \mathcal{L}(x,y,z,\lambda,\mu):=f(x,y,z)-\lambda g(x,y,z)-\mu h(x,y,z). \end{equation*}

Il teorema dei moltiplicatori di Lagrange garantisce che i punti critici di \mathcal{L} corrisponderanno esattamente agli estremi relativi di f su A. Calcoliamo la Lagrangiana:

\begin{equation*} \begin{split}     \mathcal{L}(x,y,z,\lambda,\mu):=xy^2z-\lambda(x+z)-\mu\left(\dfrac{x^2}{4}+\dfrac{y^2}{9}+z^2-1\right). \end{split} \end{equation*}

Andiamo a cercare i suoi punti critici, risolvendo con attenzione il seguente sistema

\begin{equation*}     \left\{\begin{array}{l}         \dfrac{\partial\mathcal{L}}{\partial x}=y^2z-\lambda-\dfrac{1}{2}\mu x=0  \\\\         \dfrac{\partial\mathcal{L}}{\partial y}=2xyz-\dfrac{2}{9}\mu y=0  \\\\         \dfrac{\partial\mathcal{L}}{\partial z}=xy^2-\lambda-2\mu z=0  \\\\         \dfrac{\partial\mathcal{L}}{\partial \lambda}=-(x+z)=0 \\\\         \dfrac{\partial\mathcal{L}}{\partial \mu}=-\left(\dfrac{x^2}{4}+\dfrac{y^2}{9}+z^2-1\right)=0. \\\\     \end{array}\right. \end{equation*}

Dalla quarta equazione troviamo la relazione z=-x da utilizzare in ciascuna delle altre equazioni:

\begin{equation*}     \left\{\begin{array}{l}         -xy^2-\lambda-\dfrac{1}{2}\mu x=0  \\\\         -2x^2y-\dfrac{2}{9}\mu y=0  \\\\         xy^2-\lambda+2\mu x=0  \\\\         z=-x \\\\         \dfrac{x^2}{4}+\dfrac{y^2}{9}+x^2=1  \\\\     \end{array}\right.\Leftrightarrow\left\{\begin{array}{l}         -xy^2-\lambda-\dfrac{1}{2}\mu x=0  \\\\         y\left(x^2+\dfrac{1}{9}\mu\right)=0  \\\\         xy^2-\lambda+2\mu x=0  \\\\         z=-x \\\\         \dfrac{5}{4}x^2+\dfrac{y^2}{9}=1.  \\\\     \end{array}\right. \end{equation*}

La seconda equazione si annulla se y=0 oppure se \mu=-9x^2. Supponiamo inizialmente y=0, l’ultima equazione diventa

\begin{equation*}     \dfrac{5}{4}x^2=1\Leftrightarrow x=\pm\dfrac{2}{\sqrt{5}} \end{equation*}

e, ricordando che z=-x, troviamo z=\mp\dfrac{2}{\sqrt{5}}. Abbiamo, al momento, trovato due punti critici:

\begin{equation*}     P_1=\left(\dfrac{2}{\sqrt{5}},0,-\dfrac{2}{\sqrt{5}}\right)\qquad e \qquad P_2=\left(-\dfrac{2}{\sqrt{5}},0,\dfrac{2}{\sqrt{5}}\right). \end{equation*}

A questo punto, possiamo supporre y\neq0 e dunque assumere \mu=-9x^2. Inseriamo questa informazione nel sistema, dopo aver sostituito la prima equazione con la somma tra di essa e la terza equazione:

\begin{equation*}     \left\{\begin{array}{l}         -2\lambda+\dfrac{3}{2}\mu x=0  \\\\         y\left(x^2+\dfrac{1}{9}\mu\right)=0  \\\\         xy^2-\lambda+2\mu x=0  \\\\         z=-x \\\\         \dfrac{5}{4}x^2+\dfrac{y^2}{9}=1  \\\\     \end{array}\right.\Leftrightarrow\left\{\begin{array}{l}         -2\lambda-\dfrac{27}{2} x^3=0  \\\\         \mu=-9x^2  \\\\         xy^2-\lambda-18x^3=0  \\\\         z=-x \\\\         \dfrac{5}{4}x^2+\dfrac{y^2}{9}=1.  \\\\     \end{array}\right. \end{equation*}

Dalla prima equazione risulta

\begin{equation*}     \lambda=-\dfrac{27}{4}x^3, \end{equation*}

che, inserito nella terza equazione, dà

\begin{equation*}     xy^2+\dfrac{27}{4}x^3-18x^3=0\Leftrightarrow x\left(y^2-\dfrac{45}{4}x^2\right)=0. \end{equation*}

Quest’ultima equazione è soddisfatta se x=0, oppure se y^2=\dfrac{45}{4}x^2, cioè y=\pm\dfrac{3\sqrt{5}}{2}x. Ricaviamo velocemente le soluzioni derivanti da x=0: l’ultima equazione risulta

\begin{equation*} \dfrac{y^2}{9}=1\Leftrightarrow y=\pm3.     \end{equation*}

Poiche z=-x=-0, abbiamo trovato altri due punti critici:

\begin{equation*}     P_3=(0,3,0)\qquad e \qquad P_4=(0,-3,0). \end{equation*}

Infine, indipendentemente dal segno che sussiste nella relazione tra x ed y, inseriamo y^2=\dfrac{45}{4}x^2 nell’ultima equazione per ricavare le ultime soluzioni del sistema:

\begin{equation*}     \dfrac{5}{4}x^2+\dfrac{\dfrac{45}{4}x^2}{9}=1\Leftrightarrow\dfrac{5}{4}x^2+\dfrac{5}{4}x^2=1\Leftrightarrow\dfrac{5}{2}x^2=1\Leftrightarrow x=\pm\sqrt{\dfrac{2}{5}}. \end{equation*}

Ricaviamo le altre coordinate attraverso le precedenti relazioni, facendo attenzione a tutte le possibili combinazioni di segni: poichè z=-x

\begin{equation*}     x=\pm\sqrt{\dfrac{2}{5}}\Rightarrow z=\mp\sqrt{\dfrac{2}{5}}; \end{equation*}

inoltre, da y=\pm\dfrac{3\sqrt{5}}{2}x,

\begin{equation*}     x=\sqrt{\dfrac{2}{5}}\Rightarrow y=\pm\dfrac{3\sqrt{5}}{2}\sqrt{\dfrac{2}{5}}=\pm\dfrac{3\sqrt{2}}{2}, \end{equation*}

\begin{equation*}     x=-\sqrt{\dfrac{2}{5}}\Rightarrow y=\pm\dfrac{3\sqrt{5}}{2}\sqrt{\dfrac{2}{5}}=\pm\dfrac{3\sqrt{2}}{2}. \end{equation*}

Abbiamo quindi trovato altri quattro punti critici:

\begin{equation*}     \begin{split}         &P_5=\left(\sqrt{\dfrac{2}{5}},\dfrac{3\sqrt{2}}{2},-\sqrt{\dfrac{2}{5}}\right),\qquad P_6=\left(\sqrt{\dfrac{2}{5}},-\dfrac{3\sqrt{2}}{2},-\sqrt{\dfrac{2}{5}}\right)\\         &P_7=\left(-\sqrt{\dfrac{2}{5}},\dfrac{3\sqrt{2}}{2},\sqrt{\dfrac{2}{5}}\right),\qquad P_8=\left(-\sqrt{\dfrac{2}{5}},-\dfrac{3\sqrt{2}}{2},\sqrt{\dfrac{2}{5}}\right).     \end{split} \end{equation*}

Abbiamo trovato 8 candidati ad essere massimo e minimo di f su A.

Mettiamo in evidenza una sostanziale differenza che sussiste tra i primi quattro candidati e gli altri. I punti P_1, P_2, P_3 e P_4 sono in realtà punti critici di f che si trovano su A. Il teorema dei moltiplicatori di Lagrange rileva banalmente anche gli stessi punti critici di f1 appartenenti al vincolo, poiché il vettore nullo è complanare ad ogni coppia di vettori: basta prendere \lambda=\mu=0 per ottenere la combinazione lineare banale

\[0=\nabla f(x,y,z)=0\nabla g(x,y,z)-0\nabla h(x,y,z).\]

I punti P_5, P_6, P_7 e P_8 sono conclusioni non banali del teorema dei moltiplicatori di Lagrange. Dimostriamo esplicitamente che in tali punti il gradiente di f è complanare con quello di g ed h, ovvero che esiste una combinazione lineare (non nulla) di \nabla g e \nabla h che ricava \nabla f. Iniziamo da P_5:

\begin{equation*} \begin{split}     \nabla f(x,y,z)=(y^2z,2xyz,xy^2)\Rightarrow&\nabla f(P_5)=\nabla f\left(\sqrt{\dfrac{2}{5}},\dfrac{3\sqrt{2}}{2},-\sqrt{\dfrac{2}{5}}\right)=\left(-\dfrac{9}{\sqrt{10}},-\dfrac{6\sqrt{2}}{5},\dfrac{9}{\sqrt{10}}\right);\\     \nabla g(x,y,z)=(1,0,-1)\Rightarrow&\nabla g(P_5)=\nabla g\left(\sqrt{\dfrac{2}{5}},\dfrac{3\sqrt{2}}{2},-\sqrt{\dfrac{2}{5}}\right)=(1,0,1);\\     \nabla h(x,y,z)=\left(\dfrac{x}{2},\dfrac{2}{9}y,2z\right)\Rightarrow&\nabla h(P_5)=\nabla h\left(\sqrt{\dfrac{2}{5}},\dfrac{3\sqrt{2}}{2},-\sqrt{\dfrac{2}{5}}\right)=\left(\dfrac{1}{\sqrt{10}},\dfrac{\sqrt{2}}{3},-\dfrac{4}{\sqrt{10}}\right). \end{split} \end{equation*}

A questo punto dovremo trovare \lambda,\mu\in\mathbb{R} tali che \nabla f(P_5)=\lambda \nabla g(P_5)+\mu\nabla h(P_5). Osserviamo che essendo vettori in \mathbb{R}^3, si tratta di un sistema con tre equazioni e due incognite, ovvero un sistema sovradeterminato. Sarà proprio il fatto che P_5 è soluzione del teorema di Lagrange a rendere risolvibile il sistema risolvibile.

Dobbiamo dunque trovare \lambda,\mu\in\mathbb{R} tali che

\begin{equation*}     \left\{\begin{array}{l}            -\dfrac{9}{\sqrt{10}}=\lambda +\mu\dfrac{1}{\sqrt{10}}\\\\           -\dfrac{6\sqrt{2}}{5}=\mu\dfrac{\sqrt{2}}{3}\\\\           \dfrac{9}{\sqrt{10}}=\lambda-\mu\dfrac{4}{\sqrt{10}}.     \end{array}\right. \end{equation*}

Dalla seconda equazione troviamo subito \mu=-\dfrac{18}{5}. Inseriamo il valore di \mu nella prima equazione per calcolare \lambda:

\begin{equation*}     -\dfrac{9}{\sqrt{10}}=\lambda -\dfrac{18}{5\sqrt{10}}\Leftrightarrow\lambda=\dfrac{-45+18}{5\sqrt{10}}=-\dfrac{27}{5\sqrt{10}}. \end{equation*}

A questo punto vediamo se la terza equazione è soddisfatta. Ciò implicherebbe la complanarità dei tre vettori gradienti in P_5:

\begin{equation*}     \dfrac{9}{\sqrt{10}}=\lambda-\mu\dfrac{4}{\sqrt{10}}=-\dfrac{27}{5\sqrt{10}}+\dfrac{18}{5}\dfrac{4}{\sqrt{10}}=\dfrac{45}{5\sqrt{10}}=\dfrac{9}{\sqrt{10}}. \end{equation*}

\[\quad\]

\[\quad\]

\[\quad\]

massimi e minimi vincolati

Figura 23: illustrazione della complanarità dei gradienti di f, g e h in P_5. La stessa situazione si verifica per P_6, P_7 e P_8.

\[\quad\]

\[\quad\]

La complanarità dei punti P_6, P_7 e P_8 si dimostra in modo del tutto analogo.

Giunti a questo punto, non ci resta che calcolare il valore attraverso f degli otto candidati e confrontarli.

\begin{equation*}     \begin{split}         &f(P_1)=f\left(\dfrac{2}{\sqrt{5}},0,-\dfrac{2}{\sqrt{5}}\right)=0,\qquad f(P_2)=f\left(-\dfrac{2}{\sqrt{5}},0,\dfrac{2}{\sqrt{5}}\right)=0,\qquad f(P_3)=f(0,3,0)=0\\         &f(P_4)=f(0,-3,0)=0,\qquad f(P_5)=f\left(\sqrt{\dfrac{2}{5}},\dfrac{3\sqrt{2}}{2},-\sqrt{\dfrac{2}{5}}\right)=-\dfrac{9}{5}, \qquad f(P_6)=f\left(\sqrt{\dfrac{2}{5}},-\dfrac{3\sqrt{2}}{2},-\sqrt{\dfrac{2}{5}}\right)=-\dfrac{9}{5}\\         &f(P_7)=f\left(-\sqrt{\dfrac{2}{5}},\dfrac{3\sqrt{2}}{2},\sqrt{\dfrac{2}{5}}\right)=-\dfrac{9}{5},\qquad f(P_8)=f\left(-\sqrt{\dfrac{2}{5}},-\dfrac{3\sqrt{2}}{2},\sqrt{\dfrac{2}{5}}\right)=-\dfrac{9}{5}.     \end{split} \end{equation*}

\[\quad\]

La soluzione dell’esercizio è la seguente: f ha massimo su A pari a 0, assunto nei punti P_1=\left(\dfrac{2}{\sqrt{5}},0,-\dfrac{2}{\sqrt{5}}\right), P_2=\left(-\dfrac{2}{\sqrt{5}},0,\dfrac{2}{\sqrt{5}}\right), P_3=(0,3,0) e P_4=(0,-3,0) e valore minimo -\dfrac{9}{5}, assunto in P_5=\left(\sqrt{\dfrac{2}{5}},\dfrac{3\sqrt{2}}{2},-\sqrt{\dfrac{2}{5}}\right), P_6=\left(\sqrt{\dfrac{2}{5}},-\dfrac{3\sqrt{2}}{2},-\sqrt{\dfrac{2}{5}}\right), P_7=\left(-\sqrt{\dfrac{2}{5}},\dfrac{3\sqrt{2}}{2},\sqrt{\dfrac{2}{5}}\right) e P_8=\left(-\sqrt{\dfrac{2}{5}},\dfrac{3\sqrt{2}}{2},\sqrt{\dfrac{2}{5}}\right).

   


  1. Tra le ipotesi del teorema dei moltiplicatori di Lagrange vi è che il punto in questione non dev’essere punto critico delle funzioni definenti i vincoli, ma non vi è alcuna ipotesi riguardo il gradiente della funzione di cui si sta calcolando massimo e minimo. Al contrario, ogni suo punto critico soddisfaciente l’equazione del vincolo sarà in particolare punto critico della Lagrangiana con moltiplicatori nulli.

 
 

Esercizio 3  (\bigstar\largewhitestar\largewhitestar\largewhitestar\largewhitestar). Data la funzione f:\mathbb{R}^3\rightarrow\mathbb{R}, definita da

\[f(x,y,z):=x^2+y^2+y(z+x-1),\]

determinare, se esistono, il massimo e il minimo di f sull’insieme

\[A:=\left\{(x,y,z)\in\mathbb{R}^3\mid x+y+z=0, \ x^2+y^2=1\right\}.\]

Svolgimento 1.

massimi e minimi vincolati

Figura 26: A è l’ellisse intersezione tra il cilindro e il piano.

\[\quad\]

\[\quad\]

Osserviamo che f è un polinomio, pertanto è una funzione continua su \mathbb{R}^3, mentre l’insieme A è chiuso e limitato. La chiusura segue dal fatto che A è intersezione di controimmagini di insiemi chiusi di \mathbb{R} tramite funzioni continue, mentre per la limitatezza dimostriamo che esso è contenuto in una palla di raggio R=\sqrt{3}. Sia infatti (x,y,z)\in A qualsiasi, stimiamo il quadrato della sua norma:

\begin{equation*}     ||(x,y,z)||^2=x^2+y^2+z^2=1+z^2, \end{equation*}

dove abbiamo sfruttato il fatto che su A vale x^2+y^2=1.

L’altra relazione definente A implica z=-x-y, che inserita nella precedente equazione ci dà

\begin{equation*}     ||(x,y,z)||^2=1+x^2+y^2-2xy=2-2xy. \end{equation*}

Ora non ci resta che valutare il contributo di h(x,y):=xy su A. Possiamo parametrizzare la proiezione A'=\pi(A)\subset\mathbb{R}^2 di A sul piano xy mediante

\begin{equation*}     \gamma(t):=(\cos t,\sin t), \qquad t\in[0,2\pi], \end{equation*}

dunque stimare h su A studiando h\circ\gamma su [0,2\pi]:

\begin{equation*}     (h\circ\gamma)(t)=\cos t\sin t, \end{equation*}

\begin{equation*} \begin{split}     &(h\circ\gamma)'(t)=\cos^2(t)-\sin^2(t)=(\cos t+\sin t)(\cos t-\sin t)=0\Leftrightarrow\\     &\Leftrightarrow \cos t=\pm\sin t\Leftrightarrow t_1=\dfrac{\pi}{4}\lor t_2=\dfrac{3}{4}\pi\lor t_3=\dfrac{5}{4}\pi\lor t_4=\dfrac{7}{4}\pi. \end{split} \end{equation*}

Su ciascuno di questi valori t_k osserviamo che |h(t_k)|=\dfrac{1}{2}. Ciò significa che -\dfrac{1}{2}\leq xy\leq\dfrac{1}{2}. Tale informazione può essere utilizzata per stimare definitivamente la norma di (x,y,z):

\begin{equation*}     ||(x,y,z)||^2=2-2xy\leq2+1=3 \end{equation*}

e questo conclude la dimostrazione del fatto che A è un insieme limitato, contenuto in una sfera di raggio R=\sqrt{3}.

Per il teorema di Heine-Borel A è un insieme compatto di \mathbb{R}^3. Possiamo allora applicare il teorema di Weierstrass, il quale garantisce l’esistenza del massimo e del minimo di f su A.

Notiamo che A^\circ=\emptyset, cioè A coincide con la sua frontiera. Utilizziamo il teorema dei moltiplicatori di Lagrange, osservando che A è intersezione di due vincoli: A=\mathcal{C}\cap\mathcal{P}, dove \mathcal{C} è il cilindro

\begin{equation*}     \mathcal{C}:=\{(x,y,z)\in\mathbb{R}^3\mid x^2+y^2=1\} \end{equation*}

e \mathcal{P} è il piano di equazione x+y+z=0.

\[\quad\]

\[\quad\]

\[\quad\]

massimi e minimi vincolati

\[\quad\]

\[\quad\]

Esprimiamo i vincoli come luoghi di zeri di funzioni per applicare il metodo dei moltiplicatori:

\begin{equation*}     \mathcal{C}:=\{(x,y,z)\in\mathbb{R}^3\mid g(x,y,z)=0\}, \end{equation*}

con

\begin{equation*}     g(x,y,z):=x^2+y^2-1 \end{equation*}

e

\begin{equation*}     \mathcal{P}:=\{(x,y,z)\in\mathbb{R}^3\mid h(x,y,z)=0\}, \end{equation*}

con

\begin{equation*}     h(x,y,z):=x+y+z. \end{equation*}

Il teorema dei moltiplicatori di Lagrange, declinato in questo contesto, afferma che, in corrispondenza dei punti di massimo e di minimo relativi di f su A=\mathcal{P}\cap\mathcal{E} succede un fenomeno che possiamo rileggere sia in chiave geometrica che algebrica: geometricamente lo si può interpretare dal fatto che i gradienti di f, g ed h sono complanari. Da un punto di vista algebrico, detto (x,y,z) un punto di estremo relativo, esisteranno \lambda,\mu\in\mathbb{R} tali che

\begin{equation*}     \nabla f(x,y,z)=\lambda\nabla g(x,y,z)+\mu\nabla h(x,y,z). \end{equation*}

Per questo motivo, costruiamo la Lagrangiana \mathcal{L}:\mathbb{R}^5\rightarrow\mathbb{R}, definita tramite

\begin{equation*}     \mathcal{L}(x,y,z,\lambda,\mu):=f(x,y,z)-\lambda g(x,y,z)-\mu h(x,y,z). \end{equation*}

Il teorema dei moltiplicatori di Lagrange garantisce che i punti critici di \mathcal{L} corrisponderanno esattamente agli estremi relativi di f su A. Infatti, calcolare i punti critici di \mathcal{L} significa risolvere il sistema

\begin{equation*}     \left\{\begin{array}{l}         \dfrac{\partial\mathcal{L}}{\partial x}=0  \\\\         \dfrac{\partial\mathcal{L}}{\partial y}=0  \\\\         \dfrac{\partial\mathcal{L}}{\partial z}=0  \\\\         \dfrac{\partial\mathcal{L}}{\partial \lambda}=0 \\\\         \dfrac{\partial\mathcal{L}}{\partial \mu}=0  \\\\     \end{array}\right.\Leftrightarrow\left\{\begin{array}{l}         \dfrac{\partial f}{\partial x}-\lambda\dfrac{\partial g}{\partial x}-\mu\dfrac{\partial h}{\partial x}=0  \\\\         \dfrac{\partial f}{\partial y}-\lambda\dfrac{\partial g}{\partial y}-\mu\dfrac{\partial h}{\partial y}=0  \\\\         \dfrac{\partial f}{\partial z}-\lambda\dfrac{\partial g}{\partial z}-\mu\dfrac{\partial h}{\partial z}=0  \\\\         g=0 \\\\         h=0.\\\\     \end{array}\right.\Leftrightarrow\left\{\begin{array}{l}          \nabla f=\lambda\nabla g+\mu\nabla h  \\\\\\          g=0 \\\\         h=0.\\\\     \end{array}\right. \end{equation*}

Le prime tre equazioni affermano proprio che \nabla f appartiene al piano individuato da \nabla g e \nabla h. Le ultime equazioni assicurano che il punto si trovi su \partial A=\{g\equiv 0\}\cap\{h\equiv0\}.

Calcoliamo la Lagrangiana:

\begin{equation*} \begin{split}     \mathcal{L}(x,y,z,\lambda,\mu):=x^2+y^2+y(z+x-1)-\lambda(x^2+y^2)-\mu(x+y+z). \end{split} \end{equation*}

Andiamo a cercare i suoi punti critici, risolvendo con attenzione il seguente sistema

\begin{equation*}     \left\{\begin{array}{l}         \dfrac{\partial\mathcal{L}}{\partial x}=2x+y-2\lambda x-\mu=0  \\\\         \dfrac{\partial\mathcal{L}}{\partial y}=2y+z+x-1-2\lambda y-\mu=0  \\\\         \dfrac{\partial\mathcal{L}}{\partial z}=y-\mu=0  \\\\         \dfrac{\partial\mathcal{L}}{\partial \lambda}=-(x^2+y^2-1)=0 \\\\         \dfrac{\partial\mathcal{L}}{\partial \mu}=-\left(x+y+z\right)=0  \\\\     \end{array}\right. \end{equation*}

La terza equazione impone immediatamente \mu=y, che sostituiamo laddove è utile:

\begin{equation*}     \left\{\begin{array}{l}         2x+y-2\lambda x-y=0  \\\\         2y+z+x-1-2\lambda y-y=0  \\\\         \mu=y  \\\\         x^2+y^2-1=0 \\\\         x+y+z=0.  \\\\     \end{array}\right.\Leftrightarrow\left\{\begin{array}{l}         2x(1-\lambda)=0  \\\\         y+z+x-1-2\lambda y=0  \\\\         \mu=y  \\\\         x^2+y^2-1=0 \\\\         x+y+z=0.  \\\\     \end{array}\right. \end{equation*}

Confrontando l’ultima equazione con la seconda, possiamo riscrivere il sistema in questo modo:

\begin{equation*}     \left\{\begin{array}{l}         x(1-\lambda)=0  \\\\         -1-2\lambda y=0  \\\\         \mu=y  \\\\         x^2+y^2-1=0 \\\\         x+y+z=0.  \\\\     \end{array}\right. \end{equation*}

A questo punto, osserviamo la prima equazione: essa è soddisfatta solamente se x=0 oppure se \lambda =1. Se supponiamo x=0, il sistema si risolve immediatamente

\begin{equation*}     \left\{\begin{array}{l}         x=0  \\\\         \lambda y=\mp\dfrac{1}{2}  \\\\         \mu=\pm1  \\\\         y^2=\pm1 \\\\         z=\mp1.  \\\\     \end{array}\right. \end{equation*}

Abbiamo quindi trovato i primi due punti critici della Lagrangiana:

\[P_1=(0,1,-1)\qquad \text{e}\qquad P_2=(0,-1,1).\]

Supponiamo invece \lambda=1, da cui risulta

\begin{equation*}     \left\{\begin{array}{l}         \lambda=1  \\\\         -1-2 y=0  \\\\         \mu=y  \\\\         x^2+y^2-1=0 \\\\         x+y+z=0  \\\\     \end{array}\right.\Leftrightarrow\left\{\begin{array}{l}         \lambda=1  \\\\         y=-\dfrac{1}{2}  \\\\         \mu=-\dfrac{1}{2}  \\\\         x^2+\dfrac{1}{4}-1=0 \\\\         x-\dfrac{1}{2}+z=0  \\\\     \end{array}\right.\Leftrightarrow\left\{\begin{array}{l}         \lambda=1  \\\\         y=-\dfrac{1}{2}  \\\\         \mu=-\dfrac{1}{2}  \\\\         x=\pm\dfrac{\sqrt{3}}{2} \\\\         z=\dfrac{1\mp\sqrt{3}}{2}.  \\\\     \end{array}\right. \end{equation*}

Abbiamo trovato altri due punti critici, ovvero

\begin{equation*}     P_3=\left(\dfrac{\sqrt{3}}{2},-\dfrac{1}{2},\dfrac{1-\sqrt{3}}{2}\right)\qquad\text{e}\qquad P_4=\left(\dfrac{-\sqrt{3}}{2},-\dfrac{1}{2},\dfrac{1+\sqrt{3}}{2}\right). \end{equation*}

Abbiamo trovato quattro candidati ad essere punti di massimo e di minimo di f su A.

\[\quad\]

\[\quad\]

\[\quad\]

massimi e minimi vincolati

Figura 27: tutti i candidati ad essere punti di massimo e di minimo di f su A.

\[\quad\]

\[\quad\]

Calcoliamo i loro valori tramite f e confrontiamoli:

\begin{equation*}     \begin{split}         f(P_1)&=f(0,1,-1)=-1,\qquad f(P_2)=f(0,-1,1)=1,\\         f(P_3)&=f\left(\dfrac{\sqrt{3}}{2},-\dfrac{1}{2},\dfrac{1-\sqrt{3}}{2}\right)=\dfrac{5}{4} \qquad\text{e}\qquad f(P_4)=f\left(\dfrac{-\sqrt{3}}{2},-\dfrac{1}{2},\dfrac{1+\sqrt{3}}{2}\right)=\dfrac{5}{4}.     \end{split} \end{equation*}

\[\quad\]

La soluzione dell’esercizio è la seguente: f ha massimo su A pari a \dfrac{5}{4}, assunto nei punti P_3=\left(\dfrac{\sqrt{3}}{2},-\dfrac{1}{2},\dfrac{1-\sqrt{3}}{2}\right) e P_4=\left(\dfrac{-\sqrt{3}}{2},-\dfrac{1}{2},\dfrac{1+\sqrt{3}}{2}\right) e valore minimo -1, assunto in P_1=(0,1,-1).

Svolgimento 2.

Possiamo risolvere il problema in maniera puramente algebrica. Infatti, utilizzando le equazioni dei vincoli

\begin{equation*} \left\{\begin{array}{l}       x^2+y^2=1\\\\       x+y+z=0, \end{array}\right. \end{equation*}

la funzione diventa

\begin{equation*}     f(x,y,z)=x^2+y^2+y(z+x+y-y-1)=1+y(-y-1)=-y^2-y+1,\qquad\text{con }y\in[-1,1]. \end{equation*}

L’espressione della funzione si semplifica estremamente, risultando funzione della sola variabile y. Definiamo (con abuso di notazione) la funzione di una singola variabile

\begin{equation*}     \begin{array}{rcl}         f:[-1,1]\subset\mathbb{R} & \rightarrow&\mathbb{R} \\         y & \mapsto&-y^2-y+1     \end{array} \end{equation*}

Possiamo pertanto studiare il massimo ed il minimo di f sull’intervallo [-1,1]. Tale problema ha sicuramente soluzione, grazie al teorema di Weierstrass, essendo f continua e [-1,1] compatto. Una volta trovati i valori di y basterà trovare i punti corrispondenti appartenenti al vincolo.

A tal fine, essendo f anche differenziabile, il teorema di Fermat ci fornisce uno strumento per riconoscere eventuali punti di massimo o minimo di f per quanto riguarda l’insieme aperto (-1,1)=[-1,1]^\circ: tali punti, saranno in particolare punti critici di f. Nulla possiamo invece dire su \partial[-1,1]=\{-1\}\cup\{1\}: questi due valori andranno studiati separatamente.

Come indicato dal teorema di Fermat, studiamo i punti critici di f, ovvero i valori y\in(-1,1) tali che f'(y)=0:

\begin{equation*}     f'(y)=-2y-1=0\Leftrightarrow y=-\dfrac{1}{2}. \end{equation*}

Abbiamo dunque trovato il valore y=-\dfrac{1}{2}, a cui corrispondono i punti soluzione del sistema precedente, con y=-\dfrac{1}{2}:

\begin{equation*}     \left\{\begin{array}{l}          x^2+\dfrac{1}{4}=1\\\\       x-\dfrac{1}{2}+z=0,     \end{array}\right.\Leftrightarrow\left\{\begin{array}{l}          x=\pm\dfrac{\sqrt{3}}{2}\\\\       z=\dfrac{1\mp\sqrt{3}}{2}.     \end{array}\right. \end{equation*}

Abbiamo quindi trovato due candidati:

\begin{equation*} P_1=\left(\dfrac{\sqrt{3}}{2},-\dfrac{1}{2},\dfrac{1-\sqrt{3}}{2}\right)\qquad\text{e}\qquad P_2=\left(\dfrac{-\sqrt{3}}{2},-\dfrac{1}{2},\dfrac{1+\sqrt{3}}{2}\right). \end{equation*}

Dobbiamo aggiungere manualmente i punti corrispondenti agli estremi y=\pm1, poiché non abbiamo alcun motivo per escluderli. Risolviamo nuovamente il sistema con y=\pm1:

\begin{equation*}     \left\{\begin{array}{l}          x^2+1=1\\\\       x\pm1+z=0,     \end{array}\right.\Leftrightarrow\left\{\begin{array}{l}          x=0\\\\       z=\mp1     \end{array}\right. \end{equation*}

Abbiamo trovato altri due candidati

\begin{equation*}     P_3=(0,1,-1)\qquad \text{e} \qquad P_4=(0,-1,1). \end{equation*}

Calcoliamo i loro valori tramite f e confrontiamoli:

\begin{equation*}     \begin{split}         f(P_1)&=f\left(\dfrac{\sqrt{3}}{2},-\dfrac{1}{2},\dfrac{1-\sqrt{3}}{2}\right)=\dfrac{5}{4},\qquad f(P_2)=f\left(\dfrac{-\sqrt{3}}{2},-\dfrac{1}{2},\dfrac{1+\sqrt{3}}{2}\right)=\dfrac{5}{4},\\         f(P_3)&=f(0,1,-1)=-1 \qquad\text{e}\qquad f(P_4)=f(0,-1,1)=1.     \end{split} \end{equation*}

\[\quad\]

La soluzione dell’esercizio è la seguente: f ha massimo su A pari a \dfrac{5}{4}, assunto nei punti P_1=\left(\dfrac{\sqrt{3}}{2},-\dfrac{1}{2},\dfrac{1-\sqrt{3}}{2}\right) e P_2=\left(\dfrac{-\sqrt{3}}{2},-\dfrac{1}{2},\dfrac{1+\sqrt{3}}{2}\right) e valore minimo -1, assunto in P_3=(0,1,-1).

Svolgimento 3.

Il terzo modo per risolvere il problema sfrutta l’esplicita prametrizzazione di A. Per trovarla, indaghiamo il sistema contenente le equazioni dei vincoli:

\begin{equation*} \left\{\begin{array}{l}       x^2+y^2=1\\\\       x+y+z=0, \end{array}\right.\Leftrightarrow\left\{\begin{array}{l}       x^2+y^2=1\\\\       z=-x-y. \end{array}\right. \end{equation*}

La prima equazione suggerisce di parametrizzare le prime due variabili attraverso le coordinate polari: (x,y)=(\cos\theta,\sin\theta), con \theta\in[0,2\pi], mentre dall’ultima equazione otteniamo z=-\cos\theta-\sin\theta. Ecco dunque ottenuta la parametrizzazione di A=\gamma([0,2\pi]), con

\begin{equation*}     \begin{array}{rcl}          \gamma:[0,2\pi]&\rightarrow&\mathbb{R}^3  \\          \theta&\mapsto&(\cos\theta,\sin\theta,-\cos\theta-\sin\theta).      \end{array} \end{equation*}

\[\quad\]

\[\quad\]

\[\quad\]

massimi e minimi vincolati

\[\quad\]

\[\quad\]

A questo punto, basterà studiare la funzione composta f\circ\gamma sull’intervallo [0,2\pi]. Essendo f e \gamma continue, anche la loro composizione lo è, inoltre [0,2\pi] è un insieme compatto, quindi per il teorema di Weierstrass troveremo il massimo ed il minimo di f\circ\gamma su [0,2\pi]. Poichè la parametrizzazione \gamma è 2\pi-periodica, non sarà necessario studiare gli estremi del dominio \{0\} e \{2\pi\} (questo è facilmente intuibile dal fatto che la periodicità permette di traslare a piacimento il dominio della parametrizzazione, cambiando in questo modo gli estremi del dominio), ma sarà sufficiente studiare i punti critici di f\circ\gamma.

Calcoliamo dunque la funzione composta e la sua derivata:

\begin{equation*}     f(\gamma(\theta))=f(\cos\theta,\sin\theta,-\cos\theta-\sin\theta)=\cos^2(\theta)+\sin^2(\theta)+\sin(\theta)(-\cos(\theta)-\sin(\theta)+\cos(\theta)-1)=1-\sin^2(\theta)-\sin(\theta), \end{equation*}

\begin{equation*}     (f\circ\gamma)'(\theta)=-2\sin(\theta)\cos(\theta)-\cos(\theta)=-\cos(\theta)(1+2\sin(\theta)) \end{equation*}

Dunque i punti critici di f\circ\gamma soddisfano

\begin{equation*}     \cos(\theta)=0\qquad\lor\qquad\sin(\theta)=-\dfrac{1}{2} \end{equation*}

a cui corrispondono i valori di \theta\in[0,2\pi]

\begin{equation*}     \theta_1=\dfrac{\pi}{2},\qquad\theta_2=\dfrac{3}{2}\pi,\qquad\theta_3=\dfrac{7}{6}\pi,\qquad\theta_4=\dfrac{11}{6}\pi. \end{equation*}

Calcoliamo le corrispondenti immagini attraverso la parametrizzazione:

\begin{equation*} \begin{split}     &P_1=\gamma(\theta_1)=(0,1,-1),\qquad P_2=\gamma(\theta_2)=(0,-1,1)\\ &P_3=\gamma(\theta_3)=\left(\dfrac{-\sqrt{3}}{2},-\dfrac{1}{2},\dfrac{1+\sqrt{3}}{2}\right),\qquad P_4=\gamma(\theta_4)=\left(\dfrac{\sqrt{3}}{2},-\dfrac{1}{2},\dfrac{1-\sqrt{3}}{2}\right). \end{split} \end{equation*}

\[\quad\]

\[\quad\]

\[\quad\]

massimi e minimi vincolati

\[\quad\]

\[\quad\]

Calcoliamo i loro valori tramite f e confrontiamoli:

\begin{equation*}     \begin{split}         f(P_1)&=f(0,1,-1)=-1,\qquad f(P_2)=f(0,-1,1)=1,\\         f(P_3)&=f\left(-\dfrac{\sqrt{3}}{2},-\dfrac{1}{2},\dfrac{1+\sqrt{3}}{2}\right)=\dfrac{5}{4} \qquad\text{e}\qquad f(P_4)=f\left(\dfrac{\sqrt{3}}{2},-\dfrac{1}{2},\dfrac{1-\sqrt{3}}{2}\right)=\dfrac{5}{4}.     \end{split} \end{equation*}

\[\quad\]

La soluzione dell’esercizio è la seguente: f ha massimo su A pari a \dfrac{5}{4}, assunto nei punti P_3=\left(-\dfrac{\sqrt{3}}{2},-\dfrac{1}{2},\dfrac{1+\sqrt{3}}{2}\right) e P_4=\left(\dfrac{\sqrt{3}}{2},-\dfrac{1}{2},\dfrac{1-\sqrt{3}}{2}\right) e valore minimo -1, assunto in P_1=(0,1,-1).

 
 

Esercizio 4  (\bigstar\bigstar\bigstar\largewhitestar\largewhitestar). Data la funzione f:\mathbb{R}^3\rightarrow\mathbb{R}, definita da

\[f(x,y,z):=2xy+xz^2,\]

determinare, se esistono, il massimo e il minimo di f sull’insieme

\[A:=\left\{(x,y,z)\in\mathbb{R}^3\mid x^2+y^2+z^2\leq3, \ x^2+z^2-y^2\leq 1\right\}.\]

Svolgimento.

massimi e minimi vincolati

Figura 34: l’insieme A.

\[\quad\]

\[\quad\]

Osserviamo che f è un polinomio, pertanto è una funzione continua su \mathbb{R}^3, mentre l’insieme A è chiuso e limitato. La chiusura segue dal fatto che A è intersezione di controimmagini di insiemi chiusi di \mathbb{R} tramite funzioni continue, mentre per la limitatezza, la prima condizione assicura che A è contenuto nella palla di centro l’origine e raggio \sqrt{3}. Dunque è un chiuso e limitato di \mathbb{R}^3, compatto per il teorema di Heine-Borel. Possiamo allora applicare il teorema di Weierstrass, il quale garantisce l’esistenza del massimo e del minimo di f su A.

Distinguiamo l’insieme A nella sua parte interna e la sua frontiera: A=A^\circ\sqcup\partial A, dove

\begin{equation*}     A^\circ:=\left\{(x,y,z)\in\mathbb{R}^3\mid x^2+y^2+z^2<3, \ x^2+z^2-y^2< 1\right\} \end{equation*}

e

\begin{equation*}     \partial A=A\setminus A^\circ. \end{equation*}

Studiamo separatamente e in maniera differente le due componenti.

A^\circ) Essendo f differenziabile sull’insieme aperto A^\circ, il teorema di Fermat ci assicura che gli estremi relativi di f in A^\circ vanno ricercati tra i suoi eventuali punti critici in A^\circ, ovvero i punti P\in A^\circ tali che \nabla f(P)=(0,0,0) e scartare tutti i punti che non soddisfano tale condizione. Calcoliamo il gradiente di f svolgendo le derivate parziali:

\[\begin{cases} \dfrac{\partial f}{\partial x}(x,y)=2y+z^2\\\\ \dfrac{\partial f}{\partial y}(x,y)=2x\\\\ \dfrac{\partial f}{\partial z}(x,y)=2xz, \end{cases}\]

dunque

\[\nabla f(x,y,z)=(2y+z^2,2x,2xz).\]

Pertanto, i punti critici di f soddisfano

\begin{equation*}     \begin{cases}     2y+z^2=0\\\\     2x=0\\\\     2xz=0     \end{cases}\Leftrightarrow     \begin{cases}     y=-\dfrac{z^2}{2}\\\\     x=0.     \end{cases} \end{equation*}

Possiamo allora parametrizzare i punti critici di f attraverso la curva

\begin{equation*} \gamma(t):=\left(0,-\dfrac{t^2}{2},t\right). \end{equation*}

Naturalmente, siamo interessati alla porzione di curva che giace in A^\circ. Indaghiamo la natura di questi punti critici attraverso lo studio della matrice Hessiana, valutata sui punti della curva.

Ricordiamo, prima di svolgere esplicitamente i calcoli, l’importante teorema che sta alla base di questo procedimento:

Sia f:A\subset\mathbb{R}^n\rightarrow\mathbb{R} di classe \mathcal{C}^2(A), con A sottoinsieme aperto e x_0\in A un punto critico di f. Sia \mathcal{H}_f(x_0) la matrice Hessiana di f valutata in x_0, allora se

\cdot \mathcal{H}_f(x_0) è definita positiva, x_0 è un punto di minimo relativo per f

\cdot \mathcal{H}_f(x_0) è definita negativa, x_0 è un punto di massimo relativo per f

\cdot \mathcal{H}_f(x_0) è indefinita, x_0 è un punto di sella per f.

Ricordiamo infine cosa significa per una matrice simmetrica (come la matrice Hessiana di una funzione di classe \mathcal{C}^2) essere definita o indefinita:

una matrice simmetrica si dice definita positiva (negativa rispettivamente) se tutti i suoi autovalori sono positivi (negativi risp.)

una matrice simmetrica si dice indefinita se esistono almeno due autovalori di segno opposto.

Consci del precedente teorema, calcoliamo la matrice Hessiana di f in un generico punto (x,y,z):

\begin{equation*}     \mathcal{H}_f(x,y,z)=\begin{pmatrix}     0&2&2z\\     2&0&0\\     2z&0&2x     \end{pmatrix} \end{equation*}

e valutiamola, dunque, nei punti appartenenti alla curva

\begin{equation*}     \mathcal{H}_f(\gamma(t))=\begin{pmatrix}     0&2&2t\\     2&0&0\\     2t&0&0     \end{pmatrix}. \end{equation*}

A questo punto, per indagare la natura dei punti critici giacenti sulla curva, dovremo studiare la definitezza della matrice Hessiana su ciascuno di questi punti. Per farlo, calcoliamo il segno degli autovalori della matrice, che saranno in funzione del parametro t, di evoluzione della curva. Per definizione, gli autovalori della matrice \mathcal{H}_f(\gamma(t)) sono i valori \lambda\in\mathbb{R}, radici del polinomio caratteristico (possiamo affermare che gli autovalori saranno numeri reali, poiché la matrice Hessiana è una matrice simmetrica):

\begin{equation*}     P(\lambda):=\det(\mathcal{H}_f(\gamma(t)-\lambda I_3), \end{equation*}

dove I_3 è la matrice identità di dimensione 3.

Calcoliamo P(\lambda) e cerchiamo le sue radici.

\begin{equation*}     \mathcal{H}_f(\gamma(t))-\lambda I_3=\begin{pmatrix}     0&2&2t\\     2&0&0\\     2t&0&0     \end{pmatrix}-\lambda\begin{pmatrix}     1&0&0\\     0&1&0\\     0&0&1     \end{pmatrix}=\begin{pmatrix}     -\lambda&2&2t\\     2&-\lambda&0\\     2t&-0&-\lambda     \end{pmatrix}, \end{equation*}

quindi

\begin{equation*}     P(\lambda)=\det \begin{pmatrix}     -\lambda&2&2t\\     2&-\lambda&0\\     2t&0&-\lambda     \end{pmatrix}=-\lambda^3+4\lambda-4\lambda t^2=-\lambda(\lambda^2-4-4t^2). \end{equation*}

I valori che annullano P(\lambda) sono

\begin{equation*}     \lambda_1=0, \end{equation*}

\begin{equation*}     \lambda_{2,3}=\pm2\sqrt{1+t^2}. \end{equation*}

Osserviamo che, indipendentemente dal parametro t, gli autovalori \lambda_2 e \lambda_3 hanno segno opposto, ovvero la matrice Hessiana \mathcal{H}_f(\gamma(t)) è ovunque indefinita e per il teorema sopra citato, ne evinciamo che tutti i punti sulla curva sono punti di sella, che dunque non possono essere ne massimi ne minimi relativi.

Questo conclude lo studio di A^\circ.

\partial A) Innanzitutto, dobbiamo capire com’è fatto \partial A. Osserviamo che

\begin{equation*} \begin{split}     \partial A &= \{(x,y,z)\in\mathbb{R}^3\mid x^2+y^2+z^2=3, \ x^2+z^2-y^2\leq 1\}\cup \\ & \quad \cup \{(x,y,z)\in\mathbb{R}^3\mid x^2+y^2+z^2\leq3, \ x^2+z^2-y^2= 1\}. \end{split} \end{equation*}

Analizziamo la prima componente, mettendo a sistema l’equazione con la disequazione:

\begin{equation*}     \left\{\begin{array}{l}           x^2+y^2+z^2=3 \\\\           x^2+z^2-y^2\leq 1     \end{array}     \right.\Leftrightarrow\left\{\begin{array}{l}           x^2+z^2=3-y^2 \\\\           3-2y^2\leq 1     \end{array}     \right.\Leftrightarrow\left\{\begin{array}{l}           x^2+z^2=3-y^2 \\\\           y^2\geq 1     \end{array}\right.     \Leftrightarrow\left\{\begin{array}{l}           x^2+z^2=3-y^2 \\\\           y\leq -1\lor y\geq 1.     \end{array}\right. \end{equation*}

Ovviamente, affinché la prima equazione abbia soluzione nel campo dei reali, si richiede anche y\in\left[-\sqrt{3},\sqrt{3}\right]. Pertanto, la prima componente rappresenta due porzioni di sfera di raggio \sqrt{3}, S^\pm, definite tramite

\begin{equation*}     S^+:=\left\{(x,y,z)\in\mathbb{R}^3\mid x^2+y^2+z^2=3, \ y\in\left[1,\sqrt{3}\right]\right\} \end{equation*}

e

\begin{equation*}     S^-:=\left\{(x,y,z)\in\mathbb{R}^3\mid x^2+y^2+z^2=3, \ y\in\left[-\sqrt{3},-1\right]\right\}. \end{equation*}

\[\quad\]

\[\quad\]

\[\quad\]

massimi e minimi vincolati

Figura 35: le due parti di sfera che compongono la frontiera di A.

\[\quad\]

\[\quad\]

Ripetiamo lo stesso ragionamento anche per la seconda componente, mettendo a sistema l’equazione e la disequazione che la definiscono

\begin{equation*}     \left\{\begin{array}{l}           x^2+y^2+z^2\leq3 \\\\           x^2+z^2-y^2= 1     \end{array}     \right.\Leftrightarrow\left\{\begin{array}{l}            1+2y^2\leq 3\\\\           x^2+z^2=1+y^2     \end{array}     \right.\Leftrightarrow\left\{\begin{array}{l}            y^2\leq 1\\\\           x^2+z^2=1+y^2     \end{array}     \right.\Leftrightarrow\left\{\begin{array}{l}            -1\leq y\leq1\\\\           x^2+z^2=1+y^2.     \end{array}     \right. \end{equation*}

Abbiamo trovato che la seconda componente è una porzione di iperboloide I, rappresentato da

\begin{equation*}     I:=\left\{(x,y,z)\in\mathbb{R}^3\mid x^2+z^2-y^2=1, \ y\in\left[-1,1\right]\right\}. \end{equation*}

\[\quad\]

\[\quad\]

\[\quad\]

massimi e minimi vincolati

Figura 36: la parte di ipeboloide che compone il resto di \partial A.

\[\quad\]

\[\quad\]

Siamo dunque in grado di esprimere \partial A attraverso

\begin{equation*}     \partial A=S^-\cup I\cup S^+. \end{equation*}

A questo punto, studiamo separatamente il comportamento di f ristretto a S^\pm e I, separatamente.

S^+) Essendo S^+ una porzione di sfera, utilizziamo le coordinate sferiche

\begin{equation*}         \psi(\theta,\varphi):=\left(\sqrt{3}\cos\theta\sin\varphi,\sqrt{3}\cos\varphi,\sqrt{3}\sin\theta\sin\varphi\right),     \end{equation*}

prendendo inizialmente come dominio (\theta,\varphi)\in[0,2\pi]\times[0,\pi]. In questo modo, tuttavia, otteniamo una parametrizzazione dell’intera sfera di centro l’origine e raggio \sqrt{3}. Per parametrizzare S^+, dovremo restringere il dominio della parametrizzazione \psi. In particolare, notiamo che S^+ è definito dal vincolo y\in\left[1,\sqrt{3}\right], da cui troviamo

\begin{equation*}         \begin{cases}         \sqrt{3}\cos\varphi\geq1\\\\         \sqrt{3}\cos\varphi\leq\sqrt{3}         \end{cases}         \Leftrightarrow         \begin{cases}         \cos\varphi\geq\dfrac{\sqrt{3}}{3}\\\\         \cos\varphi\leq1         \end{cases}         \Leftrightarrow \varphi\in\bigcup_{k\in\mathbb{Z}}\left[-\arccos{\left(\dfrac{\sqrt{3}}{3}\right)}+2k\pi,\arccos{\left(\dfrac{\sqrt{3}}{3}\right)}+2k\pi\right].     \end{equation*}

Intersecando il vincolo appena trovato con il dominio di partenza relativo a \varphi, \varphi\in[0,\pi], otteniamo

\[\varphi\in\left[0,\arccos{\left(\dfrac{\sqrt{3}}{3}\right)}\right].\]

Nessun vincolo, invece, riguarda \theta, libero di variare in [0,2\pi]. Affermiamo che una parametrizzazione regolare di S^+ sarà

\begin{equation*}         \begin{array}{ccl}              \psi^+:[0,2\pi]\times\left[0,\arccos{\left(\dfrac{\sqrt{3}}{3}\right)}\right]&\rightarrow&\mathbb{R}^3  \\\\              (\theta,\varphi)&\mapsto& \left(\sqrt{3}\cos\theta\sin\varphi,\sqrt{3}\cos\varphi,\sqrt{3}\sin\theta\sin\varphi\right),         \end{array}     \end{equation*}

infatti, detto U:=(0,2\pi)\times\left(0,\arccos{\left(\dfrac{\sqrt{3}}{3}\right)}\right), risulta che U è un aperto, la cui chiusura è compatta, \psi^+|_U è iniettiva e di classe \mathcal{C}^1 su U, \det J_{\psi^+}\neq 0 su U ed infine vale

\[\psi^+\left([0,2\pi]\times\left[0,\arccos{\left(\dfrac{\sqrt{3}}{3}\right)}\right]\right)=S^+.\]

\[\quad\]

\[\quad\]

\[\quad\]

massimi e minimi vincolati

Figura 37: la parametrizzazione di S^+.

\[\quad\]

\[\quad\]

Questo ragionamento ci permette di rimpiazzare lo studio del massimo e del minimo di f sulla varietà S^+\subset\mathbb{R}^3 con lo studio della funzione composta f\circ\psi^+ sulla regione [0,2\pi]\times\left[0,\arccos{\left(\dfrac{\sqrt{3}}{3}\right)}\right]\subset\mathbb{R}^2, il che comporta un vantaggio notevole, vista la semplicità del dominio.

Per prima cosa, calcoliamo esplicitamente la composizione delle due funzioni:

\begin{equation*}         (f\circ\psi^+)(\theta,\varphi)=f\left(\sqrt{3}\cos\theta\sin\varphi,\sqrt{3}\cos\varphi,\sqrt{3}\sin\theta\sin\varphi\right)=6\cos\theta\sin\varphi\cos\varphi+3\sqrt{3}\cos\theta\sin^2\theta\sin^3\varphi.     \end{equation*}

Cerchiamo, dunque, il massimo ed il minimo della funzione f\circ\psi^+ su [0,2\pi]\times\left[0,\arccos{\left(\dfrac{\sqrt{3}}{3}\right)}\right]. Essendo f e \psi^+ di classe \mathcal{C}^1, sono a fortiori continue, cosi come lo è la loro composizione, inoltre [0,2\pi]\times\left[0,\arccos{\left(\dfrac{\sqrt{3}}{3}\right)}\right] è un compatto di \mathbb{R}^2, dunque per il Teorema di Weierstrass, esisterà il massimo ed il minimo di f\circ\psi^+ su [0,2\pi]\times\left[0,\arccos{\left(\dfrac{\sqrt{3}}{3}\right)}\right].

Vorremmo distinguere la parte interna di [0,2\pi]\times\left[0,\arccos{\left(\dfrac{\sqrt{3}}{3}\right)}\right], che abbiamo denotato con U e la sua frontiera, che andranno studiate in maniera diversa. Osserviamo tuttavia che la frontiera della varietà S^+ è parametrizzata da

\[\partial S^+=\psi^+\left([0,2\pi]\times\left\{\arccos{\left(\dfrac{\sqrt{3}}{3}\right)}\right\}\right).\]

Ciò significa che è possibile riparametrizzare S^+ in maniera tale che i valori di \theta=0,2\pi e \varphi=0 non siano più estremi del dominio, ma stiano nella sua parte interna. Dunque, detti R:=[0,2\pi]\times\left[0,\arccos{\left(\dfrac{\sqrt{3}}{3}\right)}\right] ed L:=[0,2\pi]\times\left\{\arccos{\left(\dfrac{\sqrt{3}}{3}\right)}\right\}, per lo studio dei punti di massimo e minimo è conveniente distinguere R\setminus L, da affrontare mediante il teorema di Fermat ed L, attraverso una sua esplicita parametrizzazione.

\[\quad\]

\[\quad\]

\[\quad\]

massimi e minimi vincolati

Figura 38: studio della frontiera di S^+.

\[\quad\]

\[\quad\]

R\setminus L) Utilizziamo il teorema di Fermat per trovare gli eventuali punti di massimo e minimo relativo di f\circ\psi^+ su R\setminus L, nonostante esso non sia un insieme aperto, per le considerazioni fatte in precedenza. Su R\setminus L, condizione necessaria affinché un punto (\theta_0,\varphi_0)\in U sia punto di massimo o minimo relativo della funzione differenziabile f\circ\psi^+ su U è che esso sia un suo punto critico, ovvero che si verifichi \nabla(f\circ\psi^+)(\theta_0,\varphi_0)=(0,0). Calcoliamo allora il gradiente di f\circ\psi^+ attraverso il calcolo delle derivate parziali:

\begin{equation*}             \begin{split}                 \dfrac{\partial}{\partial\theta}(f\circ\psi^+)&=-6\sin\theta\sin\varphi\cos\varphi-3\sqrt{3}\sin^3\theta\sin^3\varphi+6\sqrt{3}\cos^2\theta\sin\theta\sin^3\varphi\\                 &=3\sin\theta\sin\varphi\left(-2\cos\varphi-\sqrt{3}\sin^2\theta\sin^2\varphi+2\sqrt{3}\cos^2\theta\sin^2\varphi\right),             \end{split}         \end{equation*}

\begin{equation*}             \begin{split}                 \dfrac{\partial}{\partial\varphi}(f\circ\psi^+)&=6\cos\theta\left(\cos^2\varphi-\sin^2\varphi\right)+9\sqrt{3}\cos\theta\sin^2\theta\cos\varphi\sin^2\varphi\\                 &=3\cos\theta\left(2\cos^2\varphi-2\sin^2\varphi+3\sqrt{3}\sin^2\theta\cos\varphi\sin^2\varphi\right),             \end{split}         \end{equation*}

dunque

\begin{equation*}             \begin{split}                 \nabla (f\circ\psi^+)=&\left(3\sin\theta\sin\varphi\left(-2\cos\varphi-\sqrt{3}\sin^2\theta\sin^2\varphi+2\sqrt{3}\cos^2\theta\sin^2\varphi\right),\right.\\                 &\qquad\left.3\cos\theta\left(2\cos^2\varphi-2\sin^2\varphi+3\sqrt{3}\sin^2\theta\cos\varphi\sin^2\varphi\right)\right).             \end{split}         \end{equation*}

Calcoliamo allora i punti critici, ovvero i punti che annullano il gradiente:

(3) \begin{equation*}             \left\{\begin{array}{l}                   3\sin\theta\sin\varphi\left(-2\cos\varphi-\sqrt{3}\sin^2\theta\sin^2\varphi+2\sqrt{3}\cos^2\theta\sin^2\varphi\right)=0 \\\\                   3\cos\theta\left(2\cos^2\varphi-2\sin^2\varphi+3\sqrt{3}\sin^2\theta\cos\varphi\sin^2\varphi\right)=0.             \end{array}\right.         \end{equation*}

Osserviamo immediatamente che la prima equazione si annulla se \sin\theta=0 o \sin\varphi=0. Supponiamo inizialmente \sin\theta=0, che si verifica per \theta=0,\pi, nei quali casi \cos\theta=\pm1 e inserendo quest’informazione nella seconda equazione troviamo

\begin{equation*}             \pm3\left(2\cos^2\varphi-2\sin^2\varphi\right)=0,         \end{equation*}

che si annulla se e solo se

\begin{equation*}             \cos\varphi=\pm\sin\varphi\Leftrightarrow\varphi=\dfrac{\pi}{4}+k\dfrac{\pi}{2}.         \end{equation*}

Notiamo che per k=0,

\[\varphi=\dfrac{\pi}{4}\in\left[0,\arccos{\left(\dfrac{\sqrt{3}}{3}\right)}\right],\]

dunque abbiamo trovato i primi punti da considerare:

\begin{equation*}             \tilde{P}_1=\left(0,\dfrac{\pi}{4}\right),\qquad \tilde{P}_2=\left(\pi,\dfrac{\pi}{4}\right),         \end{equation*}

a cui corrispondono i punti

\begin{equation*}             P_1=\psi^+\left(\tilde{P}_1\right)=\left(\dfrac{\sqrt{6}}{2},\dfrac{\sqrt{6}}{2},0\right),\qquad P_2=\psi^+\left(\tilde{P}_2\right)=\left(-\dfrac{\sqrt{6}}{2},\dfrac{\sqrt{6}}{2},0\right)         \end{equation*}

su S^+.

Se invece \sin\varphi=0, che per l’intervallo interessato da \varphi si verifica solamente per \varphi=0, la seconda equazione si riduce a

\begin{equation*}             6\cos\theta=0\Leftrightarrow\theta=\dfrac{\pi}{2},\dfrac{3}{2}\pi.         \end{equation*}

Abbiamo trovato in questo modo altri due punti da considerare:

\begin{equation*}             \tilde{P}_3=\left(\dfrac{\pi}{2},0\right),\qquad \tilde{P}_4=\left(\dfrac{3}{2}\pi,0\right).         \end{equation*}

Essi hanno la stessa immagine rispetto a \psi^+ e indivuano il punto

\begin{equation*}             P_3=\psi^+\left(\tilde{P}_3\right)=\psi^+\left(\tilde{P}_4\right)=\left(0,\sqrt{3},0\right).         \end{equation*}

La seconda equazione si annulla se \cos\theta=0, ovvero per \theta=\dfrac{\pi}{2},\dfrac{3}{2}\pi, che inserito nella prima equazione implica

\begin{equation*}             \pm3\sin\varphi\left(-2\cos\varphi-\sqrt{3}\sin^2\varphi\right)=0.         \end{equation*}

Questa equazione si annulla se \sin\varphi=0, che ci porta a ritrovare i punti \tilde{P}_3 e \tilde{P}_4, oppure se

\begin{equation*}             -2\cos\varphi-\sqrt{3}\sin^2\varphi=0.         \end{equation*}

Risoviamo l’equazione:

\begin{equation*}             0=2\cos\varphi+\sqrt{3}\sin^2\varphi=2\cos\varphi+\sqrt{3}-\sqrt{3}\cos^2\varphi.         \end{equation*}

Definiamo t:=\cos\varphi, dunque l’equazione precedente risulta essere

\begin{equation*}             0=2t+\sqrt{3}-\sqrt{3}t^2=3t^2-2\sqrt{3}t-3,         \end{equation*}

che si annulla per

\begin{equation*}             t_{1,2}=\dfrac{\sqrt{3}\pm\sqrt{3+9}}{3}=\dfrac{\sqrt{3}\pm2\sqrt{3}}{3}=\begin{cases}             t_1=\sqrt{3}\\\\             t_2=-\dfrac{\sqrt{3}}{3}.             \end{cases}         \end{equation*}

Poiché t=\cos\varphi\in[-1,1], la prima soluzione non è accettabile, mentre la seconda implica

\begin{equation*}             \varphi=\arccos{\left(-\dfrac{\sqrt{3}}{3}\right)}\notin\left[0,\arccos{\left(\dfrac{\sqrt{3}}{3}\right)}\right].         \end{equation*}

Ciò detto, possiamo supporre \sin\theta,\cos\theta,\sin\varphi\neq0, dunque il sistema (3) si riduce a

\begin{equation*}             \left\{\begin{array}{l}                   -2\cos\varphi-\sqrt{3}\sin^2\theta\sin^2\varphi+2\sqrt{3}\cos^2\theta\sin^2\varphi=0 \\\\                   2\cos^2\varphi-2\sin^2\varphi+3\sqrt{3}\sin^2\theta\cos\varphi\sin^2\varphi=0.             \end{array}\right.         \end{equation*}

A questo punto, è conveniente moltiplicare entrambi i membri della prima equazione per \cos\varphi e per farlo, dobbiamo supporre \cos\varphi\neq0, secondo i principi delle equazioni. Tuttavia, prima dobbiamo controllare che non perdiamo soluzioni: se fosse \cos\varphi=0, la seconda equazione implicherebbe \sin\varphi=0, ma nelle nostre ipotesi, \sin\varphi\neq0. Siamo quindi liberi per moltiplicare la prima equazione per \cos\varphi:

\begin{equation*}             \left\{\begin{array}{l}                   -2\cos^2\varphi-\sqrt{3}\sin^2\theta\cos\varphi\sin^2\varphi+2\sqrt{3}\cos^2\theta\cos\varphi\sin^2\varphi=0 \\\\                   2\cos^2\varphi-2\sin^2\varphi+3\sqrt{3}\sin^2\theta\cos\varphi\sin^2\varphi=0.             \end{array}\right.         \end{equation*}

e sommiamo nella prima riga le due equazioni del sistema:

\begin{equation*}             \left\{\begin{array}{l}                   -2\sin^2\varphi+2\sqrt{3}\cos^2\theta\cos\varphi\sin^2\varphi=0 \\\\                   2\cos^2\varphi-2\sin^2\varphi+3\sqrt{3}\sin^2\theta\cos\varphi\sin^2\varphi=0,             \end{array}\right.         \end{equation*}

\begin{equation*}             \left\{\begin{array}{l}                   -2\sin^2\varphi\left(1-\sqrt{3}\cos^2\theta\cos\varphi\right)=0 \\\\                   2\cos^2\varphi-2\sin^2\varphi+3\sqrt{3}\cos\varphi\sin^2\varphi-3\sqrt{3}\cos^2\theta\cos\varphi\sin^2\varphi=0.             \end{array}\right.         \end{equation*}

Poiché \sin\varphi\neq0, la prima equazione comporta

\begin{equation*}             \sqrt{3}\cos^2\theta\cos\varphi=1,         \end{equation*}

che, inserito nella seconda equazione, ci porta all’equazione nella sola variabile \varphi

\begin{equation*}             2\cos^2\varphi-2\sin^2\varphi+3\sqrt{3}\cos\varphi\sin^2\varphi-3\sin^2\varphi=2\cos^2\varphi-5\sin^2\varphi+3\sqrt{3}\cos\varphi\sin^2\varphi=0.         \end{equation*}

Utilizzando ancora l’identità fondamentale goniometrica, troviamo

\begin{equation*}             2\cos^2\varphi+5\cos^2\varphi+3\sqrt{3}\cos\varphi-3\sqrt{3}\cos^3\varphi=0         \end{equation*}

\begin{equation*}             \cos\varphi\left(-3\sqrt{3}\cos^2\varphi+7\cos\varphi+3\sqrt{3}\right)=0         \end{equation*}

Ricordando che \cos\varphi\neq0, la precedente equazione si annulla se

\begin{equation*}             -3\sqrt{3}\cos^2\varphi+7\cos\varphi+3\sqrt{3}=9\cos^2+7\sqrt{3}\cos\theta-9=0,         \end{equation*}

che si verifica per

\begin{equation*}             \cos\theta_{1,2}=\dfrac{7\sqrt{3}\pm\sqrt{147+324}}{18}\notin\left[\dfrac{\sqrt{3}}{3},1\right].         \end{equation*}

Questo conclude lo studio di R\setminus L.

L) Osserviamo che su L, f\circ\psi^+ si riduce a

\begin{equation*}             (f\circ\psi^+)|_L(\theta)=f\left(\sqrt{2}\cos\theta,1,\sqrt{2}\sin\theta\right)=2\sqrt{2}\cos\theta(1+\sin^2\theta).         \end{equation*}

Essendo una funzione derivabile di una variabile, periodica sul suo dominio, basterà studiare i punti che annullano la sua derivata prima:

\begin{equation*}             \begin{split}                 (f\circ\psi^+)'|_L(\theta)&=2\sqrt{2}\left[-\sin\theta\left(1+\sin^2\theta\right)+2\sin\theta\cos\theta\right]\\                 &=2\sqrt{2}\sin\theta\left(3\cos^2\theta-2\right),             \end{split}         \end{equation*}

che si annulla solamente se

\begin{equation*}             \sin\theta=0,\qquad\cos\theta=\pm\sqrt{\dfrac{2}{3}},         \end{equation*}

ovvero per

\begin{equation*}             \theta=0,\pi,2\pi,\arccos{\left(\sqrt{\dfrac{2}{3}}\right)},\arccos{\left(-\sqrt{\dfrac{2}{3}}\right)},2\pi-\arccos{\left(\sqrt{\dfrac{2}{3}}\right)},2\pi-\arccos{\left(-\sqrt{\dfrac{2}{3}}\right)}.         \end{equation*}

Troviamo i corrispondenti punti su \partial S^+:

\begin{equation*}             \begin{split}                 &P_4=\psi^+\left(0,\arccos{\left(\dfrac{\sqrt{3}}{3}\right)}\right)=\psi^+\left(2\pi,\arccos{\left(\dfrac{\sqrt{3}}{3}\right)}\right)=\left(\sqrt{2},1,0\right),\\  &P_5=\psi^+\left(\pi,\arccos{\left(\dfrac{\sqrt{3}}{3}\right)}\right)=\left(-\sqrt{2},1,0\right),\\ &P_6=\psi^+\left(\arccos{\left(\sqrt{\dfrac{2}{3}}\right)},\arccos{\left(\dfrac{\sqrt{3}}{3}\right)}\right)=\left(\dfrac{2}{\sqrt{3}},1,\sqrt{\dfrac{2}{3}}\right),\\ &P_7=\psi^+\left(\arccos{\left(-\sqrt{\dfrac{2}{3}}\right)},\arccos{\left(\dfrac{\sqrt{3}}{3}\right)}\right)=\left(-\dfrac{2}{\sqrt{3}},1,\sqrt{\dfrac{2}{3}}\right),\\                 &P_8=\psi^+\left(2\pi-\arccos{\left(\sqrt{\dfrac{2}{3}}\right)},\arccos{\left(\dfrac{\sqrt{3}}{3}\right)}\right)=\left(\dfrac{2}{\sqrt{3}},1,-\sqrt{\dfrac{2}{3}}\right),\\ &P_9=\psi^+\left(2\pi-\arccos{\left(-\sqrt{\dfrac{2}{3}}\right)},\arccos{\left(\dfrac{\sqrt{3}}{3}\right)}\right)=\left(-\dfrac{2}{\sqrt{3}},1,-\sqrt{\dfrac{2}{3}}\right).             \end{split}         \end{equation*}

Questo conclude lo studio di S^+.

\[\quad\]

\[\quad\]

\[\quad\]

\[\quad\]

\[\quad\]

S^-) La situazione è inizialmente molto simile rispetto allo studio di S^+, infatti anche S^- è una porzione di sfera di raggio \sqrt{3}. Utilizziamo nuovamente le coordinate sferiche

\begin{equation*}         \psi(\theta,\varphi):=\left(\sqrt{3}\cos\theta\sin\varphi,\sqrt{3}\cos\varphi,\sqrt{3}\sin\theta\sin\varphi\right),     \end{equation*}

e selezioniamo accuratamente il dominio di queste coordinate per ottenere una parametrizzazione regolare di S^-. Poiché S^- è definito dal vincolo y\in\left[-\sqrt{3},-1\right], avremo

\begin{equation*} \begin{split}         \begin{cases}         \sqrt{3}\cos\varphi\leq-1\\\\         \sqrt{3}\cos\varphi\geq-\sqrt{3}         \end{cases} & \iff         \begin{cases}         \cos\varphi\leq-\dfrac{\sqrt{3}}{3}\\\\         \cos\varphi\geq-1         \end{cases} \\         & \iff  \varphi\in\bigcup_{k\in\mathbb{Z}}\left[\arccos{\left(-\dfrac{\sqrt{3}}{3}\right)}+2k\pi,-\arccos{\left(-\dfrac{\sqrt{3}}{3}\right)}+2(k+1)\pi\right]. \end{split}     \end{equation*}

Intersecando con il naturale dominio della parametrizzazione della sfera completa, per cui solitamente vale \varphi\in[0,\pi], si ha

\begin{equation*}         \varphi\in\left[\arccos{\left(-\dfrac{\sqrt{3}}{3}\right)},\pi\right].     \end{equation*}

Di nuovo, la variabile \theta non ha vincoli. Possiamo allora affermare, riprendendo le giustificazioni fornite al passaggio precedente, che una parametrizzazione regolare di S^- è la seguente

\begin{equation*}         \begin{array}{ccl}              \psi^-:[0,2\pi]\times\left[\arccos{\left(-\dfrac{\sqrt{3}}{3}\right)},\pi\right]&\rightarrow&\mathbb{R}^3  \\\\              (\theta,\varphi)&\mapsto& \left(\sqrt{3}\cos\theta\sin\varphi,\sqrt{3}\cos\varphi,\sqrt{3}\sin\theta\sin\varphi\right).         \end{array}     \end{equation*}

Questo ragionamento ci permette nuovamente di semplificare lo studio del massimo e minimo di f sulla varietà S^- attraverso il più comodo problema di determinare massimo e minimo della funzione composta f\circ\psi^- sulla regione [0,2\pi]\times\left[\arccos{\left(-\dfrac{\sqrt{3}}{3}\right)},\pi\right].

Osserviamo che, essendo l’espressione funzionale di \psi^- uguale a quella di \psi^+, si trova come prima

\begin{equation*}         (f\circ\psi^+)(\theta,\varphi)=6\cos\theta\sin\varphi\cos\varphi+3\sqrt{3}\cos\theta\sin^2\theta\sin^3\varphi.     \end{equation*}

Ciò che cambia rispetto a prima è ovviamente il dominio.

Essendo il dominio compatto e la funzione continua, il Teorema di Weierstrass garantisce l’esistenza dei punti di massimo e minimo. Come prima, definiamo R':=[0,2\pi]\times\left[\arccos{\left(-\dfrac{\sqrt{3}}{3}\right)},\pi\right] e L':=[0,2\pi]\times\left\{\arccos{\left(-\dfrac{\sqrt{3}}{3}\right)}\right\} e distinguiamo R'\setminus L' da L' che andranno studiati in maniera diversa.

\[\quad\]

\[\quad\]

\[\quad\]

Figura 41: parametrizzazione di S^-.

\[\quad\]

\[\quad\]

R'\setminus L') Lo strumento che ci è d’aiuto in queste situazioni è sempre il Teorema di Fermat: poiché f\circ\psi^- è differenziabile su R'\setminus L', per trovare i suoi punti di massimo e minimo, dovremo andare alla ricerca dei punti critici di f\circ\psi^- su R'\setminus L'. Calcoliamo dunque il gradiente di f\circ\psi^- e i punti che lo annullano.

Possiamo avvalerci di tutti i calcoli svolti in precedenza, dal momento che la funzione da studiare è esattamente la stessa:

\begin{equation*}             \begin{split}                 \nabla (f\circ\psi^+)=&\left(3\sin\theta\sin\varphi\left(-2\cos\varphi-\sqrt{3}\sin^2\theta\sin^2\varphi+2\sqrt{3}\cos^2\theta\sin^2\varphi\right),\right.\\                 &\qquad\left.3\cos\theta\left(2\cos^2\varphi-2\sin^2\varphi+3\sqrt{3}\sin^2\theta\cos\varphi\sin^2\varphi\right)\right).             \end{split}         \end{equation*}

Affrontiamo lo studio dei punti critici come abbiamo fatto in precedenza, con l’unica differenza che ora \varphi\in\left[\arccos{\left(-\dfrac{\sqrt{3}}{3}\right)},\pi\right]: supponendo \sin\theta=0, abbiamo trovato \varphi=\dfrac{\pi}{4}+k\dfrac{\pi}{2}, che solo per k=1 può appartenere a V. Abbiamo dunque trovato due punti da considerare:

\begin{equation*}             \tilde{P}_{10}=\left(0,\dfrac{3}{4}\pi\right),\qquad\tilde{P}_{11}=\left(\pi,\dfrac{3}{4}\pi\right),         \end{equation*}

a cui corrispondono i punti su S^-

\begin{equation*}             P_{10}=\psi^-\left(\tilde{P}_{11}\right)=\left(\dfrac{\sqrt{6}}{2},-\dfrac{\sqrt{6}}{2},0\right),\qquad P_{11}=\psi^-\left(\tilde{P}_{12}\right)=\left(-\dfrac{\sqrt{6}}{2},-\dfrac{\sqrt{6}}{2},0\right).         \end{equation*}

Supponiamo poi \sin\varphi=0, che può verificarsi solo per \varphi=\pi, che porta a \theta=\dfrac{\pi}{2},\dfrac{3}{2}\pi e dunque ai punti

\begin{equation*}             \tilde{P}_{12}=\left(\dfrac{\pi}{2},\pi\right),\qquad \tilde{P}_{13}=\left(\dfrac{3}{2}\pi,\pi\right),         \end{equation*}

i quali ambedue individuano il punto su S^-

\begin{equation*}             P_{12}=\psi^-\left(\tilde{P}_{12}\right)=\psi^-\left(\tilde{P}_{13}\right)=\left(0,-\sqrt{3},0\right).         \end{equation*}

Supponendo invece \cos\theta=0, cioè \theta=\dfrac{\pi}{2},\dfrac{3}{2}\pi troviamo

\begin{equation*}             \cos\varphi=\left\{\begin{array}{l}                 \sqrt{3}   \\\\                  -\dfrac{\sqrt{3}}{3} ,             \end{array}\right.         \end{equation*}

in cui solo l’ultima soluzione è accettabile e comporta \varphi=\arccos{\left(-\dfrac{\sqrt{3}}{3}\right)}, a cui corrispondono i punti

\begin{equation*}             \tilde{P}_{14}=\left(\dfrac{\pi}{2},\arccos{\left(-\dfrac{\sqrt{3}}{3}\right)}\right),\qquad \tilde{P}_{15}=\left(\dfrac{3}{2}\pi,\arccos{\left(-\dfrac{\sqrt{3}}{3}\right)}\right),         \end{equation*}

che tuttavia appartengono ad L', sui cui il Teorema di Fermat non ha valenza.

Procedendo come prima, troviamo che l’ultima possibile soluzione al sistema si verifica per

\begin{equation*}             \cos\theta_{1,2}=\dfrac{7\sqrt{3}\pm\sqrt{147+324}}{18}\notin\left[-1,-\dfrac{\sqrt{3}}{3}\right]         \end{equation*}

quindi non si hanno altre soluzioni.

L') Poiché su L' vale \cos\varphi=-\dfrac{\sqrt{3}}{3} e \sin\varphi=\sqrt{\dfrac{2}{3}}, la restrizione di f\circ\psi^- su L' vale

\begin{equation*}             \begin{split}                 (f\circ\psi^-)|_{L'}(\theta)=f\left(\sqrt{2}\cos\theta,-1,\sqrt{2}\sin\theta\right)=-2\sqrt{2}\cos\theta+2\sqrt{2}\sin^2\theta\cos\theta=-2\sqrt{2}\cos^3\theta,             \end{split}         \end{equation*}

che assume punti critici per \theta=k\dfrac{\pi}{2}, con k=0,1,2,3,4. Ad essi, corrispondono i punti su \partial S^-

\begin{equation*}             \begin{split}                 &P_{13}=\psi^-\left(0,\arccos{\left(-\dfrac{\sqrt{3}}{3}\right)}\right)=\psi^-\left(2\pi,\arccos{\left(-\dfrac{\sqrt{3}}{3}\right)}\right)=\left(\sqrt{2},-1,0\right),\\                 &P_{14}=\psi^-\left(\dfrac{\pi}{2},\arccos{\left(-\dfrac{\sqrt{3}}{3}\right)}\right)=\left(0,-1,\sqrt{2}\right),\\                 &P_{15}=\psi^-\left(\pi,\arccos{\left(-\dfrac{\sqrt{3}}{3}\right)}\right)=\left(-\sqrt{2},-1,0\right),\\                 &P_{16}=\psi^-\left(\dfrac{3}{2}\pi,\arccos{\left(-\dfrac{\sqrt{3}}{3}\right)}\right)=\left(0,-1,-\sqrt{2}\right).             \end{split}         \end{equation*}

Questo conclude lo studio della seconda varietà, S^-.

\[\quad\]

\[\quad\]

\[\quad\]

\[\quad\]

\[\quad\]

I) Notiamo che I è un iperboloide, ottenibile come superficie di rivoluzione della curva

\begin{equation*}         \gamma(t):=\left(\sqrt{1+t^2},t\right),\qquad t\in[-1,1].     \end{equation*}

Pertanto, una parametrizzazione regolare di I è la seguente:

\begin{equation*}         \begin{array}{rcl}              \Phi:[0,2\pi]\times[-1,1]&\rightarrow&\mathbb{R}^3  \\              (\theta,t)&\mapsto&\left(\sqrt{1+t^2}\cos\theta,t,\sqrt{1+t^2}\sin\theta\right).          \end{array}     \end{equation*}

Osserviamo anche che \partial I=\partial S^+\cup\partial S^-, che sono già state precedentemente studiate. Dunque sarà necessario occuparsi esclusivamente di I^\circ=\Phi\left([0,2\pi]\times(-1,1)\right).

\[\quad\]

\[\quad\]

\[\quad\]

Figura 44: parametrizzazione di I.

\[\quad\]

\[\quad\]

Servendoci del teorema di Fermat, andiamo ad indagare l’insieme dei punti critici della funzione composta f\circ\Phi per trovare i punti di massimo o minimo relativi di f su I. Per prima cosa calcoliamo f\circ\Phi:

\begin{equation*}         \begin{split}             (f\circ\Phi)(\theta,t)&=f\left(\sqrt{1+t^2}\cos\theta,t,\sqrt{1+t^2}\sin\theta\right)=2t\cos\theta\sqrt{1+t^2}+(1+t^2)^{\frac{3}{2}}\sin^2\theta\cos\theta\\             &=\sqrt{1+t^2}\cos\theta\left(2t+(1+t^2)\sin^2\theta\right).         \end{split}     \end{equation*}

Ora, calcoliamo il suo gradiente, per trovare i punti su [0,2\pi]\times(-1,1) che lo annullano:

\begin{equation*}         \begin{split}             \dfrac{\partial}{\partial\theta}\left(f\circ\Phi\right)(\theta,t)&=-\sqrt{1+t^2}\sin\theta\left(2t+(1+t^2)\sin^2\theta\right)+2\sin\theta\cos^2\theta(1+t^2)^{\frac{3}{2}}\\             &=\sqrt{1+t^2}\sin\theta\left[-2t-(1+t^2)\sin^2\theta+2(1+t^2)\cos^2\theta\right]\\             &=\sqrt{1+t^2}\sin\theta\left[-2t-3(1+t^2)\sin^2\theta+2(1+t^2)\right]\\             &=\sqrt{1+t^2}\sin\theta\left[(1+t^2)(2-3\sin^2\theta)-2t\right],         \end{split}     \end{equation*}

\begin{equation*}         \begin{split}             \dfrac{\partial}{\partial t}\left(f\circ\Phi\right)(\theta,t)&=\dfrac{t}{\sqrt{1+t^2}}\cos\theta\left(2t+(1+t^2)\sin^2\theta\right)+\left(2+2t\sin^2\theta\right)\sqrt{1+t^2}\cos\theta\\             &=\sqrt{1+t^2}\cos\theta\left[\dfrac{2t^2+(1+t^2)t\sin^2\theta}{1+t^2}+2+2t\sin^2\theta\right]\\             &=\dfrac{\sqrt{1+t^2}\cos\theta}{1+t^2}\left[2t^2+(1+t^2)t\sin^2\theta+2(1+t^2)+2t(1+t^2)\sin^2\theta\right]\\             &=\dfrac{\cos\theta}{\sqrt{1+t^2}}\left[3t(1+t^2)\sin^2\theta+4t^2+2\right],         \end{split}     \end{equation*}

da cui

\begin{equation*}         \nabla (f\circ\Phi)(\theta,t)=\left(\sqrt{1+t^2}\sin\theta\left[(1+t^2)(2-3\sin^2\theta)-2t\right],\dfrac{\cos\theta}{\sqrt{1+t^2}}\left[3t(1+t^2)\sin^2\theta+4t^2+2\right]\right).     \end{equation*}

I punti critici di f\circ\Phi sono le coppie (\theta,t) che soddisfano

\begin{equation*}         \left\{\begin{array}{l}                \sqrt{1+t^2}\sin\theta\left[(1+t^2)(2-3\sin^2\theta)-2t\right]=0\\\\                \dfrac{\cos\theta}{\sqrt{1+t^2}}\left[3t(1+t^2)\sin^2\theta+4t^2+2\right]=0.         \end{array}\right.     \end{equation*}

Consideriamo prima le possibili soluzioni banali: la prima equazione si annulla se \sin\theta=0, che inserito nella seconda fornisce

\begin{equation*}         \dfrac{\cos\theta}{\sqrt{1+t^2}}\left[4t^2+2\right]=0\Leftrightarrow\cos\theta=0,     \end{equation*}

che è chiaramente incompatibile con l’ipotesi \sin\theta=0. Allo stesso modo, la seconda equazione si annulla per \cos\theta=0, da cui \sin\theta=\pm1, dunque la prima equazione diventerebbe

\begin{equation*}         \pm\sqrt{1+t^2}(-1-t^2-2t)=0\Leftrightarrow t=-1,     \end{equation*}

valore che non possiamo accettare poiché si trova sulla frontiera del dominio considerato e non nella sua parte interna.

A questo punto, è lecito supporre \sin\theta,\cos\theta\neq0, dunque, osservando che \sqrt{1+t^2}\neq 0 il sistema precedente si riduce a

\begin{equation*}         \left\{\begin{array}{l}              (1+t^2)(2-3\sin^2\theta)-2t=0  \\               3t(1+t^2)\sin^2\theta+4t^2+2=0.         \end{array}\right.     \end{equation*}

Converrebbe moltiplicare la prima equazione per t, tuttavia è necessario supporre t\neq0, per il secondo principio delle equazioni. Prima, però, dobbiamo controllare se il caso t=0 comporta delle soluzioni: la seconda equazione diventerebbe impossibile per t=0, dunque possiamo supporre t\neq0 e moltiplicare la prima equazione del sistema per t:

\begin{equation*}         \left\{\begin{array}{l}              2t+2t^3-3t(1+t^2)\sin^2\theta-2t^2=0  \\               3t(1+t^2)\sin^2\theta+4t^2+2=0.         \end{array}\right.     \end{equation*}

Sommando nella prima riga le equazioni tra loro risulta

\begin{equation*}         \left\{\begin{array}{l}              2t^3+2t^2+2t+2=0  \\               3t(1+t^2)\sin^2\theta+4t^2+2=0.         \end{array}\right.     \end{equation*}

La prima equazione è nella sola variabile t e può essere risolta esplicitamente, infatti

\begin{equation*}         2t^3+2t^2+2t+2=2(t+1)(t^2+1)=0\Leftrightarrow t=-1,     \end{equation*}

che di nuovo non accettiamo. Questo significa che su I non vi sono punti interessanti.

Abbiamo trovato in tutto 16 candidati ad essere punti di massimo o minimo di f su A.

\[\quad\]

\[\quad\]

\[\quad\]

Figura 45: illustrazione di tutti i 16 punti, candidati ad essere punti di massimo o minimo di f su A.

\[\quad\]

\[\quad\]

Calcoliamo i loro valori secondo f e confrontiamoli.

\begin{equation*}     \begin{split}         f(P_1)&=f\left(\dfrac{\sqrt{6}}{2},\dfrac{\sqrt{6}}{2},0\right)=3,\qquad f(P_2)=f\left(-\dfrac{\sqrt{6}}{2},\dfrac{\sqrt{6}}{2},0\right)=-3,\\         f(P_3)&=f\left(0,\sqrt{3},0\right)=0,\qquad f(P_4)=f\left(\sqrt{2},1,0\right)=2\sqrt{2},\\         f(P_5)&=f\left(-\sqrt{2},1,0\right)=-2\sqrt{2},\qquad f(P_6)=f\left(\dfrac{2}{\sqrt{3}},1,\sqrt{\dfrac{2}{3}}\right)=\dfrac{16}{9}\sqrt{3},\\         f(P_7)&=f\left(-\dfrac{2}{\sqrt{3}},1,\sqrt{\dfrac{2}{3}}\right)=-\dfrac{16}{9}\sqrt{3},\qquad f(P_8)=f\left(\dfrac{2}{\sqrt{3}},1,-\sqrt{\dfrac{2}{3}}\right)=\dfrac{16}{9}\sqrt{3},\\         f(P_9)&=f\left(-\dfrac{2}{\sqrt{3}},1,-\sqrt{\dfrac{2}{3}}\right)=-\dfrac{16}{9}\sqrt{3},\qquad f(P_{10})=f\left(\dfrac{\sqrt{6}}{2},-\dfrac{\sqrt{6}}{2},0\right)=-3\\         f(P_{11})&=f\left(-\dfrac{\sqrt{6}}{2},-\dfrac{\sqrt{6}}{2},0\right)=3,\qquad f(P_{12})=f\left(0,-\sqrt{3},0\right)=0,\\         f(P_{13})&=f\left(\sqrt{2},-1,0\right)=-2\sqrt{2},\qquad f(P_{14})=f\left(0,-1,\sqrt{2}\right)=0,\\ f(P_{15})&=f\left(-\sqrt{2},-1,0\right)=2\sqrt{2},\qquad f(P_{16})=f\left(0,-1,\sqrt{2}\right)=0.     \end{split} \end{equation*}

\[\quad\]

Si evince che la soluzione dell’esercizio è la seguente: f ha massimo su A pari a \dfrac{16}{9}\sqrt{3}\cong3.1, assunto nei punti P_6=\left(\dfrac{2}{\sqrt{3}},1,\sqrt{\dfrac{2}{3}}\right) e P_8=\left(\dfrac{2}{\sqrt{3}},1,-\sqrt{\dfrac{2}{3}}\right), mentre il minimo vale -\dfrac{16}{9}\sqrt{3} assunto in P_7=\left(-\dfrac{2}{\sqrt{3}},1,\sqrt{\dfrac{2}{3}}\right) e in P_9=\left(-\dfrac{2}{\sqrt{3}},1,-\sqrt{\dfrac{2}{3}}\right).

 
 

Esercizio 5  (\bigstar\largewhitestar\largewhitestar\largewhitestar\largewhitestar). Data la funzione f:\mathbb{R}^3\rightarrow\mathbb{R}, definita da

\[f(x,y,z):=\dfrac{x^2+y^2+z^2}{2},\]

determinare, se esistono, il massimo e il minimo di f sull’insieme

\[A:=\{(x,y,z)\in\mathbb{R}^3\mid x+y+1=0, \ y+z+1=0\}.\]

Svolgimento.

Figura 48: la loro intersezione è l’insieme A, raffigurato in nero.

\[\quad\]

\[\quad\]

Osserviamo che l’insieme A è intersezione di piani non paralleli, pertanto forma una retta in \mathbb{R}^3. Ciò significa che non è un sottoinsieme limitato di \mathbb{R}^3 e quindi non possiamo applicare il teorema di Weierstrass. Tuttavia, osserviamo che f è strettamente convessa e A è un insieme chiuso e convesso di \mathbb{R}^3, quindi possiamo quantomeno affermare che f assume il minimo su A.

Osserviamo che A^\circ=\emptyset, dunque A=\partial A.

Cerchiamo una parametrizzazione \varphi di A e studiamo la funzione composta f\circ\varphi. Per trovare la parametrizzazione, risolviamo il sistema formato dalle equazioni definenti A:

\begin{equation*}     \left\{\begin{array}{l}         x+y+1=0   \\\\         y+z+1=0       \end{array}\right.\Leftrightarrow\left\{\begin{array}{l}         y+1=-x   \\\\         y+1=-z.       \end{array}\right. \end{equation*}

Confrontando le equazioni tra loro, troviamo x=z e y=-x-1. Allora, una possibile parametrizzazione di A è la seguente:

\begin{equation*}     \varphi:(-\infty,+\infty)\rightarrow\mathbb{R}^3 \end{equation*}

\begin{equation*}     \varphi(t):=(t,-t-1,t). \end{equation*}

Calcoliamo f\circ\varphi:\mathbb{R}\rightarrow\mathbb{R} e il suo punto critico (l’unicità segue dalla convessità di f e l’iniettività di \varphi).

\begin{equation*}     (f\circ\varphi)(t)=f(t,-t-1,t)=\dfrac{3t^2+2t+1}{3}. \end{equation*}

Dunque

\begin{equation*}     (f\circ\varphi)'(t)=\dfrac{6t+2}{2}=3t+1, \end{equation*}

da cui

\begin{equation*}     (f\circ\varphi)'(t)=0\Leftrightarrow t=-\dfrac{1}{3}. \end{equation*}

Abbiamo dunque trovato un solo punto critico, come annunciato, il quale non può che essere il punto minimizzante la funzione f su A. Esso corrisponde al punto

\begin{equation*}     P=\varphi\left(-\dfrac{1}{3}\right)=\left(-\dfrac{1}{3},-\dfrac{4}{3},-\dfrac{1}{3}\right). \end{equation*}

\[\quad\]

\[\quad\]

\[\quad\]

Figura 49: P è l’unico punto critico di f\circ\varphi.

\[\quad\]

\[\quad\]

Il valore di f in quel punto è

\begin{equation*}     f(P)=f\left(-\dfrac{1}{3},-\dfrac{2}{3},-\dfrac{1}{3}\right)=\dfrac{1}{3}. \end{equation*}

Osserviamo, invece, che non esiste il massimo di f su A. In particolare, proveremo che \sup_Af=+\infty e ciò è sufficiente a garantire che f non assuma su A il suo estremo superiore, ovvero che il massimo non esista.

Abbiamo già parametrizzato A tramite \varphi e calcolato (f\circ\varphi)(t)=\dfrac{3t^2+2t+1}{3}. Calcoliamo allora

\begin{equation*} \lim_{t\to\pm\infty}(f\circ\varphi)(t)=\lim_{t\to\pm\infty}\dfrac{3t^2+2t+1}{3}=+\infty. \end{equation*}

Ciò dimostra effettivamente che \sup_Af=+\infty e che dunque f non ammette massimo su A.

La soluzione dell’esercizio è la seguente: f ha minimo su A pari a \dfrac{1}{3}, assunto nel punto P=\left(-\dfrac{1}{3},-\dfrac{2}{3},-\dfrac{1}{3}\right), mentre f non ha massimo su A.

 
 

Esercizio 6  (\bigstar\bigstar\bigstar\bigstar\largewhitestar). Tra tutti i triangoli di fissato perimetro 2p, trovare quello di area massima.

Svolgimento 1.

Sia \overset{\triangle}{ABC} un triangolo generico di vertici A=(x_A,y_A), B=(x_B,y_B) e C=(x_C,y_C). Osserviamo che con una rototraslazione possiamo sempre supporre C=(0,0) e y_A=0. Infatti, consideriamo l’isometria di \mathbb{R}^2

\begin{equation*}     F:\mathbb{R}^2\rightarrow\mathbb{R}^2 \end{equation*}

\begin{equation*}     F:=R\circ T, \end{equation*}

dove T:\mathbb{R}^2\rightarrow\mathbb{R}^2 è la traslazione

\begin{equation*}     T(x,y):=(x-x_C,y-y_C), \end{equation*}

che trasla il piano \mathbb{R}^2 in modo tale che il punto C nell’origine e la rotazione R:\mathbb{R}^2\rightarrow\mathbb{R}^2 è della forma

\begin{equation*}     R(x,y):=R_{\overline{\theta}}\cdot (x,y)^t, \end{equation*}

dove R_{\overline{\theta}} è la matrice

\begin{equation*}     R_{\overline{\theta}}:=\begin{pmatrix}     \cos\overline{\theta}&-\sin\overline{\theta}\\     \sin\overline{\theta}&\cos\overline{\theta}     \end{pmatrix}, \end{equation*}

con \overline{\theta} un opportuno angolo definito dalla relazione

\begin{equation*}     \tan\overline{\theta}=\dfrac{y_C-y_A}{x_A-x_C} \end{equation*}

\[\quad\]

\[\quad\]

\[\quad\]

\[\quad\]

\[\quad\]

Si dimostra immediatamente che F(C)=(0,0), mentre

\begin{equation*}     \begin{split}         F(A)&= R_{\overline{\theta}}\cdot (x_A-x_C,y_A-y_C)^t= \\ &= (\cos\overline{\theta}(x_A-x_C)-\sin\overline{\theta}(y_A-y_C),\sin\overline{\theta}(x_A-x_C)+\cos\overline{\theta}(y_A-y_C))^t= \\         &= \cos\overline{\theta}\left(x_A-x_C+\dfrac{(y_A-y_C)^2}{x_A-x_C},y_C-y_A+y_A-y_C\right)^t= \\ &= \cos\overline{\theta}\left(x_A-x_C+\dfrac{(y_A-y_C)^2}{x_A-x_C},0\right)^t,     \end{split} \end{equation*}

in particolare, F_2(A)=0.

\[\quad\]

\[\quad\]

\[\quad\]

Figura 52: rotazione che sposta la traslazione di A sull’asse x, ottenendo la configurazione descritta.

\[\quad\]

\[\quad\]

Poiché F è un’isometria, essa lascia inalterate aree e lunghezze. Per questo motivo possiamo dunque supporre, senza perdita di generalità, che il triangolo sia della forma \overset{\triangle}{OAB}, con O=(0,0), A=(x_A,0) e B=(x_B,y_B).

Introduciamo le variabili x,y,z per esprimere le coordinate dei punti:

\begin{equation*}     \left\{\begin{array}{l}          x:=x_A  \\\\          y:=x_B\\\\          z:=y_B     \end{array}\right. \end{equation*}

\[\quad\]

\[\quad\]

\[\quad\]

Figura 53: le coordinate x, y e z corrispondono rispettivamente alla prima coordinata di A e alle coordinate di B.

\[\quad\]

\[\quad\]

A questo punto, è facile esprimere il perimetro e l’area del triangolo \overset{\triangle}{OAB} in queste coordinate:

\begin{equation*}     2p=|x|+\sqrt{y^2+z^2}+\sqrt{(x-y)^2+z^2}, \end{equation*}

e l’area, indicata con \mathcal{A}

\begin{equation*}     \mathcal{A}(x,y,z)=\dfrac{1}{2}|xz|. \end{equation*}

Supponiamo per semplicità x,z>0. Ciò è sempre possibile, utilizzando, se necessario, le riflessioni

\begin{equation*} D_x(x,y,z):=(-x,y,z),\qquad D_z(x,y,z):=(x,y,-z), \end{equation*}

essendo perimetro e area invarianti per riflessioni. Possiamo cosi esprimere il problema, inizialmente geometrico, in un problema di ottimizzazione vincolata, ovvero trovare il massimo della funzione

\[\mathcal{A}=\dfrac{1}{2}xz\]

soggetta al vincolo

\begin{equation*}     \mathcal{P}_p:=\{(x,y,z)\in\mathbb{R}^3\mid g_p(x,y,z)=0\}, \end{equation*}

dove

\begin{equation*} g_p(x,y,z):=x+\sqrt{y^2+z^2}+\sqrt{(x-y)^2+z^2}-2p. \end{equation*}

\[\quad\]

\[\quad\]

\[\quad\]

\[\quad\]

\[\quad\]

Utilizziamo a tal fine il metodo dei moltiplicatori di Lagrange, costruendo la funzione Lagrangiana \mathcal{L}(x,y,z):\mathbb{R}^4\rightarrow\mathbb{R}^4,

\begin{equation*}     \mathcal{L}(x,y,z,\lambda):=\dfrac{1}{2}xz+\lambda\left(x+\sqrt{y^2+z^2}+\sqrt{(x-y)^2+z^2}-2p\right). \end{equation*}

Dal teorema dei moltiplicatori di Lagrange sappiamo che ai punti critici della funzione \mathcal{A} vincolata all’insieme \mathcal{P}_p corrispondono i punti critici non vincolati della funzione \mathcal{L}. Ricerchiamo allora questi ultimi, cioè i punti (x,y,z,\lambda)\in\mathbb{R}^4 tali che

\begin{equation*}     \left\{\begin{array}{l}         \dfrac{\partial\mathcal{L}}{\partial x}(x,y,z,\lambda)=0  \\\\         \dfrac{\partial\mathcal{L}}{\partial y}(x,y,z,\lambda)=0  \\\\         \dfrac{\partial\mathcal{L}}{\partial z}(x,y,z,\lambda)=0  \\\\         \dfrac{\partial\mathcal{L}}{\partial \lambda}(x,y,z,\lambda)=0. \\\\     \end{array}\right. \end{equation*}

Applicato alla nostra Lagrangiana, il sistema diventa

\begin{equation*}     \left\{\begin{array}{l}         \dfrac{z}{2}+\lambda+\dfrac{\lambda(x-y)}{\sqrt{(x-y)^2+z^2}}=0   \\\\         \dfrac{\lambda y}{\sqrt{y^2+z^2}}+\dfrac{\lambda(y-x)}{\sqrt{(x-y)^2+z^2}}=0\\\\         \dfrac{x}{2}+\dfrac{\lambda z}{\sqrt{y^2+z^2}}+\dfrac{\lambda z}{\sqrt{(x-y)^2+z^2}}=0\\\\         x+\sqrt{y^2+z^2}+\sqrt{(x-y)^2+z^2}=2p\\\\     \end{array}\right.\Leftrightarrow\left\{\begin{array}{l}         \dfrac{z}{2}+\lambda+\dfrac{\lambda(x-y)}{\sqrt{(x-y)^2+z^2}}=0   \\\\         \lambda\left[\dfrac{y}{\sqrt{y^2+z^2}}+\dfrac{y-x}{\sqrt{(x-y)^2+z^2}}\right]=0\\\\         \dfrac{x}{2}+\dfrac{\lambda z}{\sqrt{y^2+z^2}}+\dfrac{\lambda z}{\sqrt{(x-y)^2+z^2}}=0\\\\         x+\sqrt{y^2+z^2}+\sqrt{(x-y)^2+z^2}=2p.\\\\     \end{array}\right. \end{equation*}

Osserviamo che la prima equazione si annulla se \lambda=0, tuttavia di conseguenza tutte le altre variabili si annullerebbero producendo un triangolo di area nulla, che non può chiaramente rappresentare il massimo dell’area possibile.

Possiamo allora supporre \lambda\neq0 e ridurre il precedente sistema a

\begin{equation*}     \left\{\begin{array}{l}         \dfrac{z}{2}+\lambda+\dfrac{\lambda(x-y)}{\sqrt{(x-y)^2+z^2}}=0   \\\\         \dfrac{y}{\sqrt{y^2+z^2}}=\dfrac{x-y}{\sqrt{(y-x)^2+z^2}}\\\\         \dfrac{x}{2}+\dfrac{\lambda z}{\sqrt{y^2+z^2}}+\dfrac{\lambda z}{\sqrt{(x-y)^2+z^2}}=0\\\\         x+\sqrt{y^2+z^2}+\sqrt{(x-y)^2+z^2}=2p.\\\\     \end{array}\right. \end{equation*}

Manipoliamo algebricamente la seconda equazione per ottenere la prima informazione preziosa:

\begin{equation*}     \begin{split}         \dfrac{y}{\sqrt{y^2+z^2}}&=\dfrac{x-y}{\sqrt{(y-x)^2+z^2}}\Leftrightarrow \dfrac{y^2}{y^2+z^2}=\dfrac{(x-y)^2}{(y-x)^2+z^2}\Leftrightarrow \\\\         \dfrac{y^2+z^2-z^2}{y^2+z^2}&=\dfrac{(x-y)^2+z^2-z^2}{(x-y)^2+z^2}\Leftrightarrow         1-\dfrac{z^2}{y^2+z^2}=1-\dfrac{z^2}{(x-y)^2+z^2}\Leftrightarrow\\\\         \dfrac{1}{y^2+z^2}&=\dfrac{1}{(x-y)^2+z^2}\Leftrightarrow y^2+z^2=(x-y)^2+z^2.     \end{split} \end{equation*}

Ma questo implica che i denominatori della seconda equazione coincidono, pertanto coincideranno anche i numeratori:

\begin{equation*}     y=x-y\Leftrightarrow y=\dfrac{x}{2}. \end{equation*}

Quest’informazione esprime geometricamente che il triangolo \overset{\triangle}{OAB} è isoscele. Dobbiamo mostrare che si tratta di un triangolo equilatero, caso più particolare di un triangolo isoscele.

Risolviamo il sistema, con le relazioni y=\dfrac{x}{2}, y^2+z^2=(x-y)^2+z^2:

\begin{equation*}     \left\{\begin{array}{l}         \dfrac{z}{2}+\lambda+\dfrac{\lambda(x-y)}{\sqrt{(x-y)^2+z^2}}=0   \\\\         x=2y\\\\         \dfrac{x}{2}+\dfrac{\lambda z}{\sqrt{y^2+z^2}}+\dfrac{\lambda z}{\sqrt{(x-y)^2+z^2}}=0\\\\         x+\sqrt{y^2+z^2}+\sqrt{(x-y)^2+z^2}=2p\\\\     \end{array}\right.\Leftrightarrow\left\{\begin{array}{l}         \dfrac{z}{2}+\lambda+\dfrac{\lambda y}{\sqrt{y^2+z^2}}=0   \\\\         x=2y\\\\         y+\dfrac{2\lambda z}{\sqrt{y^2+z^2}}=0\\\\         x+2\sqrt{y^2+z^2}=2p.\\\\     \end{array}\right. \end{equation*}

Dall’ultima equazione risulta

\begin{equation*}     \sqrt{y^2+z^2}=p-\dfrac{x}{2}=p-y, \end{equation*}

che inserita nelle altre equazioni comporta

\begin{equation*}     \left\{\begin{array}{l}         \dfrac{z}{2}+\lambda+\dfrac{\lambda y}{p-y}=0   \\\\         x=2y\\\\         y+\dfrac{2\lambda z}{p-y}=0\\\\         \sqrt{y^2+z^2}=p-y.\\\\     \end{array}\right. \end{equation*}

Esplicitiamo z nella prima equazione:

\begin{equation*}     \dfrac{z}{2}=-\lambda\left(1+\dfrac{y}{p-y}\right)=-\left(\dfrac{\lambda p}{p-y}\right), \end{equation*}

da cui

(4) \begin{equation*}     z=\dfrac{2\lambda p}{y-p}. \end{equation*}

Sostituiamo questa espressione nella terza equazione per ottenere \lambda:

\begin{equation*}     y+\dfrac{2\lambda\dfrac{2\lambda p}{y-p}}{p-y}=0\Leftrightarrow y-\dfrac{4\lambda^2 p}{(p-y)^2}=0\Leftrightarrow \lambda=\pm\dfrac{p-y}{2}\sqrt{\dfrac{y}{p}}. \end{equation*}

Sostituiamo questa espressione in (4) per trovare

\begin{equation*}     z=\mp\sqrt{\dfrac{y}{p}}p=\mp\sqrt{yp}, \end{equation*}

da cui

\begin{equation*}     z^2=yp. \end{equation*}

Sostituendo quest’espressione nell’ultima equazione, finalmente ricaviamo y:

\begin{equation*}     \sqrt{y^2+yp}=p-y\Leftrightarrow y^2+yp=p^2+y^2-2yp\Leftrightarrow 3yp=p^2\Leftrightarrow y=\dfrac{p}{3}. \end{equation*}

Possiamo immediatamente trovare le altre variabili:

\begin{equation*}     x=\dfrac{2}{3}p,\qquad z=\dfrac{\sqrt{3}}{3}p. \end{equation*}

\[\quad\]

\[\quad\]

\[\quad\]

Figura 56: in entrambe le situazioni i gradienti di f e g_p nei punti prestabiliti sono linearmente dipendenti, essendo paralleli.

\[\quad\]

\[\quad\]

Abbiamo dunque trovato il triangolo \overset{\triangle}{OAB} di coordinate

\begin{equation*}     O=(0,0),\qquad A=\left(\dfrac{2}{3}p,0\right),\qquad C=\left(\dfrac{p}{3},\dfrac{\sqrt{3}}{3}p\right). \end{equation*}

\[\quad\]

\[\quad\]

\[\quad\]

Figura 57: il triangolo \overset{\triangle}{OAB} è equilatero.

\[\quad\]

\[\quad\]

Notiamo che esse individuano un triangolo equilatero di perimetro 2p, infatti i lati hanno lunghezza uguale

\begin{equation*}     \overline{OA}=x_A=x=\dfrac{2}{3}p, \end{equation*}

\begin{equation*}     \overline{OB}=\sqrt{x_B^2+y_B^2}=\sqrt{y^2+z^2}=\dfrac{2}{3}p, \end{equation*}

\begin{equation*}     \overline{AB}=\sqrt{(x_A-x_B)^2+(y_A-y_B)^2}=\sqrt{(x-y)^2+z^2}=\dfrac{2}{3}p. \end{equation*}

e perimetro assegnato, avendo

\begin{equation*}     \overline{OA}+\overline{OB}+\overline{AB}=\dfrac{2}{3}p+\dfrac{2}{3}p+\dfrac{2}{3}p=2p. \end{equation*}


Svolgimento 2.

Questo esercizio si poteva risolvere per via del tutto algebrica, facendo uso della disuguaglianza tra media aritmetica e geometrica. Ricordiamo infatti il seguente fatto: Siano x_1,...,x_n\in\mathbb{R}^+, allora vale

\begin{equation*}             \sqrt[n]{x_1\cdot...\cdot x_n}\leq\dfrac{x_1+...+x_n}{n},         \end{equation*}

con l’uguaglianza che sussiste se e solo se x_1=...=x_n.

Consideriamo allora un triangolo di perimetro 2p e, detti x, y e z le lunghezze dei suoi lati, risulta \dfrac{x+y+z}{2}=p.

Ricordiamo, infine, la formula di Erone, che esprime l’area di un triangolo in funzione del perimetro e dei suoi lati:

\begin{equation*}         \mathcal{A}(x,y,z)=\sqrt{p(p-x)(p-y)(p-z)}.     \end{equation*}

Utilizziamo allora la disuguaglianza precedente con n=3, x_1:=p-x, x_2:=p-y e x_3:=p-z, ottenendo

\begin{equation*}         \sqrt[3]{(p-x)(p-y)(p-z)}\leq \dfrac{p-x+p-y+p-z}{3}=\dfrac{p}{3}.     \end{equation*}

D’altra parte,

\begin{equation*}         \sqrt[3]{(p-x)(p-y)(p-z)}=\sqrt[3]{\dfrac{\mathcal{A}(x,y,z)^2}{p}},     \end{equation*}

da cui

\begin{equation*}         \mathcal{A}(x,y,z)\leq \dfrac{\sqrt{3}}{9}p^2.     \end{equation*}

Osserviamo infine che l’uguaglianza e quindi il valore di area massima, si ottiene solamente se p-x=p-y=p-z, ovvero se il triangolo è equilatero.


 
 

Esercizio 7  (\bigstar\bigstar\largewhitestar\largewhitestar\largewhitestar). Data la funzione f:\mathbb{R}^3\rightarrow\mathbb{R}, definita da

\[f(x,y,z):=x-2y+z,\]

determinare, se esistono, il massimo e il minimo di f sull’insieme

\[A:=\{(x,y,z)\in\mathbb{R}^3\mid x^2+z^2\leq y\leq 1\}.\]

Svolgimento.

Figura 58: l’insieme A.

\[\quad\]

\[\quad\]

Osserviamo che f è un polinomio, pertanto è una funzione continua su \mathbb{R}^3, mentre l’insieme A è chiuso e limitato. La chiusura segue dal fatto che A è intersezione di controimmagini di insiemi chiusi di \mathbb{R} tramite funzioni continue, infatti possiamo scrivere A nel seguente modo:

\begin{equation*}     A:=\{(x,y,z)\in\mathbb{R}^3\mid x^2+z^2-y\leq 0\}\cap \{(x,y,z)\in\mathbb{R}^3\mid 0\leq y\leq 1\}, \end{equation*}

le funzioni h_1(x,y,z):=x^2+z^2-y e h_2(x,y,z)=y sono funzioni continue e gli intervalli (-\infty,0], [0,1] sottoinsiemi chiusi di \mathbb{R}, per cui si verifica A=h^{-1}_1((-\infty,0])\cap h^{-1}_2([0,1]). Per la limitatezza, osserviamo che ciascun punto (x,y,z)\in A soddisfa

\begin{equation*}     x^2+y^2+z^2\leq 1+y^2\leq 2, \end{equation*}

da cui A è contenuto in una sfera di raggio \sqrt{2}. Dunque è un chiuso e limitato di \mathbb{R}^3, compatto per il teorema di Heine-Borel. Possiamo allora applicare il teorema di Weierstrass, il quale garantisce l’esistenza del massimo e del minimo di f su A.

Distinguiamo l’insieme A nella sua parte interna e la sua frontiera: A=A^\circ\sqcup\partial A. Studiamo separatamente e in maniera differente le due componenti.

A^\circ) Essendo f differenziabile sull’insieme aperto A^\circ, il teorema di Fermat ci assicura che gli estremi relativi di f in A^\circ vanno ricercati tra i suoi eventuali punti critici in A^\circ, ovvero i punti P\in A^\circ tali che \nabla f(P)=(0,0,0) e scartare tutti i punti che non soddisfano tale condizione. Osserviamo tuttavia che f, in qualità di funzione lineare, non ammette punti critici. Pertanto, lo studio di A^\circ non comporta alcun punto da annotare.

\partial A) Possiamo descrivere \partial A come unione di due componenti, ovvero \partial A=B\cup C, con

\begin{equation*}         B:=\{(x,y,z)\in\mathbb{R}^3\mid x^2+z^2=y, x^2+z^2\leq 1\},     \end{equation*}

\begin{equation*}         C:=\{(x,y,z)\in\mathbb{R}^3\mid y=1, x^2+z^2\leq 1\}.     \end{equation*}

\[\quad\]

\[\quad\]

\[\quad\]

Figura 59: B e C, le due componenti della frontiera di A.

\[\quad\]

\[\quad\]

Al fine di risolvere l’esercizio è utile trovare una parametrizzazione regolare per ciascuna delle componenti, \psi_B e \psi_C e studiare le funzioni composte f\circ\psi_B e f\circ\psi_C.

B) È immediato osservare che B è unione di circonferenze parallele al piano xz di raggio y, dunque una sua possibile parametrizzazione è la seguente:

\begin{equation*}             \psi_B:[0,1]\times[0,2\pi]\rightarrow\mathbb{R}^3         \end{equation*}

\begin{equation*}             \psi_B(\rho,\theta):=(\rho\cos\theta,\rho^2,\rho\sin\theta).         \end{equation*}

\[\quad\]

\[\quad\]

\[\quad\]

Figura 60: la parametrizzazione di B con rappresentazione di alcune linee coordinate.

\[\quad\]

\[\quad\]

Sia R:=[0,1]\times[0,2\pi] il dominio della parametrizzazione e R^\circ=(0,1)\times(0,2\pi) la sua parte interna. \psi_B è una parametrizzazione regolare di B poiché \psi_B|_{R^\circ} è iniettiva e di classe \mathcal{C}^1 su R^\circ, \det J_{\psi_B}\neq 0 su R^\circ ed infine vale

\[\psi_B(R)=B.\]

Osserviamo che sia f sia \psi_B sono funzioni continue, dunque lo è anche la loro composizione. Infine, R è un chiuso e limitato di \mathbb{R}^2, dunque compatto. Per il teorema di Weierstrass, esisteranno il massimo ed il minimo di f\circ\psi_B su R. Per trovarli, distinguiamo nuovamente l’insieme nella sua parte interna, R^\circ e la sua frontiera, formata dai 4 lati del rettangolo.

Per quanto riguarda l’insieme aperto R^\circ, essendo f\circ\psi_B una funzione differenziabile, il teorema di Fermat afferma che condizione necessaria affinchè P\in R^\circ sia un punto di massimo o di minimo è che \nabla (f\circ\psi_B)(P)=(0,0). Calcoliamo allora il gradiente di f\circ\psi_B tramite le derivate parziali:

\begin{equation*}     (f\circ\psi_B)(\rho,\theta)=f(\rho\cos\theta,\rho^2,\rho\sin\theta)=\rho(\sin\theta+\cos\theta-2\rho), \end{equation*}

\[\begin{cases} \dfrac{\partial (f\circ\psi_B)}{\partial \rho}(\rho,\theta)=\cos\theta+\sin\theta-4\rho\\\\ \dfrac{\partial (f\circ\psi_B)}{\partial \theta}(\rho,\theta)=-\rho\sin\theta+\rho\cos\theta. \end{cases}\]

Dunque

(5) \begin{equation*}     \nabla (f\circ\psi_B)(\rho,\theta)=(0,0)\Leftrightarrow\displaystyle\left\{\begin{array}{l} \cos\theta+\sin\theta-4\rho=0\\\\ -\rho\sin\theta+\rho\cos\theta=0 \end{array} \right.\Leftrightarrow\displaystyle\left\{\begin{array}{l} \cos\theta+\sin\theta-4\rho=0\\\\ \rho(\cos\theta-\sin\theta)=0. \end{array} \right. \end{equation*}

Poiché su R^\circ, \rho>0, la seconda equazione si annulla solamente se \cos\theta=\sin\theta, cioè per \theta_1=\dfrac{\pi}{4}, \theta_2=\dfrac{5}{4}\pi. Dalla prima equazione risulta

\begin{equation*}     \rho=\pm\dfrac{\sqrt{2}}{4}, \end{equation*}

che possiamo accettare solo con segno positivo. Abbiamo trovato il punto critico di f\circ\psi_B:

\begin{equation*}     \Tilde{P}_1:=\left(\dfrac{\sqrt{2}}{4},\dfrac{\pi}{4}\right), \end{equation*}

a cui corrisponde il punto su B

\begin{equation*}     P_1=\psi_B\left(\dfrac{\sqrt{2}}{4},\dfrac{\pi}{4}\right)=\left(\dfrac{1}{4},\dfrac{1}{8},\dfrac{1}{4}\right). \end{equation*}

\[\quad\]

\[\quad\]

\[\quad\]

Figura 61: \tilde{P}_1 è l’unico punto critico di f\circ\psi_B. Il suo corrispondente punto su B è P_1=\psi_B\left(\tilde{P}_1\right).

\[\quad\]

\[\quad\]

Per quanto riguarda la frontiera \partial R, essa è composta dai 4 lati L_1,L_2,L_3 e L_4, come mostrato in figura. Notiamo, tuttavia, che non è necessario studiare f\circ\psi_B su L_2 e L_4, i lati corrispondenti ai valori di \theta=0, \theta=2\pi, poichè la funzione f\circ\psi_B è 2\pi-periodica rispetto a \theta. Infatti, avremmo potuto parametrizzare B tramite \psi_B':[0,1]\times[-\pi,\pi], con \psi_B'(\rho,\theta)=\psi_B(\rho,\theta), laddove entrambe definite. In tal modo la parte di frontiera relativa a \theta=0 non avrebbe svolto alcun ruolo nella ricerca dei massimi e minimi di f\circ\psi_B su R. Lo stesso discorso vale analogamente per \theta=2\pi.

\[\quad\]

\[\quad\]

\[\quad\]

Figura 62: studio di \partial B attraverso la sua parametrizzazione. Basterà studiare i lati L_1 e L_3.

\[\quad\]

\[\quad\]

Il lato L_1, corrispondente al settore \rho=0 viene parametrizzato da

\begin{equation*}     \begin{array}{rcl}         \alpha_1:[0,2\pi] & \rightarrow&\mathbb{R}^2 \\          t&\mapsto&(0,t)     \end{array} \end{equation*}

e

\begin{equation*}     (f\circ\psi_B)(\alpha_1(t))=f(\psi_B(0,t))=f(0,0,0)=0. \end{equation*}

Ciò significa che f\circ\psi_B è costante su L_1=\{0\}\times[0,2\pi] che risulta quindi un insieme di punti critici per f\circ\psi_B, a cui corrisponde su B il solo punto \{(0,0,0)\}=\psi_B(L_1), che denotiamo con P_2.\ Per quanto riguarda L_3=\{1\}\times[0,2\pi], parametrizzato da

\begin{equation*}     \begin{array}{rcl}         \alpha_3:[0,2\pi] & \rightarrow&\mathbb{R}^2 \\          t&\mapsto&(1,t).     \end{array} \end{equation*}

calcoliamo (f\circ\psi_B)(\alpha_3(t)) e i suoi punti critici.

\begin{equation*}     (f\circ\psi_B)(\alpha_3(t))=f(\cos t,1,\sin t)=\cos t+\sin t-2, \end{equation*}

\begin{equation*}     ((f\circ\psi_B)\circ\alpha_3)'(t)=-\sin t+\cos t. \end{equation*}

Dunque i punti critici di ((f\circ\psi_B)\circ\alpha_3), ovvero i punti che annullano la sua derivata prima, sono i valori di t\in[0,2\pi] per cui vale

\begin{equation*}     \cos t=\sin t, \end{equation*}

cioè per t=\dfrac{\pi}{4} e t=\dfrac{5}{4}\pi. Troviamo dunque i punti

\[\Tilde{P}_3=\left(1,\dfrac{\pi}{4}\right),\qquad \Tilde{P}_4=\left(1,\dfrac{5}{4}\pi\right),\]

a cui corrispondono su B i punti

\begin{equation*}     P_3=\psi_B\left(1,\dfrac{\pi}{4}\right)=\left(\dfrac{\sqrt{2}}{2},1,\dfrac{\sqrt{2}}{2}\right),\qquad P_4=\psi_B\left(1,\dfrac{5}{4}\pi\right)=\left(-\dfrac{\sqrt{2}}{2},1,-\dfrac{\sqrt{2}}{2}\right). \end{equation*}

\[\quad\]

\[\quad\]

\[\quad\]

Figura 63: P_3 e P_4 corrispondono a punti critici su L_3 e il punto P_2 è immagine del lato L_1.

\[\quad\]

\[\quad\]

Questo conclude lo studio di f su B che ha portato alla nostra attenzione 4 candidati ad essere punti di massimo o di minimo di f su A: i punti P_1,P_2,P_3 e P_4.

C) Osserviamo immediatamente che C è un cerchio di raggio 1 a quota y=1. Pertanto esso è parametrizzabile da

\begin{equation*}     \psi_C:R \rightarrow\mathbb{R}^3 \end{equation*}

\begin{equation*}     \psi_C(\rho,\theta):=(\rho\cos\theta,1,\rho\sin\theta). \end{equation*}

\[\quad\]

\[\quad\]

\[\quad\]

Figura 64: la parametrizzazione di C, con alcune linee coordinate.

\[\quad\]

\[\quad\]

Calcoliamo f\circ\psi_C:

\begin{equation*}     (f\circ\psi_C)(\rho,\theta)=f(\rho\cos\theta,1,\rho\sin\theta)=\rho\cos\theta+\rho\sin\theta-2. \end{equation*}

Analogamente a quanto detto in precedenza, anche per f\circ\psi_C il teorema di Weierstrass garantisce l’esistenza del massimo e del minimo su R. Studiamo inizialmente il comportamento di f\circ\psi_C sull’insieme aperto R^\circ tramite la ricerca dei punti critici, per poi studiare f\circ\psi_C su \partial R.

Calcoliamo le derivate parziali di f\circ\psi_C:

\[\begin{cases} \dfrac{\partial (f\circ\psi_C)}{\partial \rho}(\rho,\theta)=\cos\theta+\sin\theta\\\\ \dfrac{\partial (f\circ\psi_C)}{\partial \theta}(\rho,\theta)=-\rho\sin\theta+\rho\cos\theta. \end{cases}\]

Dunque

(6) \begin{equation*}     \nabla (f\circ\psi_C)(\rho,\theta)=(0,0)\Leftrightarrow\displaystyle\left\{\begin{array}{l} \cos\theta+\sin\theta=0\\\\ -\rho\sin\theta+\rho\cos\theta=0, \end{array} \right. \end{equation*}

ma questo sistema non ha soluzioni: ciò significa che non ci sono punti critici di f\circ\psi_C su R^\circ. Per quanto riguarda \partial R, poichè anche f\circ\psi_C è 2\pi-periodica rispetto a \theta, lo stesso discorso fatto in precedenza ci autorizza ad ignorare i lati corrispondenti ai valori \theta=0 e \theta=2\pi. I lati rimasti sono M_1=\{0\}\times[0,2\pi] e M_3=\{1\}\times[0,2\pi]. Osserviamo tuttavia che \psi_C(M_1)=\{(0,1,0)\}, dunque annotiamo per semplicità P_5=(0,1,0) tra i possibili candidati.

Infine, per quanto riguarda M_3, osserviamo che f\circ\psi_C|_{M_3}=f\circ\psi_B|_{L_3}, che abbiamo ampiamente studiato in precedenza.

Questo conclude lo studio di \partial A=B\cup C.

\[\quad\]

\[\quad\]

\[\quad\]

Figura 65: studio della frontiera di C. Anche in questo caso, i lati L_2 e L_4 non svolgono alcun ruolo.

\[\quad\]

\[\quad\]

I possibili candidati ad essere punti di massimo o di minimo di f su A sono i seguenti

\begin{equation*} \begin{split}      &P_1=\left(\dfrac{1}{4},\dfrac{1}{8},\dfrac{1}{4}\right),\qquad P_2=(0,0,0),\qquad P_3=\left(\dfrac{\sqrt{2}}{2},1,\dfrac{\sqrt{2}}{2}\right),\\      & P_4=\left(-\dfrac{\sqrt{2}}{2},1,-\dfrac{\sqrt{2}}{2}\right),\qquad P_5=(0,1,0), \end{split} \end{equation*}

\[\quad\]

\[\quad\]

\[\quad\]

Figura 66: tutti i candidati ad essere punti di massimo o minimo di f su A.

\[\quad\]

\[\quad\]

i cui valori rispetto a f sono

\begin{equation*} \begin{split}     &f(P_1)=\dfrac{1}{4},\qquad f(P_2)=0,\qquad f(P_3)=\sqrt{2}-2,\\     &f(P_4)=-\sqrt{2}-2,\qquad f(P_5)=-2. \end{split} \end{equation*}

\[\quad\]

La soluzione dell’esercizio è la seguente: f ha massimo su A pari a \dfrac{1}{4}, assunto nel punto P_1=\left(\dfrac{1}{4},\dfrac{1}{8},\dfrac{1}{4}\right) e valore minimo -\sqrt{2}-2, assunto in P_4=\left(-\dfrac{\sqrt{2}}{2},1,-\dfrac{\sqrt{2}}{2}\right).

 
 

Esercizio 8  (\bigstar\bigstar\largewhitestar\largewhitestar\largewhitestar). Data la funzione f:\mathbb{R}^3\rightarrow\mathbb{R}, definita da

\[f(x,y,z):=x^3+y^3+z^3,\]

determinare, se esistono, il massimo e il minimo di f sull’insieme

\[A:=\{(x,y,z)\in\mathbb{R}^3\mid x^2+y^2-z^2=1, \ x+y=1, \ -2\leq z\leq 2\}.\]

Svolgimento.

Figura 69: l’insieme A, in nero, ottenuto come intersezione tra il cilindro e il piano, con vincolo -2\leq z\leq 2.

\[\quad\]

\[\quad\]

Osserviamo che f è un polinomio, pertanto è una funzione continua su \mathbb{R}^3, mentre l’insieme A è chiuso e limitato. La chiusura segue dal fatto che A è intersezione di controimmagini di insiemi chiusi di \mathbb{R} tramite funzioni continue, la limitatezza dalle equazioni che definiscono A. Infatti, per ipotesi z\in[-2,2], mentre dalla prima equazione risulta

\begin{equation*}     x^2+y^2=1+z^2\leq5, \end{equation*}

da cui possiamo affermare che l’insieme A è contenuto in una sfera di raggio r=\sqrt{5+5+4}=\sqrt{14}, in particolare A è un insieme limitato. Abbiamo dunque trovato che A è un chiuso e limitato di \mathbb{R}^3, compatto per il teorema di Heine-Borel. Possiamo allora applicare il teorema di Weierstrass, il quale garantisce l’esistenza del massimo e del minimo di f su A.

Osserviamo che A^\circ=\emptyset, ovvero A è un insieme chiuso che coincide con la sua frontiera. È utile, tuttavia, separare A tramite le due componenti

\begin{equation*}     A_1:=\{(x,y,z)\in\mathbb{R}^3\mid x^2+y^2-z^2=1, \ x+y=1, \ -2< z< 2\}, \end{equation*}

\begin{equation*}     A_2:=\{(x,y,z)\in\mathbb{R}^3\mid x^2+y^2-z^2=1, \ x+y=1, \ z=\pm2\}, \end{equation*}

\[\quad\]

\[\quad\]

\[\quad\]

Figura 70: raffigurati i due insiemi che formano A: A_1 in nero e A_2 composto dai 4 punti rossi.

\[\quad\]

\[\quad\]

da cui risulta

\begin{equation*}     A=A_1\cup A_2. \end{equation*}

Ricerchiamo i punti di massimo e minimo di f su A_1 attraverso il metodo dei moltiplicatori di Lagrange, mentre indaghiamo A_2 in maniera diretta, essendo unione di punti isolati, come vedremo. Osserviamo che il vincolo A_1 si può esprimere come intersezione di luogo di zeri di due funzioni: siano g,h:\mathbb{R}^3\rightarrow\mathbb{R} definite da

\begin{equation*}     g(x,y,z):=x^2+y^2-z^2-1, \end{equation*}

\begin{equation*}     h(x,y,z):=x+y-1. \end{equation*}

Detti

\begin{equation*}     B:=\{(x,y,z)\in\mathbb{R}^2\times(-2,2)\mid g(x,y,z)=0\} \end{equation*}

e

\begin{equation*}     C:=\{(x,y,z)\in\mathbb{R}^2\times(-2,2)\mid h(x,y,z)=0\}, \end{equation*}

\[\quad\]

\[\quad\]

\[\quad\]

Figura 71: si applica il metodo dei moltiplicatori di Lagrange ai vincoli B e C, la cui intersezione è il vincolo effettivo.

\[\quad\]

\[\quad\]

vale

\begin{equation*}     A_1=B\cap C. \end{equation*}

Il metodo dei moltiplicatori di Lagrange ci induce a studiare la funzione Lagrangiana

\[\mathcal{L}(x,y,z,\lambda,\mu):=f(x,y,z)-\lambda g(x,y,z)-\mu h(x,y,z)\]

e ricercare i suoi punti critici non vincolati. Infatti, per il teorema dei moltiplicatori di Lagrange, ad essi corrisponderanno i punti critici vincolati di f su A_1. Dovremo dunque risolvere il sistema

\begin{equation*}     \left\{\begin{array}{l}         \dfrac{\partial\mathcal{L}}{\partial x}=0  \\\\         \dfrac{\partial\mathcal{L}}{\partial y}=0  \\\\         \dfrac{\partial\mathcal{L}}{\partial z}=0  \\\\         \dfrac{\partial\mathcal{L}}{\partial \lambda}=0 \\\\         \dfrac{\partial\mathcal{L}}{\partial \mu}=0  \\\\     \end{array}\right.\Leftrightarrow\left\{\begin{array}{l}         \dfrac{\partial f}{\partial x}-\lambda\dfrac{\partial g}{\partial x}-\mu\dfrac{\partial h}{\partial x}=0  \\\\         \dfrac{\partial f}{\partial y}-\lambda\dfrac{\partial g}{\partial y}-\mu\dfrac{\partial h}{\partial y}=0  \\\\         \dfrac{\partial f}{\partial z}-\lambda\dfrac{\partial g}{\partial z}-\mu\dfrac{\partial h}{\partial z}=0  \\\\         g=0 \\\\         h=0.\\\\     \end{array}\right.\Leftrightarrow\left\{\begin{array}{l}          \nabla f=\lambda\nabla g+\mu\nabla h  \\\\\\          g=0 \\\\         h=0.\\\\     \end{array}\right. \end{equation*}

Nel nostro caso

\begin{equation*}     \mathcal{L}(x,y,z,\lambda,\mu)=x^3+y^3+z^3-\lambda(x^2+y^2-z^2-1)-\mu(x+y-1), \end{equation*}

i cui punti critici devono soddisfare il sistema

\begin{equation*}     \left\{\begin{array}{l}         3x^2+2\lambda x+\mu=0  \\\\         3y^2+2\lambda y+\mu=0  \\\\         3z^2-2\lambda z=0  \\\\         x^2+y^2-z^2=1 \\\\         x+y=1  \\\\     \end{array}\right.\Leftrightarrow\left\{\begin{array}{l}         3x^2+2\lambda x+\mu=0  \\\\         3y^2+2\lambda y+\mu=0  \\\\         (3z-2\lambda) z=0  \\\\         x^2+y^2-z^2=1 \\\\         x+y=1.  \\\\     \end{array}\right. \end{equation*}

Sostituiamo nella prima riga la differenza tra le prime due equazioni e utilizziamo il prodotto notevole x^2-y^2=(x+y)(x-y):

\begin{equation*}     \left\{\begin{array}{l}         3(x^2-y^2)+2\lambda(x-y)=0  \\\\         3y^2+2\lambda y+\mu=0  \\\\         (3z-2\lambda) z=0  \\\\         x^2+y^2-z^2=1 \\\\         x+y=1  \\\\     \end{array}\right.\Leftrightarrow\left\{\begin{array}{l}         (x-y)\left[3(x+y)+2\lambda\right]=0  \\\\         3y^2+2\lambda y+\mu=0  \\\\         (3z-2\lambda) z=0  \\\\         x^2+y^2-z^2=1 \\\\         x+y=1  \\\\     \end{array}\right.\Leftrightarrow\left\{\begin{array}{l}         (x-y)\left(3+2\lambda\right)=0  \\\\         3y^2+2\lambda y+\mu=0  \\\\         (3z-2\lambda) z=0  \\\\         x^2+y^2-z^2=1 \\\\         x+y=1,  \\\\     \end{array}\right. \end{equation*}

dove abbiamo utilizzato l’equazione x+y=1 per semplificare la prima equazione. Osserviamo che la terza equazione si annulla solamente se z=0 o \lambda=\dfrac{3}{2}z. Supponiamo prima z=0, da cui il sistema risulta

\begin{equation*}     \left\{\begin{array}{l}         (x-y)\left(3+2\lambda\right)=0  \\\\         3y^2+2\lambda y+\mu=0  \\\\         z=0  \\\\         x^2+y^2=1 \\\\         x+y=1  \\\\     \end{array}\right.\implies\left\{\begin{array}{l}     z=0\\\\         x^2+y^2=1 \\\\         x+y=1  \\\\     \end{array}\right.\Leftrightarrow\left\{\begin{array}{l}     z=0\\\\         x=1 \\\\         y=0  \\\\     \end{array}\right.\lor\left\{\begin{array}{l}     z=0\\\\         x=0 \\\\         y=1  \\\\     \end{array}\right. \end{equation*}

Abbiamo allora trovato i primi due punti interessanti: P_5=(1,0,0) e P_6=(0,1,0).

Supponendo, invece, \lambda=\dfrac{3}{2}z, il sistema diventa

\begin{equation*}     \left\{\begin{array}{l}         (x-y)\left(3+3z\right)=0  \\\\         3y^2+3z y+\mu=0  \\\\         \lambda=\dfrac{3}{2}z  \\\\         x^2+y^2-z^2=1 \\\\         x+y=1.  \\\\     \end{array}\right. \end{equation*}

La prima equazione si annulla solo se x=y o z=-1. Se fosse x=y, l’ultima equazione implicherebbe x,y=\dfrac{1}{2}, ma dalla quarta equazione avremmo

\begin{equation*}     z^2=x^2+y^2-1=\dfrac{1}{4}+\dfrac{1}{4}-1=-\dfrac{1}{2}, \end{equation*}

che è chiaramente impossibile. Se, invece, z=-1, focalizziamoci ancora sulle ultime due equazioni del sistema che si riducono a

\begin{equation*}     \left\{\begin{array}{l}         x^2+y^2=2 \\\\         x+y=1.  \\\\     \end{array}\right. \end{equation*}

Ricavando y=1-x, otteniamo l’equazione in funzione della sola variabile x

\begin{equation*}     2x^2+1-2x=2\Leftrightarrow2x^2-2x-1=0\Leftrightarrow x_{1,2}=\dfrac{1\pm\sqrt{1+2}}{2}=\dfrac{1\pm\sqrt{3}}{2}, \end{equation*}

da cui

\begin{equation*}     y_{1,2}=1-\dfrac{1\pm\sqrt{3}}{2}=\dfrac{1\mp\sqrt{3}}{2}. \end{equation*}

Abbiamo quindi trovato altri 2 punti da considerare:

\begin{equation*}     P_7=\left(\dfrac{1+\sqrt{3}}{2},\dfrac{1-\sqrt{3}}{2},-1\right),\qquad P_8=\left(\dfrac{1-\sqrt{3}}{2},\dfrac{1+\sqrt{3}}{2},-1\right). \end{equation*}

Questo conclude lo studio di A_1.

Occupiamoci ora di A_2 e osserviamo che è composto da 4 punti isolati. Infatti, dalle equazioni che definiscono A_2, troviamo

\begin{equation*}     \left\{\begin{array}{l}          x^2+y^2-z^2=1  \\\\          x+y=1\\\\           z=\pm2     \end{array}     \right.\Leftrightarrow\left\{\begin{array}{l}          x^2+y^2=5  \\\\          x+y=1\\\\           z=\pm2.     \end{array}     \right. \end{equation*}

Sostituiamo y=1-x nella prima equazione per trovare x:

\begin{equation*}     2x^2-2x+1=5\Leftrightarrow x^2-x-2=0\Leftrightarrow(x-2)(x+1)=0\Leftrightarrow x=2\lor x=-1, \end{equation*}

da cui si trova

\begin{equation*}     y=-1\lor y=2. \end{equation*}

Pertanto,

\begin{equation*}     A_2=P_1\cup P_2\cup P_3\cup P_4, \end{equation*}

dove

\begin{equation*}     P_1=(2,-1,2),\qquad P_2=(-1,2,2),\qquad P_3=(2,-1,-2),\qquad P_4=(-1,2,-2). \end{equation*}

Abbiamo trovato otto candidati ad essere punti di massimo e di minimo di f su A.

\[\quad\]

\[\quad\]

\[\quad\]

Figura 72: tutti i candidati ad essere punti di massimo o minimo di f su A.

\[\quad\]

\[\quad\]

Calcoliamo i loro valori tramite f e confrontiamoli:

\begin{equation*}     \begin{split}      f(P_1)=f(2,-1,2)=15,&\qquad f(P_2)=f(-1,2,2)=15,\\         f(P_3)=f(2,-1,-2)=-1,&\qquad f(P_4)=f(-1,2,-2)=-1,\\       f(P_5)=f\left(1,0,0\right)=1,&\qquad f(P_6)=f\left(0,1,0\right)=1,\\         f(P_7)=f\left(\dfrac{1+\sqrt{3}}{2},\dfrac{1-\sqrt{3}}{2},-1\right)=\dfrac{3}{2},&\qquad f(P_8)=f\left(\dfrac{1-\sqrt{3}}{2},\dfrac{1+\sqrt{3}}{2},-1\right)=\dfrac{3}{2}.\\     \end{split} \end{equation*}

La soluzione dell’esercizio è la seguente: f ha massimo su A pari a 15, assunto nei punti P_1=(2,-1,2) e P_2=(-1,2,2) valore minimo -1, assunto nei punti P_3=(2,-1,-2) e P_4=(-1,2,-2).

 
 

Esercizio 9  (\bigstar\bigstar\largewhitestar\largewhitestar\largewhitestar). Data la funzione f:\mathbb{R}^3\rightarrow\mathbb{R}, definita da

\[f(x,y,z):=z-x,\]

determinare, se esistono, il massimo e il minimo di f sull’insieme

\[A:=\{(x,y,z)\in\mathbb{R}^3\mid x^2+y^2+z=1, \ x^2+z^2=3\}.\]

Svolgimento.

Figura 73: tratteggiato, in nero, l’insieme A, ottenuto come intersezione tra un cilindro e un paraboloide.

\[\quad\]

\[\quad\]

Osserviamo che f è un polinomio, pertanto è una funzione continua su \mathbb{R}^3, mentre l’insieme A è chiuso e limitato. La chiusura segue dal fatto che A è intersezione di controimmagini di insiemi chiusi di \mathbb{R} tramite funzioni continue, la limitatezza dalle equazioni che definiscono A. Infatti, dalla seconda condizione possiamo immediatamente affermare che x,z\in[-3,3], da cui x^2\in[0,9], pertanto dalla prima equazione possiamo ottenere una stima di y:

\begin{equation*}     y^2=1-x^2-z\in[-11,4], \end{equation*}

da cui y\in[-2,2]. Abbiamo trovato che ciascun punto (x,y,z) in A dev’essere contenuto in un parallelepipedo di centro l’origine e lati 6,4 e 6, a sua volta contenuto in una sfera di raggio r=\sqrt{3^2+2^2+3^2}=\sqrt{22}. Ciò dimostra la limitatezza di A. Abbiamo dunque trovato che A è un chiuso e limitato di \mathbb{R}^3, compatto per il teorema di Heine-Borel. Possiamo allora applicare il teorema di Weierstrass, il quale garantisce l’esistenza del massimo e del minimo di f su A.

Osserviamo che A^\circ=\emptyset, ovvero A è un insieme chiuso che coincide con la sua frontiera. Ricerchiamo i punti di massimo e minimo di f su A attraverso il metodo dei moltiplicatori di Lagrange. Osserviamo che il vincolo A si può esprimere come intersezione di luogo di zeri di due funzioni: siano g,h:\mathbb{R}^3\rightarrow\mathbb{R} definite da

\begin{equation*}     g(x,y,z):=x^2+y^2+z-1, \end{equation*}

\begin{equation*}     h(x,y,z):=x^2+z^2-3. \end{equation*}

Detti

\begin{equation*}     \Gamma:=\{(x,y,z)\in\mathbb{R}^3\mid g(x,y,z)=0\} \end{equation*}

e

\begin{equation*}     \Sigma:=\{(x,y,z)\in\mathbb{R}^3\mid h(x,y,z)=0\}, \end{equation*}

\[\quad\]

\[\quad\]

\[\quad\]

\[\quad\]

\[\quad\]

vale

\begin{equation*}     A=\Gamma\cap\Sigma. \end{equation*}

Il teorema dei moltiplicatori di Lagrange afferma che in corrispondenza dei punti di massimo o minimo di f su A, che non siano punti critici dei vincoli g ed h, i gradienti delle funzioni f, g e h risultano complanari. Ciò significa che, se x_0\in A è punto di massimo o minimo di f su A e tale che \nabla g(x_0),\nabla h(x_0)\neq 0, esistono \lambda,\mu\in\mathbb{R} tali che

\begin{equation*}     \nabla f(x_0)=\lambda\nabla g(x_0)+\mu\nabla h(x_0). \end{equation*}

È conveniente, al fine di utilizzare questo metodo risolutivo, costruire direttamente la funzione Lagrangiana

\[\mathcal{L}(x,y,z,\lambda,\mu):=f(x,y,z)-\lambda g(x,y,z)-\mu h(x,y,z)\]

e ricercare i suoi punti critici non vincolati. Ad essi corrisponderanno i punti critici vincolati di f su A. Dovremo dunque risolvere il sistema

\begin{equation*}     \left\{\begin{array}{l}         \dfrac{\partial\mathcal{L}}{\partial x}=0  \\\\         \dfrac{\partial\mathcal{L}}{\partial y}=0  \\\\         \dfrac{\partial\mathcal{L}}{\partial z}=0  \\\\         \dfrac{\partial\mathcal{L}}{\partial \lambda}=0 \\\\         \dfrac{\partial\mathcal{L}}{\partial \mu}=0  \\\\     \end{array}\right.\Leftrightarrow\left\{\begin{array}{l}         \dfrac{\partial f}{\partial x}-\lambda\dfrac{\partial g}{\partial x}-\mu\dfrac{\partial h}{\partial x}=0  \\\\         \dfrac{\partial f}{\partial y}-\lambda\dfrac{\partial g}{\partial y}-\mu\dfrac{\partial h}{\partial y}=0  \\\\         \dfrac{\partial f}{\partial z}-\lambda\dfrac{\partial g}{\partial z}-\mu\dfrac{\partial h}{\partial z}=0  \\\\         g=0 \\\\         h=0.\\\\     \end{array}\right.\Leftrightarrow\left\{\begin{array}{l}          \nabla f=\lambda\nabla g+\mu\nabla h  \\\\\\          g=0 \\\\         h=0.\\\\     \end{array}\right. \end{equation*}

Nel nostro caso

\begin{equation*}     \mathcal{L}(x,y,z,\lambda,\mu)=z-x-\lambda(x^2+y^2+z-1)-\mu(x^2+z^2-3), \end{equation*}

i cui punti critici devono soddisfare il sistema

\begin{equation*}     \left\{\begin{array}{l}         -1+2\lambda x+2\mu x=0  \\\\         2\lambda y=0  \\\\         1+\lambda+2\mu z=0  \\\\         x^2+y^2+z=1 \\\\         x^2+z^2=3  \\\\     \end{array}\right.\Leftrightarrow\left\{\begin{array}{l}         2\lambda x+2\mu x=1  \\\\         \lambda y=0  \\\\         \lambda+2\mu z=-1  \\\\         x^2+y^2+z=1 \\\\         x^2+z^2=3  \\\\     \end{array}\right. \end{equation*}

Osserviamo che la seconda equazione si annulla solamente se \lambda=0 o y=0. Supponiamo inizialmente \lambda=0, allora il sistema diventa

\begin{equation*}     \left\{\begin{array}{l}         2\mu x=1  \\\\         \lambda=0  \\\\         2\mu z=-1  \\\\         x^2+y^2+z=1 \\\\         x^2+z^2=3.  \\\\     \end{array}\right. \end{equation*}

Confrontando tra loro la prima e la terza equazione, dovrà necessariamente risultare x=-z. Inserendo quest’informazione nell’ultima equazione del sistema, troviamo

\begin{equation*}     2x^2=3\implies x=\pm\sqrt{\dfrac{3}{2}}. \end{equation*}

Osserviamo che la soluzione x=-\sqrt{\dfrac{3}{2}} non è accettabile, poiché risulterebbe z=\sqrt{\dfrac{3}{2}}>1, ma dalla quarta equazione sappiamo sicuramente che z=1-x^2-y^2\leq 1. Risolviamo il sistema per x=\sqrt{\dfrac{3}{2}} e z=-\sqrt{\dfrac{3}{2}}, trovando dalla quarta equazione

\begin{equation*}     y^2=1-\dfrac{3}{2}+\sqrt{\dfrac{3}{2}}\implies y=\pm\sqrt{\sqrt{\dfrac{3}{2}}-\dfrac{1}{2}}. \end{equation*}

Per completezza, troviamo il moltiplicatore di Lagrange \mu:

\begin{equation*}     \mu=\dfrac{1}{2\sqrt{\dfrac{3}{2}}}=\dfrac{1}{\sqrt{6}}. \end{equation*}

Abbiamo trovato i primi punti critici della Lagrangiana

\[\left(\sqrt{\dfrac{3}{2}},\sqrt{\sqrt{\dfrac{3}{2}}-\dfrac{1}{2}},-\sqrt{\dfrac{3}{2}},0,\dfrac{1}{\sqrt{6}}\right),\qquad\left(\sqrt{\dfrac{3}{2}},-\sqrt{\sqrt{\dfrac{3}{2}}-\dfrac{1}{2}},-\sqrt{\dfrac{3}{2}},0,\dfrac{1}{\sqrt{6}}\right),\]

a cui corrispondono i punti su A

\begin{equation*}     P_1=\left(\sqrt{\dfrac{3}{2}},\sqrt{\sqrt{\dfrac{3}{2}}-\dfrac{1}{2}},-\sqrt{\dfrac{3}{2}}\right),\qquad P_2=\left(\sqrt{\dfrac{3}{2}},-\sqrt{\sqrt{\dfrac{3}{2}}-\dfrac{1}{2}},-\sqrt{\dfrac{3}{2}}\right). \end{equation*}

Supponendo, invece y=0, possiamo restringere la nostra attenzione alle ultime due equazioni del sistema per trovare le altre due variabili:

\begin{equation*}     \left\{\begin{array}{l}         x^2+z=1 \\\\         x^2+z^2=3  \\     \end{array}\right. \end{equation*}

per confronto tra le due equazioni troviamo

\begin{equation*}     x^2=1-z=3-z^2, \end{equation*}

ovvero

\begin{equation*}     z^2-z-2=0\Leftrightarrow(z-2)(z+1)=0\Leftrightarrow\left\{\begin{array}{l}          z=2  \\\\          z=-1      \end{array}\right. \end{equation*}

Il valore z=2 non comporta alcuna soluzione reale per x^2=1-z, per cui verrà eslcuso, mentre per z=-1 otteniamo x=\pm\sqrt{2}, ovvero i punti

\begin{equation*}     P_3=\left(\sqrt{2},0,-1\right),\qquad P_4=\left(-\sqrt{2},0,-1\right), \end{equation*}

corrispondenti ai punti critici della Lagrangiana

\begin{equation*}     \left(\sqrt{2},0,-1,\dfrac{\sqrt{2}-2}{6},\dfrac{4+\sqrt{2}}{12}\right),\qquad \left(-\sqrt{2},0,-1,\dfrac{-\sqrt{2}-2}{6},\dfrac{4-\sqrt{2}}{12}\right). \end{equation*}

Abbiamo trovato quattro candidati ad essere punti di massimo e di minimo di f su A.

\[\quad\]

\[\quad\]

\[\quad\]

Figura 76: tutti i punti candidati ad essere punti di massimo o minimo di f su A.

\[\quad\]

\[\quad\]

Mostriamo che, come prevede il teorema dei moltiplicatori di Lagrange, i gradienti di f, g e h sono complanari nei punti precedentemente trovati:

\[\quad\]

\[\quad\]

\[\quad\]

Figura 77: vi è una differenza tra P_1 e P_4: in P_1 i gradienti di f e h sono paralleli, mentre in P_4 sono complanari assieme al gradiente di g. Ciò significa che P_1 sarebbe da considerare anche nella ricerca di massimi e minimi di f sulla superficie \Sigma, mentre P_4 solo in relazione al vincolo più restrittivo \Sigma\cap\Gamma.

\[\quad\]

\[\quad\]

Calcoliamo i loro valori tramite f e confrontiamoli:

\begin{equation*}     \begin{split}     f(P_1)=f\left(\sqrt{\dfrac{3}{2}},\sqrt{\sqrt{\dfrac{3}{2}}-\dfrac{1}{2}},-\sqrt{\dfrac{3}{2}}\right)=-\sqrt{6},&\qquad f(P_2)=f\left(\sqrt{\dfrac{3}{2}},-\sqrt{\sqrt{\dfrac{3}{2}}-\dfrac{1}{2}},-\sqrt{\dfrac{3}{2}}\right)=-\sqrt{6}\\         f(P_3)=f\left(\sqrt{2},0,-1\right)=-1-\sqrt{2},&\qquad f(P_4)=f\left(-\sqrt{2},0,-1\right)=\sqrt{2}-1.     \end{split} \end{equation*}

\[\quad\]

La soluzione dell’esercizio è la seguente: f ha massimo su A pari a \sqrt{2}-1, assunto nel punto P_4=\left(-\sqrt{2},0,-1\right) e valore minimo -\sqrt{6}, assunto nei punti P_1=\left(\sqrt{\dfrac{3}{2}},\sqrt{\sqrt{\dfrac{3}{2}}-\dfrac{1}{2}},-\sqrt{\dfrac{3}{2}}\right) e P_2=\left(\sqrt{\dfrac{3}{2}},-\sqrt{\sqrt{\dfrac{3}{2}}-\dfrac{1}{2}},-\sqrt{\dfrac{3}{2}}\right).

 
 

Tutta la teoria di analisi matematica

Leggi...

  1. Teoria Insiemi
  2. Il metodo della diagonale di Cantor
  3. Logica elementare
  4. Densità dei numeri razionali nei numeri reali
  5. Insiemi Numerici \left(\mathbb{N},\, \mathbb{Z},\, \mathbb{Q}\right)
  6. Il principio di induzione
  7. Gli assiomi di Peano
  8. L’insieme dei numeri reali: costruzione e applicazioni
  9. Concetti Fondamentali della Retta Reale: Sintesi Teorica
  10. Costruzioni alternative di \mathbb{R}
  11. Binomio di Newton
  12. Spazi metrici, un’introduzione
  13. Disuguaglianza di Bernoulli
  14. Disuguaglianza triangolare
  15. Teoria sulle funzioni
  16. Funzioni elementari: algebriche, esponenziali e logaritmiche
  17. Funzioni elementari: trigonometriche e iperboliche
  18. Funzioni goniometriche: la guida essenziale
  19. Teorema di Bolzano-Weierstrass per le successioni
  20. Criterio del rapporto per le successioni
  21. Definizione e proprietà del numero di Nepero
  22. Limite di una successione monotona
  23. Successioni di Cauchy
  24. Il teorema ponte
  25. Teoria sui limiti
  26. Simboli di Landau
  27. Funzioni continue – Teoria
  28. Il teorema di Weierstrass
  29. Il teorema dei valori intermedi
  30. Il teorema della permanenza del segno
  31. Il teorema di Heine-Cantor
  32. Il teorema di esistenza degli zeri
  33. Il metodo di bisezione
  34. Teorema ponte versione per le funzioni continue
  35. Discontinuità di funzioni monotone
  36. Continuità della funzione inversa
  37. Teorema delle contrazioni o Teorema di punto fisso di Banach-Caccioppoli
  38. Teoria sulle derivate
  39. Calcolo delle derivate: la guida pratica
  40. Teoria sulle funzioni convesse
  41. Il teorema di Darboux
  42. I teoremi di de l’Hôpital
  43. Teorema di Fermat
  44. Teoremi di Rolle e Lagrange
  45. Il teorema di Cauchy
  46. Espansione di Taylor: teoria, esempi e applicazioni pratiche
  47. Polinomi di Taylor nei limiti: istruzioni per l’uso
  48. Integrali definiti e indefiniti
  49. Teorema fondamentale del calcolo integrale (approfondimento)
  50. Integrali ricorsivi
  51. Formule del trapezio, rettangolo e Cavalieri-Simpson
  52. Teoria sugli integrali impropri
  53. Funzioni integrali – Teoria
  54. Introduzione ai numeri complessi – Volume 1 (per un corso di ingegneria — versione semplificata)
  55. Introduzione ai numeri complessi – Volume 1 (per un corso di matematica o fisica)
  56. Serie numeriche: la guida completa
  57. Successioni di funzioni – Teoria
  58. Teoremi sulle successioni di funzioni
    1. 58a. Criterio di Cauchy per la convergenza uniforme
    2. 58b. Limite uniforme di funzioni continue
    3. 58c. Passaggio al limite sotto il segno di integrale
    4. 58d. Limite uniforme di funzioni derivabili
    5. 58e. Piccolo teorema del Dini
    6. 58f. Procedura diagonale e teorema di Ascoli-Arzela
  59. Serie di funzioni – Teoria
  60. Serie di potenze – Teoria
  61. Serie di Fourier – Teoria e applicazioni
  62. Integrali multipli — Parte 1 (teoria)
  63. Integrali multipli — Parte 2 (teoria e esercizi misti)
  64. Regola della Catena — Teoria ed esempi.
  65. Jacobiano associato al cambiamento di coordinate sferiche
  66. Guida ai Massimi e Minimi: Tecniche e Teoria nelle Funzioni Multivariabili
  67. Operatore di Laplace o Laplaciano
  68. Teoria equazioni differenziali
  69. Equazione di Eulero
  70. Teoria ed esercizi sulla funzione Gamma di Eulero
  71. Teoria ed esercizi sulla funzione Beta
  72. Approfondimento numeri complessi
  73. Diverse formulazioni dell’assioma di completezza
  74. Numeri di Delannoy centrali
  75. Esercizi avanzati analisi

 
 

Tutte le cartelle di Analisi Matematica

Leggi...

  1. Prerequisiti di Analisi
    1. Ripasso algebra biennio liceo
    2. Ripasso geometria analitica
    3. Ripasso goniometria e trigonometria
    4. Errori tipici da evitare
    5. Insiemi numerici N,Z,Q,R
    6. Funzioni elementari
    7. Logica elementare
    8. Insiemi
  2. Successioni
    1. Teoria sulle Successioni
    2. Estremo superiore e inferiore
    3. Limiti base
    4. Forme indeterminate
    5. Limiti notevoli
    6. Esercizi misti Successioni
    7. Successioni per ricorrenza
  3. Funzioni
    1. Teoria sulle funzioni
    2. Verifica del limite in funzioni
    3. Limite base in funzioni
    4. Forme indeterminate in funzioni
    5. Limiti notevoli in funzioni
    6. Calcolo asintoti
    7. Studio di funzione senza derivate
    8. Dominio di una funzione
    9. Esercizi misti Funzioni
    10. Esercizi misti sui Limiti
  4. Funzioni continue-lipschitziane-holderiane
    1. Teoria sulle Funzioni continue-lipschitziane-holderiane
    2. Continuità delle funzioni
    3. Continuità uniforme
    4. Teorema degli zeri
    5. Esercizi sul teorema di Weierstrass senza l’uso delle derivate
  5. Calcolo differenziale
    1. Derivate
    2. Calcolo delle derivate
    3. Retta tangente nel calcolo differenziale
    4. Punti di non derivabilità nel calcolo differenziale
    5. Esercizi sul teorema di Weierstrass con l’uso delle derivate
    6. Studio di funzione completo nel calcolo differenziale
    7. Esercizi teorici nel calcolo differenziale
    8. Metodo di bisezione
    9. Metodo di Newton
  6. Teoremi del calcolo differenziale
    1. Teoria sui Teoremi del calcolo differenziale
    2. Teorema di Rolle
    3. Teorema di Lagrange
    4. Teorema di Cauchy
    5. Teorema di De L’Hôpital
  7. Calcolo integrale
    1. Integrale di Riemann
    2. Integrali immediati
    3. Integrale di funzione composta
    4. Integrali per sostituzione
    5. Integrali per parti
    6. Integrali di funzione razionale
    7. Calcolo delle aree
    8. Metodo dei rettangoli e dei trapezi
    9. Esercizi Misti Integrali Indefiniti
    10. Esercizi Misti Integrali Definiti
  8. Integrali impropri
    1. Teoria Integrali impropri
    2. Carattere di un integrale improprio
    3. Calcolo di un integrale improprio
  9. Espansione di Taylor
    1. Teoria Espansione di Taylor
    2. Limiti di funzione con Taylor
    3. Limiti di successione con Taylor
    4. Stime del resto
  10. Funzioni integrali (Approfondimento)
    1. Teoria Funzioni integrali (Approfondimento)
    2. Studio di funzione integrale
    3. Limiti con Taylor e De L’Hôpital
    4. Derivazione di integrali parametrici (Tecnica di Feynmann)
  11. Numeri Complessi
    1. Teoria Numeri complessi
    2. Espressioni con i numeri complessi
    3. Radice di un numero complesso
    4. Equazioni con i numeri complessi
    5. Disequazioni con i numeri complessi
    6. Esercizi misti Numeri complessi
  12. Serie numeriche
    1. Teoria Serie numeriche
    2. Esercizi Serie a termini positivi
    3. Esercizi Serie a termini di segno variabile
    4. Esercizi Serie geometriche e telescopiche
  13. Successioni di funzioni
    1. Teoria Successioni di funzioni
    2. Esercizi Successioni di funzioni
  14. Serie di funzioni
    1. Teoria Serie di funzioni
    2. Esercizi Serie di funzioni
  15. Serie di potenze
    1. Teoria Serie di potenze
    2. Esercizi Serie di potenze
  16. Serie di Fourier
    1. Teoria Serie di Fourier
    2. Esercizi Serie di Fourier
  17. Trasformata di Fourier
    1. Teoria Trasformata di Fourier
    2. Esercizi Trasformata di Fourier
  18. Funzioni di più variabili
    1. Teoria Funzioni di più variabili
    2. Massimi e minimi liberi e vincolati
    3. Limiti in due variabili
    4. Integrali doppi
    5. Integrali tripli
    6. Integrali di linea di prima specie
    7. Integrali di linea di seconda specie
    8. Forme differenziali e campi vettoriali
    9. Teorema di Gauss-Green
    10. Integrali di superficie
    11. Flusso di un campo vettoriale
    12. Teorema di Stokes
    13. Teorema della divergenza
    14. Campi solenoidali
    15. Teorema del Dini
  19. Equazioni differenziali lineari e non lineari
    1. Teoria equazioni differenziali lineari e non lineari
    2. Equazioni differenziali lineari e non lineari del primo ordine omogenee
  20. Equazioni differenziali lineari
    1. Del primo ordine non omogenee
    2. Di ordine superiore al primo,a coefficienti costanti,omogenee
    3. Di ordine superiore al primo,a coefficienti costanti,non omogenee
    4. Di Eulero,di Bernoulli,di Clairaut,di Lagrange e di Abel
    5. Non omogenee avente per omogenea associata un’equazione di Eulero
    6. Sistemi di EDO
  21. Equazioni differenziali non lineari
    1. A variabili separabiliO
    2. A secondo membro omogeneo
    3. Del tipo y’=y(ax+by+c)
    4. Del tipo y’=y(ax+by+c)/(a’x+b’y+c’)
    5. Equazioni differenziali esatte
    6. Mancanti delle variabili x e y
    7. Cenni sullo studio di un’assegnata equazione differenziale non lineare
    8. Di Riccati
    9. Cambi di variabile: simmetrie di Lie
  22. Analisi complessa
    1. Fondamenti
    2. Funzioni olomorfe
    3. Integrale di Cauchy e applicazioni
    4. Teorema della curva di Jordan e teorema fondamentale dell’Algebra
    5. Teorema di inversione di Lagrange
    6. Teorema dei Residui
    7. Funzioni meromorfe
    8. Prodotti infiniti e prodotti di Weierstrass
    9. Continuazione analitica e topologia
    10. Teoremi di rigidità di funzioni olomorfe
    11. Trasformata di Mellin
  23. Equazioni alle derivate parziali
    1. Equazioni del primo ordine
    2. Equazioni del secondo ordine lineari
    3. Equazioni non-lineari
    4. Sistemi di PDE
  24. Funzioni speciali
    1. Funzione Gamma di Eulero
    2. Funzioni Beta,Digamma,Trigamma
    3. Integrali ellittici
    4. Funzioni di Bessel
    5. Funzione zeta di Riemann e funzioni L di Dirichlet
    6. Funzione polilogaritmo
    7. Funzioni ipergeometriche
  25. Analisi funzionale
    1. Misura e integrale di Lebesgue
    2. Spazi Lp,teoremi di completezza e compattezza
    3. Spazi di Hilbert,serie e trasformata di Fourier
    4. Teoria e pratica dei polinomi ortogonali
    5. Spazi di Sobolev
  26. Complementi
    1. Curiosità e approfondimenti
    2. Compiti di analisi
    3. Esercizi avanzati analisi
  27. Funzioni Convesse

 
 

Tutti gli esercizi di geometria

In questa sezione vengono raccolti molti altri esercizi che coprono tutti gli argomenti di geometria proposti all’interno del sito con lo scopo di offrire al lettore la possibilità di approfondire e rinforzare le proprie competenze inerenti a tali argomenti.

Strutture algebriche.





 
 

Risorse didattiche aggiuntive per approfondire la matematica

Leggi...

  • Math Stack Exchange – Parte della rete Stack Exchange, questo sito è un forum di domande e risposte specificamente dedicato alla matematica. È una delle piattaforme più popolari per discutere e risolvere problemi matematici di vario livello, dall’elementare all’avanzato.
  • Art of Problem Solving (AoPS) – Questo sito è molto noto tra gli studenti di matematica di livello avanzato e i partecipanti a competizioni matematiche. Offre forum, corsi online, e risorse educative su una vasta gamma di argomenti.
  • MathOverflow – Questo sito è destinato a matematici professionisti e ricercatori. È una piattaforma per domande di ricerca avanzata in matematica. È strettamente legato a Math Stack Exchange ma è orientato a un pubblico con una formazione più avanzata.
  • PlanetMath – Una comunità collaborativa di matematici che crea e cura articoli enciclopedici e altre risorse di matematica. È simile a Wikipedia, ma focalizzata esclusivamente sulla matematica.
  • Wolfram MathWorld – Una delle risorse online più complete per la matematica. Contiene migliaia di articoli su argomenti di matematica, creati e curati da esperti. Sebbene non sia un forum, è una risorsa eccellente per la teoria matematica.
  • The Math Forum – Un sito storico che offre un’ampia gamma di risorse, inclusi forum di discussione, articoli e risorse educative. Sebbene alcune parti del sito siano state integrate con altri servizi, come NCTM, rimane una risorsa preziosa per la comunità educativa.
  • Stack Overflow (sezione matematica) – Sebbene Stack Overflow sia principalmente noto per la programmazione, ci sono anche discussioni rilevanti di matematica applicata, specialmente nel contesto della scienza dei dati, statistica, e algoritmi.
  • Reddit (r/Math) – Un subreddit popolare dove si possono trovare discussioni su una vasta gamma di argomenti matematici. È meno formale rispetto ai siti di domande e risposte come Math Stack Exchange, ma ha una comunità attiva e molte discussioni interessanti.
  • Brilliant.org – Offre corsi interattivi e problemi di matematica e scienza. È particolarmente utile per chi vuole allenare le proprie capacità di problem solving in matematica.
  • Khan Academy – Una risorsa educativa globale con lezioni video, esercizi interattivi e articoli su una vasta gamma di argomenti di matematica, dalla scuola elementare all’università.






Document









Document